AMA | November 2023

Welcome to the November 2023 Ask Me Anything episode of Mindscape! These monthly excursions are funded by Patreon supporters (who are also the ones asking the questions). We take questions asked by Patreons, whittle them down to a more manageable number -- based primarily on whether I have anything interesting to say about them, not whether the questions themselves are good -- and sometimes group them together if they are about a similar topic. Enjoy!

AMA

Support Mindscape on Patreon.

AMA November 2023

Roland Weber
Will "The Biggest Ideas in the Universe, Part 2" be comprehensible by itself, or do you expect readers to have worked through part 1 already?

Tim Gianitsos
Barry Loewer said near the end of your podcast with him that the many worlds interpretation of quantum mechanics leads us to "take probabilities out of the world and treat them entirely as indexicals". What does this mean, why does he consider it a problem, and why do you not consider it a problem?

Seth
I’ve been watching the tv show Devs which includes the idea that using quantum computers they are able to project what happens both in the past and the future given total knowledge of the current state of things in the universe now and also relies on the idea that things are deterministic. My understanding is that at the quantum level things are not deterministic but probabilistic could you expand a little on quantum theory and it’s effect of how it affects determinism being true or not and if it would be possible to project backwards or forward given enough information on the current state of affairs?

****************************
Sandro Stucki
How do you reconcile being a poetic naturalist with being a reality realist as opposed to a mathematical realist. If tables and chairs are real because they provide a useful way of talking about the world, then why would vector spaces and Riemannian manifolds not be real? After all, they have proven to be very useful ways to describe the world. And if those are real, why not sets and numbers, out of which we construct vector spaces & co?

Leo Behe
I was a big fan of Our Mathematical Universe, the book by Max Tegmark (previous Mindscape guest). In the book, he talks about how, as we look closer, the basic constituents of the universe like mass and charge seem to only be describable by their relations to other similarly mathematical entities. Are you sympathetic to the view that the universe (at least as far as we can understand it) is fundamentally mathematical, or do you think there's likely to be some basic "stuff" with qualities that can't be mathematically quantified?

P Walder
Within the poetic naturalist view of reality where do abstract entities such as information, knowledge, and numbers fit? Are such entities a category of ‘thing’ that are to be considered just as real as matter/energy and therefore of equal importance in accounting for the reality we experience?
*******************************

looseleaf
Obviously you are someone who thinks very deeply about philosophy and have your own unique understanding about meanings of life. So I'm curious would you apply your own unique values to your romantic relationship and other close relationships in your life? Or would you rather go along with, or at least empathize with the more popular values that most people tend to have? And if you were to raise a child, would you try to convince him/her of you own worldview or would you try to let him/her explore the world freely without exposing him/her to your ideas?

Keith
Thanks to your AMA post [Patreon perk shoutout] I'm sitting here trying to imagine the space of all unasked questions. For an AMA where all questions are finite length and written in a finite alphabet, at least its finite. My question is: What additional constraints on this space really narrows it down to the subspace of what would be considered questions? For example, maybe a question is something with a conceivable answer or response.

Jon Schoning
I've seen other sources that claim the magnetic field is really just a derived effect from viewing an electric field from a different relativistic frame. If this is true, can it really be said there is a magnetic field fundamentally, or are there just electric fields plus relativity?

Roo Phillips
Can you talk about the future of many worlds? What would have to be true, or what milestones do you think we'll need to achieve, for the majority of the physics community to adopt many worlds as a proper model of foundational physics?

Nick C.
When one thinks back in time toward the big bang there's probably a lot of agreement back until at least elctro-weak unification, but as you go further back there's presumably a lot less agreement and experimental/observational proof and things get more speculative. How do you think one should address this when communicating with the public who don't necessarily know what's speculative and what's not? It seems like sometimes scientists speaking to the public will talk about their own highly-speculative idea in the same manner as ideas that are much more established.

Brendan
I realize you have a few trade books and textbook in the works, but have you ever considered writing science books for children? I think you have an amazing ability to convey complex concepts in a way that is easy to understand, and that seems like it would help to make really good science children books.

Jeffrey Segall
In the past, I have read speculations about manipulating black holes by making them charged and then moving them using electric fields. However, I gather that electric interactions can be expressed in terms of exchanges of photons. I was curious then how to imagine the photon exchanges with black holes if nothing ever leaves a black hole except for Hawking radiation. Do all the interactions of a black hole with an electric field involve photons getting trapped by the black hole, independent of whether the black hole is being repelled or attracted by an external electric field?

************************
Dylan Hall
Is there any explanation at all for 3 generations of matter, and specifically why it's only fermions and not bosons even though some bosons have mass? Is it inherent in the Pauli exclusion principle somehow? Feels like there's an important connection I'm missing.

Kevin O'Toole
Is there evidence against quarks heavier than top quarks, or are there theoretical arguments against them? Naively, it seems more natural to have an infinite series of ever-heavier particles than to have exactly 3 levels of quarks and leptons!
**************************

AJ
I think you've said something to the effect of free will is real but not fundamental. That makes sense to me right now, but assuming that we are at some future point able to explain the macro phenomenon of free will from whatever constitutes it, would it still be right to call it "free" will? That is, doesn't the compatibilist position breakdown at some point?

Spencer Hargiss
If a photon of light hits a telescopes CCD, what was the shape of it's wave function right before that? Is it the surface of a sphere or hemisphere the radius of the distance to the star?

SummDude
Are you a gamer? Of any sort? Video games, tabletop, sports, whatevs.

Ahmad Chaker
Is there a deeper physical reason why there no massless fermions?

Go Mezant
Under multiverse theory are all possible truths real?

Blake Brasher
Do you collect, or have you ever bought original artwork such as paintings or sculptures? If so, how do you go about finding the artwork you'd like to acquire and where do you keep it?

Alex Miller
Are there any works of literature which you have found genuinely productive in thinking about the nature of a fundamental reality? The Borges story "Tlon, Uqbar, Orbis Tertius," which you discussed in your most recent episode was one such story for me. As well as one I came to after Leonard Susskind praised it at the outset of a lecture: Kafka's "Investigations of a Dog."

Mike Mayer
What is the least interesting research project you have worked on? Or are there any publications of yours that you are not very fond of anymore?

Nalita S
Do you see a future where the basics of QM would be incorporated in schools’ curriculum?

Elif and Lucas Wagner
This is about the last section of Something Deeply Hidden. You mentioned that time and space might be emergent from something else. Do you have any thoughts/ideas on how one might represent (as opposed to position, momentum, etc) a wave function from which space and time is emergent? Thanks!

Eduard Sackinger
To prove Noether's theorem, which relates symmetries to conserved quantities, we assume that the equations of motion can be derived from the principle of least action. This is great for classical mechanics, but in quantum mechanics the Feynman path integral also includes contributions from paths other than the one with least action. Could you please comment on the applicability of Noether's theorem to quantum physics?

Mark
Do you care how and for how long you will be remembered and if so or not, why?

Travis Martin
What is science?

Craig Stevens
Scientists at Cornell recently announced the highest resolution photo of an atom ever taken. Given that atoms are not really particles, but instead probability distributions of waves, what exactly are they taking a photo of and what do they hope to see?

Michael Mohnhaupt
Does the collapse of the wave function affect the gravitational field of the particle, that is all of a sudden in a particular location?

Johan Falk
I have for some time been trying to understand why AI experts make radically different assessments concerning AI risks. I get that people who don't know a lot about AI have different conclusions, but I thought experts would agree more – or at least not disagree wildly.
I have two main hypotheses, and would love to hear your thoughts:
1) Being an expert in AI technology does not automatically make you an expert in AI risks.
2) People have different predispositions when it comes to perceiving and reacting to low-probability risks.

Aron Berger
I have heard that a scalar field is an object with some value at every point in space. I have also heard that according to some inflationary theories, we could live in a part of a cosmological multiverse in which there numerous other causally separate universes. If these two propositions are true, doesn't that imply that every universe in the cosmological multiverse would have the same set of scalar fields?

Sean B
Sorry if this is a naive question, but I've been pondering the relationship between the Many-Worlds Interpretation (MWI) of quantum mechanics and the nature of dark matter. Given that dark matter particles are massive yet weakly interacting, could they be related to particles from separate branches of the wave function in MWI?

LE McAndrew
Is it just a philosophical question or a reality as things line up within the Judicial, Legislative, and may soon be the Executive branches of our government to destroy democracy? There appears to be such a headwind against the Rule of Law and facts that it is hard to see a clear way ahead.

***********************************
John Hague
Knock Knock.
Who is there?
Linda.
Linda who?
Will you Lind-us a hand by clarifying the Linda example used by David Deutsch please?

Benjamin Barbrel
One thing I enjoy a lot about your podcast is how your teaching skills help me understand your guests arguments (for instance when you rephrase their thoughts in your own words or ask the right question) even when the subject is quite subtle.
I must admit that you somewhat failed me in the recent David Deutsch episode!
Several parts remained quite obscure to me, which is frustrating because the topics felt very interesting. Do you think you could reformulate his point about the non-transitivity of support? That feminist/banker argument he used stumped me, despite careful re-listenings.
********************************

Orin Harris
PRIORITY QUESTION
Bohr-like complementarity in QM and the relativism of SR/GR are similar in their lack of realism about observer states. E.g. "Wigner’s Friend" vs relativity of simultaneity or "black hole complementarity" for infalling vs external observers, etc. And yet for us realists, Everett provides a realist account of QM, while I’m aware of no analogous realist account of SR/GR complementarity. So: if you are unhappy with antirealist accounts of QM, why are you not similarly unhappy with the current metaphysical account of SR/GR?

Ari Moudi
My elderly mom (89 years old) is a fundamentalist Christian and is convinced we are in the end times. I am an atheist who rejects all of this. She’s at the phase in her life where she’s devastated that I’ll be in hell for eternity and tells me this all the time. I’m struggling how to deal with this heavy weight on her heart. Any advice?? Every time we speak, she begs me to renounce my atheism and this is a horrible way to live for both of us.

Ken Schneider
What is the significance of the fact that the Standard Model can't be quantized?

Rob Gebele
Could there be a consistent theory of gravity allowing also for anti-gravity, i.e. the force being repulsive instead of attractive for some particles (even if it doesn’t apply to our real world), which is compatible with both relativity and quantum mechanics (at least in a low energy EFT regime)?

Ken Wolfe
I have heard the mechaism of Hawking radiation described as a particle pair arising at the event horizon where a particle with negative mass falls into the black hole and a particle with positive mass escapes. Is this a valid way of looking at it, and if not why?

Bob Zannelli
Axions are a leading cold dark matter candidate. What process can create such low mass particles as cold dark matter; one would think they would be highly relativistic, like neutrinos?

Dave Grundgeiger
What would you do differently if you became 100% independently wealthy and had the unconstrained ability to fund your own research, including hiring researchers and buying equipment? Would you create your own research institute? Retire? Something else?

Adam Rotmil
If the theory of eternalism holds up, where moments in the past present and future are all equally real, then what happens to our sense of identity? Like are those parts of ourselves “still” conscious in their local time coordinates? I suppose Hume called this the notion of resemblance, that we continue to resemble ourselves, even though we keep changing. But you’ve said in the past that different branching copies of ourselves are “different people” — so where would we draw the line on identity and why?

Chris Rodgers
Beyond the event horizon, does a black hole have a surface?
If not, why not, coz I don’t understand.

Valinorean
PRIORITY QUESTION - What do you think of my new past-eternal cosmological model, https://en.wikipedia.org/wiki/Emergent_Universe#%22Rube_Goldberg_cosmology%22_scenario (Don Page said it, quote, looks very interesting, and George Ellis, Graham Oppy, and Avi Loeb concurred) - In it, *all* ancient Cauchy surfaces (of the default kind) are as nice as the bounce surface in your (Carroll-Chen) model, and there is no reversal of the arrow of time!

Adam Small
I was having a conversation here with the community about emergence and that it appears from a whole bunch of basic units of whatever macroscopic thing you're talking about. So, then I said do a thought experiment and remove one of the particles or units at a time until you don't get emergent behavior. One of the smart members said that's called the Sorites Paradox. Just wondering how you think about that and what it means. It seems to really mess up the concept of emergence since by definition it can't depend on a single particle or unit of the whole.

Eliot Specht
I enjoyed your podcast with Mari Ruti in which you discussed Lacan’s version of psychoanalysis. The analysis is based on universal origins for human angst posited by Lacan. You asked Ruti whether this theory was scientific (i.e. verifiable), and she conceded that it was not – Ruti described it as mythology. This answers just half the question. There are certainly useful ways of talking that are not scientific. My question: should critics (such as you) demand that scientific techniques be used to determine whether Lacan’s analysis really is useful? Anecdotal accounts seem like a weak basis for choosing a medical treatment.

Tim Converse
Great to hear about your collaborations with Scott Aaronson.
I understand from Aaronson's popularizations that there are hard physics-based limits on how much computation you can do in a region of space.
My question is: if some civilization were trying to max out their computational resources, and driving right up to physical limits, might there be observational signatures that would indicate that was happening? Something analogous to observing Dyson spheres, but for computation rather than for energy.

Casey Mahone
If I'm doing the math correctly, you met your wife relatively late in your life. Did you ever feel hopeless about finding love? I don't know if you're familiar with the current dating climate at all, but the world of dating apps seems like a complete waste of time, and it feels challenging to meet other single people once you are out of school. Any words of wisdom?

Dan Berliner
You mentioned in an earlier AMA that one of the ways gravity is special is that it interacts with everything. Does this mean gravitons would interact with themselves and, if so, would they not experience confinement in the same way as gluons?

Mike McManus
Is supersymmetry dead?

Jeff B
I'm trying to wrap my head around emergent spacetime. I think I understand that you are defining the wavefunction to exist in a more abstract space of "degrees of freedom" and these degrees of freedom happen to be connected in such a way that they produce the appearance of spacetime dimensions. My question is: if the degrees of freedom are so perfectly coordinated that they mimic spacetime geometry, then wouldn't it be more logical to simply assert that there is something called spacetime which ties these degrees of freedom together?

ValorUp
If space flight in the near future becomes commercially feasible would you participate? Would you visit a space hotel or a hotel on the moon? What sort of fun and interesting attractions could you see being presented?

hailzeus
I really enjoyed Something Deeply Hidden, and I have a question that I don’t think you explicitly addressed in your book (my apologies if you did!). When solving for the time of flight of a thrown ball, we get one positive and one negative answer, and we discard the negative answer as non-physical, which no one loses sleep over. There are numerous other examples in physics of similarly throwing out “wrong” answers. By analogy then, what is the issue with throwing out some of the answers to the Schrödinger equation? I think I understand the logic of “if the Schrödinger equation is perfect and all-encompassing, then many-worlds” but the “if” statement here seems to be carrying an awful lot of water.

onlynormalperson
Could you elaborate on how moral constructivism gets through "ought from is?" It seems like an accurate way to describe constructivism would be, "It *is* true almost all humans have moral inclinations even if specific intuitions are not universal, so we *ought* to work to make our inclinations as coherent and consistent as possible."
That certainly works for me intuitively, but I feel like I must be missing a piece of formal logic based on your other statements.

Laurent Delamare
I enjoyed watching the youtube replay of your debate on "Is Consciousness Fundamental?" with Philip Goff on Sept 8th. Your humor makes dry subjects so much easier to understand! I may be biased but I feel like Philip's way to convince people that he is right is by being super passionate at exposing strange arguments. My question to you is why do you think he never answers you when you ask whether he wants to change the core theory or not change it? It seems a simple enough question! Did he answer that elsewhere?
Also, the replay didn't say whether there was a poll of the audience at the end, and who won 🙂

Go Mezant
There's a set of philosophers/scientists out there that believe that Free Will is an illusion. Sopolsky comes to mind as a champion of this view. What I've never heard them explain the Hitler Problem, as I call it, relative to the non existence of free will. That is, doesn't the lack of free will give Hitler a pass?

Daniel Bagley
I'm a layperson reading Hawking's A Brief History of Time. In the chapter on the origin and fate of the universe, when he talks about the inflationary period after the big bang, he says that different regions should not have been able to achieve the uniformity we see now because the information would not have been able to travel faster than the speed of light. But couldn't entanglement have played a role? Such that particles weren't exchanging information but were simultaneously affecting each other. Or does this represent a fundamental misunderstanding of entanglement?

Mark Smith
please could you help me understand why the thermodynamic arrow of time is necessary to explain why we perceive there to be a past and future? We are part of the universe and the act of us experiencing something and it being committed to our memory is itself a physical process - a physical process that (like all physical processes) is reversible. I appreciate that the thermodynamic arrow of time explains why it is overwhelmingly unlikely that a physical process that is as complicated as seeing egg breaking and creating a memory of that would happen “in reverse” - but it seems to me that a more fundamental point is that even if that physical process were to happen in reverse, then we wouldn’t have the memory.

Filip Malinowski
In conversation with David Deutsch he was talking about an Everettian interpretation of QM in Heisenberg picture, when observables are evolving instead of the wavefunction. At a level of basic QM the Shroedinger and Heisenberg pictures are equivalent. But they seem to suggest different interpretations of QM. In your favorite case of many-world interpretation, is there a difference between Shroedinger-Everett and Heisenberg-Everett? It feels, for example, that Bohmian interpretation would be hard to reconcile with a Heisenberg picture.

Jason Ricciardi
I liked your X post on 10/10/23 that states, "Responses to this have been kind of depressing. I have my own degree of skepticism, but there’s been way too much superficial dismissal. Ambitious new theories will often seem weird. Knee-jerk reactions make it harder to investigate truly novel ideas." Can you explain how Dr. Cronin's Assembly Theory & their tech stack at Chemify challenges the status quo, what are the novel ideas versus already established ideas, & why has it caused friction in the community.

Castor Pollux
Is there a way to revert a black hole back to normal matter, like neutron star or just a regular star? Or, once you go black you never go back? I'm talking about the theoretical possibility not a practical solution.

giorgio
I was wondering how you on a personal level deal with the discrepancy of personal well being and the suffering of so many people around the world.
How can I ever be happy, knowing that there are humans around the world who never in their live have felt secure or protected and live in constant fear of pain or worse.
Especially of course in regards to resent events around the world I often find myself incapable of enyojing my own life that is so very different.
Do you struggle with similiar feelings and if so how do you deal with that?

Kevin Contreras
I have a philosophical question regarding maths. Thinking about how mathematical models can make sense internally but don't correspond with our experience of the world or just can't be proven, it makes me wonder , why does this happen? Its maybe like math is like any other language in the sense that it can't really grasp concepts outside of our experience, like they're just a an extension of our own capacity and by default prone to our own errors? Or do they really are something separate that we are just not interpreting correctly?

Kris V
It can be argued that it is morally possible to become a billionaire. However, do you think it is morally defensible to remain one over time (a billion simply as an arbitrary threshold).

Walter E Miller
Is there a magnetic field that permeates the entire universe? If so, what generates that field?

Joe Ader
Do you believe the lone genius drives scientific progress more than collaborative science? Do you think this has shifted over time? Do you think awards like the Nobel Prize are helping or hurting wrt this?

Adam Small
If a bubble chamber was placed between the slits and the screen in a double slit experiment and the same particles that produce tracks in a bubble chamber are passed through the slits, would you see interference or would you see the tracks as they emerge from the slits? The slits don't have detectors in the setup.

Yasas
PRIORITY QUESTION: Can the CMB (remnant energy from the early universe) be the reason for dark matter? If not, why?

Ryan Santos
As an educator do you find that variety in coursework and school activities improve student performance? If so do you have any theories as to why this is?

Owe
I'm trying to spin up on Postmodern philosophy and was reading through the Wikipedia page as a starter. It states "Many postmodernists appear to deny that an objective reality exists, and appear to deny that there are objective moral values". The latter seems reasonable to me, but the latter seems pretty hard to take at face value.
What's the basis for rejecting an objective reality? Is it anything other than a reducto-ad-absurdium ish take that the existence of reality can't *truly* be proven? How is this useful in practice?

Peter Gallay
Please explain how inflationary theory implies other universes (if indeed it does).

Qubit
While trying to understand the concept of effective field theories, I wondered about the following point. You mentioned, that the infinities of quantum field theoretic calculations disappear when formulated on a discrete lattice. Having some experience in numeric simulations, that doesn't make any sense to me. Is it really the case, that I am not allowed to choose an arbitrarily fine gird within my simulation, because otherwise my results become worse at some unknown point?

Colleen Edwards
With psychedelic drugs like MDMA, Mescaline, Psilocybin, LSD, etc. being studied more as potential therapies for treating PTSD, depression and anxiety disorders, etc. I’m curious what your take is on this- if you’re for it or against it and why? Have you ever experimented and if so we’re you lucky enough to have had a profound experience on your trip?

Kyle Stephens
Isaac Asimov’s “Foundation” series describes a future where knowledge becomes fragmented and specialized to the point where scientists become increasingly focused on narrow topics and the broader understanding of science is lost.
Do you think this fragmentation is happening today? Is any of your motivation to be cross-disciplinary to keep this from happening?

Robert Ruxandrescu
When you talk about Schrodinger’s cat you present the two scenarios where the cat is awake and standing and asleep and sitting and you mention that decoherence happens and the atoms in the cat are different in the two scenarios.
But what if the awake and asleep cat are in the exactly the same position - maybe both of the two versions are sitting still with their eyes closed, making the same exact noise, breathing the same and so on. The only difference is their conscious state - one is awake, the other is asleep.
What then?

Erik Fast
I know that I should have a credence of 1 in 1024 that a fair coin will land heads 10 times in a row. I might have the exact same credence that we will prove the twin prime conjecture in the next century, but these intuitively feel like very different types of credences. In the second example, I'm uncertain whether I should have chosen something closer to 1 in 10 or 1 in 10 million. Is it enough to represent a credence as a number, or should we use something like a set of probability distributions that capture our higher-order uncertainties?

Carlos Nunez
Robert Sapolsky just published a book showing that we don't have free will. Given your compatibilist stance, what evidence would persuade you to reject the notion of free will?

Blake R Suhre
Is irreversibility just a function of our ignorance, or fundamental (a la Prigogine)?

Zachary Danziger
How useful do you think cellular automata are as a metaphor for physical processes or computation (e.g., Conway's game of life) or as the actual substrate for a theory of physical law (e.g., Wolfram's Ruliad)?

Wayne J2 B
Nobel prizes loom large in the public imagination. Is that true for physicists as well, especially when they start out as grad students or junior faculty? To what extent was your career affected by a desire to work on problems or go to places where you'd have a shot a doing work ground-breaking enough to give you a chance at a Nobel? And finally do secretly hope that your work in the foundations of physics will lead to a Nobel?

Nick Gall
In your episode with Tyler Cohen you said, "We still need to understand what the laws of physics are that push the wave function of the universe forward, even once you've given the many worlds." Could you say more about what laws of physics are needed to understand what drives the dynamics of the wave function, ie provides the "oomph"?

Rad Antonov
Do you have a favorite topic to teach?

Chris Murray
In The Road to Reality, one objection Roger Penrose makes to many-worlds is that an entangled state can be represented equally well as sums of many different pairs of states, and that there is no mathematical preference for which pair should count as the possible observations. What say you? Even if correct, wouldn't this problem be shared by all interpretations?

physics_kitten
Can you help me understand why quantum computers are so much faster, at least for some applications? It can't be the amount of information being handled,--a bit has two possibilities (zero and one), and a Qbit just has one more possibility (spin up, spin down, and superposition of spin up and down).

Amy Ferguson
When it comes to your thought process, do you feel as if your thoughts emerge out of 'nothingness,' or is there a sense that you actively 'construct' them?

Brandon Lewis
Suppose I have a collection of nuts and bolts placed randomly into little bins. If I sort the collection by size, thread pitch, head type, etc, has the mass of the collection increased by the corresponding decrease in entropy? Or is this just a complete misunderstanding or the way entropy works?

Sid Huff
Stephen Hawking once declared that “Philosophy is dead, because it has not kept up with modern developments in science, particularly physics.” For saying this, others accused Hawking of “scientism,” i.e., putting an overly high value on science relative to other non-scientific disciplines. But is this really a bad thing? If, say, physics is a better way to understand the world, should it in fact be more highly valued than, say, philosophy and other such primarily non-scientific disciplines?

0:00:00.0 Sean Carroll: Hello everyone. Welcome to the November, 2023. Ask me Anything edition of the Mindscape Podcast. I'm your host, Sean Carroll. Not a lot of things to share with you before we get into it today, but I did think that this might be an effective place to stop and reflect a bit on teaching, which I'm doing again now. When I was at Caltech, which I was, when I started Mindscape five years ago, I basically didn't do any teaching. I taught one course the entire time I was at Caltech for whatever it was, 16 years. And of course, I'd done plenty of teaching before that at the University of Chicago, and even a little bit at MIT and Harvard. But it's been a while. I'm rusty, right? I'm outta practice. And more importantly, the kind of teaching I have been doing so far at Johns Hopkins has been different than what I am familiar with from my previous experience, which was mostly lecturing advanced physics topics.

0:00:55.5 SC: At the University of Chicago, for example, when you're a theoretical physicist, you are almost always assigned graduate level courses to teach. They have that, informal distinction there. I did teach a couple of undergraduate courses. I taught a, I started a course on undergraduate general Relativity, which is very popular. And Shetty Buche, who was a former Mindscape guest, she and I co-taught a course in a humanities course on moments in atheism, on the intellectual history of atheism throughout the ages. And that was kind of a classic thing where it was supposed to be a seminar, but then a lot of people wanted to take it. And we ended up letting a lot of people take it, and then it just turns into lectures. You can't have 40 people in the room and give everyone really a full chance to speak. So I haven't had this experience before that I've had both this year and the last year at Hopkins of really just sitting in the room with 12 people and have it truly be a discussion-based seminar, which I always love.

0:01:58.0 SC: Those are always my favorite courses. When I was an undergraduate. I do have very strong opinions that a good old fashioned lecture is still an awesome paradigm for teaching. I remember once when I was, not long after I had been at MIT and I taught a big lecture course there in general Relativity, which later turned into the textbook that I wrote, space Time and Geometry. And I was mentioning this to someone who was an expert in education, who had actually, had a degree in education. And they said, well, what kind of course was it? And I said, well, it was a graduate course in general relativity. Basically, I stood up and I taught them general relativity just via lectures. And the response was, well, that's not really teaching at all. [laughter] that doesn't count as teaching to them. If you're not interacting, if you're not having discussions, etcetera, then it's not really teaching at all.

0:02:51.5 SC: And I thought that was kind of a silly attitude to take. As I am with many things, I'm kind of a plural list about teaching methods. If you want to teach general relativity, I think that probably the best thing is to give them the content via some method, whether it's lecturing or anything else. Now, I know that not everyone agrees. Andrew Strominger, who was a former Mindscape guest, famously would have student, he would teach graduate general relativity, and he would have students do problems in real time in the class. That does save you some lecturing, preparation time if you're the professor. But I think that there's just so much stuff you gotta get across that I have no objections whatsoever to standing up in and lecturing in front of the class. But I do treasure also the sitting around a table and having everyone have their say and having a true discussion, which I've been doing this year, especially last year.

0:03:45.7 SC: Again, I had one of those things where there was a supposed to be a seminar and I let too many people in, and it turned into a lecture. And I think that honestly, I'm still not very good at it. At leading the seminar discussions. I'm getting better. I try very hard. There's a fine line, right? You have to give them some content. You have to guide. You are the boss, right? You are the one who's supposed to know what's going on. And the first year students, these are all undergraduates. The first year students especially, it's all new to them, right? I mean, high school is lectures, [laughter], or if you even want to call it that, but high school is not sitting around a table with 12 people and deeply discussing an individual text. So they're not used to that, which is fine at, at some point in our lives.

0:04:30.4 SC: None of us are used to these things. Whereas the upper level, the seniors and juniors are they've done this before. They're old hats. They're good at talking, etcetera, etcetera. But I think that, that perfect line between giving new content and letting them talk is a very tricky one. And then not to mention what do you talk about? Do you let them, do you let the students wander off into what they think is interesting, which I kind of like to do, right? If they think it's interesting, that's a win for you as the professor, as far as I'm concerned. But again, there's certain notes you have to hit. There's certain beats you have to reach in the course of your course, as it were. It's not unlike doing a podcast interview where there's certain things that you wanna talk about.

0:05:13.8 SC: That's why you invited someone to be on the podcast, but sometimes they wanna wander off. And maybe you hit something really interesting by letting them do that. I have no point to talking about this, but it's the AMA I get to ramble on a little bit at the beginning, 'cause you're gonna sit and listen to me for hours. Now, after this occasional reminder, of course, this is the AMA, if you wanna know where these questions come from, they come from people who support Mindscape on Patreon. Those people could be you. If you wanted to support Mindscape, just go to patreon.com/ Sean M. Carroll, M as in Michael, my middle name, and, sign up to send a dollar to Mindscape per episode. It's not that much money, much less than you probably spend on coffee or tea or soda every week. And you get Mindscape, of course, you already get Mindscape for free.

0:06:05.1 SC: It will always, there will always be free Mindscape. I don't, I hope that I never, I can't imagine ever putting, like, making the podcast and putting it behind a paywall. That would be weird. But you do get it ad free. And also more importantly, you get to be part of that Patreon community that asks the AMA questions. You get little reflection videos that I do after each episode, and you get a sense of accomplishment for helping you support the podcast, helping me stay motivated to do the podcast for, among other things, and the other thing I wanted to remind you of course, is the Mindscape Big Picture scholarship. I keep forgetting to mention this, but not only is this something that you can donate to, but you can also apply to if you're a high school student and you want to go to college and study big picture kinds of ideas.

0:06:53.0 SC: So this is sponsored by, bold.org. B-O-L-D.org. So go to bold.org and search for Mindscape or go to bold.org/scholarships/mindscape. And the idea here is that we are getting donations from you. Nice Mindscape listeners, you've been extraordinarily generous, we did this last year and we gave two scholarships away. We're doing it again, and we're on track to give at least one. Hopefully, though I would like to give another two this year, and we're not there yet. So, we need a few more people to, kick in and then we'll be able to do it. And, I think it's a great service, right? I mean, it's enough money, it's $10,000 to the winners. That's enough money to really help them get through college. Maybe influence some people who wouldn't have been able to do it otherwise, the application deadline, if you are a young person thinking of applying is December 15.

0:07:47.2 SC: So go to bold.org. Don't just apply to Mindscape apply to all these other scholarships. There's plenty of opportunities there. There's one thing it's free to apply. So plenty of people apply who are clearly not actually fitting the criteria. I get it. Why not? Why not apply? But I'm especially interested in people who want to, for their future career study, the deepest mysteries of the universe, whether it's in physics or philosophy or anything else, biology or mathematics or whatever. It's the bigness, it's the abstractness, it's the fundamentality that we're looking for here. So hopefully we can find some appropriate students to give money to this year. Looking forward to that. And I think that's it. In other words, let's go.

[music]

0:08:48.1 SC: Roland Weber says, will the biggest ideas in the universe part two be comprehensible by itself, or do you expect readers to have worked, worked through part one already? An excellent question, Roland, one that I struggled with, and the rough answer, like if I have to give a one word answer, is no [laughter] I do not expect it to be comprehensible by itself. Of course, the reason why there needs to be longer answer is because people are different. Different people have different levels of preparation. It's perfectly comprehensible to someone who knows the basics of calculus and classical mechanics. Now, one way of getting those basics is through the biggest ideas, volume one space, time in motion. If you already have them from some other purpose, from other, some other part of your life where you learn a little bit of calculus, a little bit of Newtonian mechanics, maybe you know what a vector is, that would be helpful.

0:09:40.7 SC: Then you can read the biggest ideas part two, no problem. But I thought about can I make it self-contained? But in part one, there's whole long discussions of what is a derivative and what is an integral, for example, there's shorter but important discussions on what is a Hamiltonian, what is a vector? And I can't just re-put those in volume two, I'm afraid. So it's pretty basic stuff. I didn't wanna make the prerequisites as it were too much so that the right kind of people can still very, very happily pick up volume two and be fine. But certainly, if the ambition is to take the smart but completely doesn't know anything about physics or math person and bring them from that level to pretty, broad knowledge of the main ideas of modern physics, which is my goal, then you should read volume one as well as volume two.

0:10:36.7 SC: Keep in mind, my original idea was a gigantic book, was single volume, right? That was my idea. The publishing industry did not like my idea. [laughter] the apparently big books don't sell that much. And these days, for whatever reason, I don't know, I blame TikTok. But they wanted to split it into multiple books. So we, they wanted more than three. We compromised on three. But it is of a piece, it is a continuous kind of narrative. It does build on itself. That's one of the things about physics. It is cumulative. So, feel free to buy them volume two, and maybe then they will be inspired to go out and buy volume one or vice versa. The other, the final thing to say about that is, of course, all of these books we're inspired or are based on videos that I did, that you can still find on YouTube.

0:11:28.0 SC: So feel free to just give the, biggest ideas, volume two, which is called Quanta and Fields, and let them catch up by watching the videos right? That's always fine. And by the way, so, if it's, I guess there probably are some people listening to this who don't know what we're talking about here. I wrote this book, the Biggest Ideas In the Universe, volume one, Space Time in Motion, it's been out, did very well, I was very gratified at the response, and it was about classical mechanics and relativity Volume two will be about quantum mechanics and quantum field theory. And I think that people, despite the fact that I say it all the time, people don't quite believe me, it's mostly quantum field theory. It's not mostly quantum mechanics. So it's not a book about many worlds or foundations of quantum mechanics or anything like that.

0:12:18.0 SC: It's a book about effective field theory and gauge theories and Feynman diagrams and renormalization and symmetry breaking and confinement, and the spin statistics theorem, and all of the stuff that goes into our modern understanding of quantum field theory and particle physics. As such, it does get pretty far removed from one's everyday experience sometimes effective. I think effective field theories are kind of the centerpiece of the book. The linchpin, the thing that I think that is the biggest difference between how much people do know about it and how much people should know about it. More people should know, about effective field theories. And this book is gonna help teach you. And it's also the book where of the three, there'll be a third volume where we talk about complexity and emergence. Volume one was different than most popular physics books because of all the equations and so forth, but it does have close relatives.

0:13:17.6 SC: The two classic examples, two most obvious examples are Roger Penrose's book, The Road to Reality, and Lenny Susskind series of books, the Theoretical Minimum, both of which former Mindscape guests, but that's okay. Their books came first, so I'm not competing with their royalties. But, volume two, even though it overlaps with some of the stuff that is in their books mostly doesn't, the stuff that is in volume two, the stuff is in quantum and Fields is mostly stuff you're just not going to get anywhere else. I mean, I hope it's good. That's not to say that I do it well, I hope that I ended up doing it well enough, but it is something that is pretty unique in terms of, a book aimed at a wide audience. So we'll see how it goes. Dave Grundgeiger says, what would you do differently if you became 100% independently wealthy and had the unconstrained ability to fund your own research, including hiring researchers and buying equipment?

0:14:12.5 SC: Would you create your own research institute to retire or something else? Well, it's a good question. It's not gonna happen. So this [laughter] this is a possible world in which I don't give a lot of credence. That's okay. Unless you're offering Dave I don't know if you have a couple billion dollars hanging around, you don't need, I'll try to put it to good use. I would not retire. I would not do that. I like what I'm doing. I wanna have more time for what I'm doing, not less time, maybe I would feel a little bit less stressed about meeting a certain book deadline or something like that, but otherwise, what I'm doing is what I wanna do. I don't even know if I would move, like we just moved into this house. It's more house than we need, frankly.

0:14:53.9 SC: I would fix the roof tomorrow and a couple other mild things. But given that moving is kind of a pain and we like our house, I would probably just stay in the same house, probably stay with the same job I could imagine. Yeah, I don't even wanna quit my job. Like I was gonna say, I could imagine quitting my job and still doing research and talking to people, but having the job, gives you certain rights and responsibilities, right? Like, I can do things at Johns Hopkins, and I like doing that, so that's good. So I'd probably even keep my job, despite the fact that it gives me certain time constraints and obligations and things like that. What I definitely would do, I mean, I would definitely have a good time. I would buy some very expensive wristwatches and nice clothes.

0:15:40.0 SC: I would suddenly show up to work in nothing but bespoke tailoring every day. And I would, absolutely give money to a research institute. In fact, I would give money to the Natural Philosophy Forum at Johns Hopkins, which is the little thing that Janan Ismael and I have created and are trying to use to promote the idea of natural philosophy, the intersection of science and philosophy. Maybe I would make another research institute also for sort of complex systems on the East Coast to mirror what Santa Fe does in the Southwest. And I would provide resources for postdocs and, and graduate students and things like that, as well as new faculty members. Meanwhile, depending on how super rich I was, I would try to do things like, cure poverty and homelessness and cancer and things like that. There's always too many things that you need to try to do and not enough money to do them in or time.

0:16:35.5 SC: But I wouldn't spend a lot of my intellectual effort in curing homelessness, because I don't think that I am the person who knows best how to do that. I would put some effort into figuring out what way, what are the best strategies for doing it, and then I would hire people who could do it, and then I would leave them alone with my money. So I have enough money to buy my nice clothes and my nice wrist watches. Otherwise, I would try to do good things with it. Tim Jinitos says, Barry Lower said near the end of your podcast with him, that the many world's interpretation of quantum mechanics leads us to "Take probabilities out of the world and treat them entirely as indexicals" What does this mean? Why does he consider it a problem? Why do you not consider it a problem?

0:17:19.0 SC: Yes, indexical probability is exactly the, that's just the technical term for when you know everything except you don't know where you are. Indexical means there is not only the situation of the world, but there's also a little index or a pointer that says, and here you are in it. That wouldn't be necessary if there was only one version of you in the world. But once you have many worlds, or when you just have a big old classical universe, there can be multiple copies of you in the world, likewise, in Boltzmann brain like scenarios or simulation arguments or whatever. And so the idea is, in many worlds that the reason why we think quantum mechanics has probabilities in it, and those probabilities obey the Born rule, namely that the is given by the wave function squared is because everything happens that is allowed by the Schrodinger equation.

0:18:13.8 SC: Everything with a non-zero amplitude, will come to pass, but they don't come to pass equally. And when they do come to pass, you are forced to answer, ask the question, and then answer it. What is the probability that I am in one branch of the way you function of the universe versus another? So that's an example of an indexical probability. I will say, by the way, that, I'm actually kind of happy that Barry said that because I do think that this is, something that Chip Sebens and I, when we wrote our paper, we, so Chip Sebens and I, Chips is now a faculty member at Caltech, actually in the philosophy department. But he and I wrote a paper on the Born rule and Probability in Everett and Quantum Mechanics, AKA many worlds, where we really used this idea that the probabilities in many worlds come from indexical probabilities, or as we call them, self locating uncertainties.

0:19:13.0 SC: And we said that taking that at face value, you can derive the Born rule. Basically, you want to not only put probabilities on the different places you could be, but you'd wanna do so in a rational way. You wanna do so in a way that obeys some very natural criteria you would put on. And we showed that that led to the Born rule. Now, we were certainly not the first to derive the Born rule in many worlds, and we were certainly not the first to point out this existence of self locating uncertainty. But I do think that we tied it together, that we really told a story that was more or less complete and comprehensive. You can disagree with the story, but it kind of emphasized the fact that what those probabilities are is indexical probabilities, and that that's not something to worry about, that it's good news, not bad news.

0:20:02.5 SC: Because given that fact, you can derive the Born rule in a way. So I think that it's, I think that our approach, even though it's not the only approach, does fit most comfortably with the question of where do the probabilities come from in the first place. Basically my current attitude toward it is, if you think that [laughter] there are probabilities in many worlds at all, and you think that you should try your best in a good faith way to figure out what those probabilities would be, of course they're gonna end up being the Born rule. There's not like a second best possibility. There's nearly no other place you could go. The real question is why there are probabilities at all, and why there's what the nature of those probabilities are. And I think that self locating uncertainties or indexical probabilities is the right answer to that.

0:20:56.0 SC: So I think that that was a contribution that we contributed to, as it were. Barry considers it a problem, the same reason why David Albert considers it a problem, if you go back to my conversation with him also, which is that he doesn't think that that qualifies as a probability, or at least it's not the traditional kind of probability, the traditional kind of probability is you flip a coin many, many times and you'll notice that the coin comes up, heads half the time and tails half the time. You look at the frequency of the coin being flipped, and you say, aha, there's something I don't know about what's gonna happen next, which is the outcome of the coin flip, but on the basis of my experience, I can say that there's a certain probability based on those frequencies. So it's a frequentist version of probability, and it seems like a little more physical.

0:21:50.5 SC: So I don't think it's the right way to think, but let's steel men at first. Let's give the reasons why people would ever think this way. The nice thing about that version of probability that you just count frequencies of things is that it seems rigorous, it seems objective, it seems out there in the world. It doesn't seem like something I invented. It doesn't seem like something that I could have disagreed with. It's a fact that in my past history, most coin flips of fair coins are half, well, all coin flips of fair coins add up to about half heads and about half tails right? Now, you extrapolate that to the future. Maybe there's some heavy lifting to be done by some predispositions or some philosophical approach, etcetera, etcetera. But at least it's harder to see that it was just something you made up that it was just totally subjective.

0:22:41.8 SC: Whereas the indexical probabilities, you admit from the start that it is subjective, that it is your choice, that you choose to put a certain credence on certain outcomes, you then argue that it's not arbitrary, and this is why. So anyway, I didn't finish the anti argument. The anti argument would be that the self locating uncertainties there's just no way to do it. There's just not something that is well-defined, maybe you could put one probability measure on someone else, puts another probability measure on. There's no way to say who's right and who's wrong. That's the argument against it. The counter argument to that is, well, look like I said, you, it is true that it's subjective, but it's not arbitrary because you can ask that your probabilities obey certain conditions. That's what Chip and I did in our paper.

0:23:35.6 SC: That's what other people do. The conditions you, they would like to obey seem pretty natural. The the one that we focused on, which is not trivial, but nevertheless, pretty reasonable seeming, is that whatever probabilities you're putting on being in the spin up branch of the wave function versus the spin down branch, it shouldn't matter what's happening elsewhere in the universe. Once you've figured out what is happening in your nearby environment, the fact that somebody's wearing green socks versus purple socks a thousand miles away should not affect your assignment of probabilities. Turns out that's all you need from that, you derive the Born rule. So I don't think it's a big problem. I think that the real difference is not getting the Born rule. The real difference is do I need to put a probability distribution on these things? And I think that that's just what scientists do.

0:24:25.9 SC: Scientists don't know what's gonna happen next in the universe. You don't know whether gravity will be attractive tomorrow, right? It's been attractive a 100% of the time so far, but there's a possible world where the force of gravity becomes repulsive tomorrow. What is the probability that you're in that world? Right? One way of thinking about that question is to say, well there's different possible worlds. I don't know which one I'm in. It's an indexical probability. How do I know what world I'm in? Now, there you don't have the nice rational decision making procedure you have with the Born rule, so you're even more trouble, right? People will disagree, I think the probability is 10 to the minus three. I think it's 10 to the minus seven, and there's no right or wrong there. So that's even worse. But you can't get around it.

0:25:17.8 SC: You still have to do it. In the case of many worlds in the Born rule, there is a decision procedure that gives you a unique answer, and you also still have to do it. So maybe people don't like it. Maybe people would like it would prefer it if it were more objective. But you don't get to choose. That's not a [laughter], that's not an option that you have. You have to deal with the world as it presents itself to you. Seth says, I've been watching the TV show Devs, which includes the idea that using quantum computers, they're able to project what happens in the past and in the future, given total knowledge of the current state of things in the universe now, and also relies on that things are deterministic. My understanding is that at the end of the, that at the quantum level, things are not deterministic, but probabilistic, could you expand a little on quantum theory and its effect of how it affects determinism being true or not, and if it would be possible to project backwards or forwards given enough information on the current state of affairs?

0:26:14.5 SC: Sure, short answer is we don't know. Okay? And this is because we don't know what the right final formulation of quantum mechanics is. There are different competing schools of thought about what quantum mechanics actually says. Some of them are perfectly deterministic, some of them are not deterministic, are are truly indeterministic, truly stochastic in some way. Now, one thing that we do think is true is that observers cannot by themselves actually predict what they're going to see happen next. Because even in the versions of quantum mechanics that are deterministic, they always include there in the formulation of the present state of the universe, some information that is inaccessible to observers. In the case of a hidden variable theory, that inaccessible information is the values of the hidden variables. In the case of many worlds, it is where you are in the way function, but one way or the other, you cannot predict literally, practically, even if you had infinite information, what you are going to see next.

0:27:23.5 SC: Now, nevertheless, in both of these theories and hidden variables, or in many worlds, there's a god's eye view or a Laplace's demon eye view, which is deterministic, and everything is perfectly predictable from the past to the future, etcetera. There's also other theories. There are theories where the wave function truly collapses stochastically, either because of a measurement or because of some random outcome that happens every so often, or some threshold is triggered or whatever. So we don't know. But again, the one thing that we can agree on, and no matter what our favorite version of quantum mechanics is, that real world observers can't do it. So for all intents and purposes, as far as actual human beings, agents in the universe are concerned, the deep down laws of physics are indeterministic, not deterministic. Now, I'm gonna group together three questions about a deep issue in mathematical philosophy or the philosophy of mathematics.

0:28:24.4 SC: Sandra Stuckey says, how do you reconcile being a poetic naturalist with being a reality realist as opposed to a mathematical realist? If tables and chairs are real because they provide a useful way of talking about the world, then why would vector spaces and Riemannian manifolds not be real? After all, they have proven to be very useful ways to describe the world. And if those are real, why not sets and numbers out of which we construct vector spaces, etcetera. Leo Behe says, I was a big fan of our mathematical universe, the book by Max Tegmark, another previous Mindscape guest. I like that. By now, the people asking the questions are including parenthetical notes saying previous Mindscape guests, 'cause they know that I do it all the time. In the book, Leo continues, he talks about how as we look closer, the basic constituents of the universe like mass and charge, seem to only be describable by their relations to other similar mathematical entities.

0:29:22.6 SC: Are you sympathetic to the view that the universe is fundamentally mathematical, or do you think there's likely to be some basic stuff with qualities that can't be mathematically quantified? Finally, P. Walder says, this is three questions I'm grouping together within the poetic naturalist view of reality. Where do abstract entities such as information, knowledge, and numbers fit, are such entities a category of thing that are to be considered just as real as matter and energy, and therefore, of equal importance in accounting for the reality we experience? So the question here is, so some of you know that I have written this paper called Reality Realism. It was prompted by conversations with Justin Clark Don, yet another former Mindscape guest. Remember when I had Justin on the podcast, we were talking about morality and mathematics. In both cases, an argument can be made that the arguments in favor of realism, in the case of morality, moral realism says moral judgments reflect to something objective and true and real about the external world.

0:30:25.9 SC: Whereas moral anti-realism says no, we kind of make them up. And mathematical realism, do the vector spaces and sets in numbers that we use to talk about math, have an independent existence like Plato would have us believe, or are they useful fictions that we use to describe things? And long story short, my paper put forward the idea, which is not only my idea by any stretch, it's one of those papers like I did, it wasn't my idea [laughter] to write the paper. So sometimes you write a paper because you have a good idea and you wanna share it with the world. Other times you write a paper because the world comes to you and says, we would like your opinion about this. So anytime you hear me talking about free will or consciousness or mathematical realism, I've probably been provoked to do it by somebody else.

0:31:16.8 SC: I don't do those things, myself intentionally, because I don't think that I'm the best person to talk about them. I think I have opinions, but there's probably people who have much more considered opinions here than I do. Anyway, my, attitude was what I think is real is the real world, the physical world. I think that the stuff of the universe, whatever that stuff is, I don't think you can say, well, what is the stuff of the world? That's not an answerable question. Like what kind of answer could you possibly be asking for? It's unique. It is sui generis, as the philosophers would say, when they don't want to say it's unique [laughter] It's the one and only world that is out there. Reality. That's what I think is real. And all this stuff, including mathematics, I suggest is not separately real.

0:32:09.6 SC: It's a way of talking about the stuff in the world that we have. And to be very super duper honest here, I'm not completely committed to this. This is the best I can do. Having talked to people, having read things, having thought about it, etcetera, this is my current position, but it's not something where I understand all the ins and outs perfectly well. So I'm open to changing my mind about this if someone can convince me. Otherwise, I've talked to various people on different sides, right? Tim Maudlin is a mathematical realist. Jodi Azuni is an anti-real list, a nominal list, etcetera, etcetera. So to, I guess let's do Leo's question first. Am I sympathetic to the view that the universe is fundamentally mathematical, or is there likely to be some basic stuff? Given what I just said, I hope it's clear.

0:33:00.1 SC: I think that there's some basic stuff. Mathematics is a way of describing what happens in the world. It's not the stuff of the world. When someone like Max Tegmark says, the world is math, I literally have no idea what that is supposed to mean. [laughter] the world is well described by certain mathematical statements, sure that I understand. But when every mathematical structure is real, things like this, I just, the words have become unmoored from their meanings in my head. So I don't know what that means. I think that physical stuff is real, and math is a way of talking about it. So to get to Sandro's question, how can I be a poetic naturalist, which is someone who thinks that there is only one world, the physical world, but many ways of talking about it, and those ways of talking about it can be equally real.

0:33:46.8 SC: So you have some fundamental level with, quarks and glue-ons and some emergent level with tables and chairs. And so Sandra was suggesting that math is a way of talking about the universe, much like tables and chairs. And therefore, just as we ascribe reality to the tables and chairs, we should ascribe reality to numbers and sets. The difference, I think, is I do think that there's a difference, which is that I don't ascribe reality to the word table. I don't ascribe reality to the idea of table-ness. I ascribe reality to tables to an individual table. There's a table right here in front of me. I ascribe reality to that [laughter] And likewise, when I have two apples in front of me, I can ascribe reality to the fact that there are two apples. That does not mean, I think that I have to ascribe reality to the notion of two as an abstract entity.

0:34:40.8 SC: It is a way of talking, but the ways of talking, I mean, sometimes probably, maybe this is my fault, that I and myself talking sloppily about it. If I say the ways of talking are real, I hope I don't ever say that. But if I do, by mistake, what I mean is the levels describe real features of the world. The ways of talking about them are vocabularies that we use to describe those real things. They are not to be confused for the things that are real. The motto of poetic naturalism is there is only one world, the natural world, the real physical world. That's what's real. So I think it's entirely compatible with being a reality realist. I am not sure, but I wonder if reality realism is what Wittgenstein was getting at when he said that the world is everything that is the case famous opening line, from Wittgenstein.

0:35:31.3 SC: I never understood what he meant by that. Like, what else could it be? But maybe he was getting at something like this. Finally, P. Walders question, I think I've already answered it right. What is the poetic naturalist view of reality, Vis-à-vis abstract numbers, etcetera. I do think that the reason why I am incompletely certain, I'm never meta physically certain, but I'm nowhere near even practically certain about this stuff is because I do think, and Justin made a good distinction in his book, that mathematics is objective. In other words, when we understand the meaning of two and plus and equals in the context of ordinary, addition, you will always agree that the answer is four. Now, there's, I know there's an Internet argument about is two plus two equals four. That's 'cause I just gave you some footnotes in that statement.

0:36:28.5 SC: If you don't agree with a conventional definition of addition, like if you're doing addition modular three, then two plus two is not four. So it does depend on your definitions. But if you agree on the definitions, then everyone should get two plus two equals four. That's objectively true. But as Justin argues, objectivity is different than realism. There's an out there-ness to realism. And he has a list of criteria of what he means by real. In this case, you have to define what you mean. And in my paper that you can find online called reality realism, I try to pinpoint where I diverge from him in the claim that mathematical, concepts have to be real. So you can look that up on the Internet, which is still working as of today's podcast. Okay, going on to the next question from Loose Leaf.

0:37:18.7 SC: Obviously, you're someone who thinks very deeply about philosophy and have your own unique understandings about meanings of life. So I'm curious, would you apply your own unique values to your romantic relationship and other close relationships in your life? Or would you rather go along with or at least empathize with the more popular values that most people tend to have? And if you were to raise a child, would you try to convince him or her of your own worldview, or would you try to let him or her explore the world freely without exposing him or her to your ideas? So I think this is cheating 'cause you have more than one question here, and we do have a rule. You shouldn't have more than one question. So I'm gonna sort of skip through it and kind of connect them. I mean, of course I'm going to follow my own values.

0:38:00.6 SC: I don't know why. Maybe there's some implicit subtext here that I'm missing. Why would I empathize with more popular values that I disagreed with in a romantic relationship or a collegial relationship or a friend relationship? I mean, that would be weird if I went to all the efforts to really think through my values and find some that for which I disagreed with the popular conception, but nevertheless stuck with the popular conception rather than mine in the particular context of friendships or romance. That would seem very weird to me. No, I don't see any reason to do that. Now that doesn't mean that you have to be a jerk [laughter] That doesn't mean that when you disagree with people, you have to be impolite to them or anything like that. There are ways to negotiate those real world situations when it comes to having children, etcetera, which I do not have.

0:38:49.2 SC: But, on the one hand, I would absolutely let them try to come up with their own worldview the best they could. On the other hand, I'm certainly going to expose them to my ideas. Again, why would I not do that? [laughter], I don't quite understand why this question is being asked, but I worry that there is some reason why it's being asked, which is why I'm trying to answer it. Yeah, I think that the whole point of going through this exercise of trying to think very hard about what your values are, etcetera, etcetera, means that you should follow your values. I guess the question is sliding back and forth a little bit between following your values and telling other people about your values. Usually I would do that. But it depends on, there might be social circumstances under which, I don't want to go into details about my atheism, for example, right?

0:39:44.8 SC: Or I don't want to go into details about whatever other choice I made, the death penalty or something like that. So I think that in general, if you have values, you should stick to them. I don't ever see why you should stick to someone else's, but that doesn't necessarily mean you have to be in your face about it. I guess that's what I'm trying to say. Keith says, thanks to your AMA post Patreon perk, shout out. So what Keith is getting at is that when I post the AMA answers on Patreon, I will say a little bit like a couple of sentences, not that much, believe me, I don't put a lot of effort into them. All patreon supporters do get a monthly picture of Ariel and Caliban, of course, usually Ariel and Caliban. Sometimes if I have a new book out or something like that, I will advertise the book, but it's usually a kitty picture.

0:40:31.1 SC: So there's some little tiny miniature Patreon perks that the, supporters get. Keith says, I'm sitting here trying to imagine the space of all unasked questions. That's because I mentioned the other day that despite the fact that we've gone through a lot of questions over the history of the AMAs, we're still nowhere close to filling the space of all unasked questions for an AMA where all questions are finite length and written in the finite alphabet, at least there's a finite space of questions. My question is, what additional constraints on this space really narrows it down to the subspace of what would be considered questions? For example, maybe a question, something with a conceivable answer or response. Sadly, I have no great answer to this. What additional constraints really narrows it down to the subspace of questions. I will mention that of course, even if this is just Borges Library of Babel.

0:41:22.8 SC: But for questions, one question that you, that people raise in the context of Library of Babel is there's a finite number of books in the library. If you didn't listen to the episode with William Egginton, we were talking about the library Babel, where every possible book is in it. So you have a finite alphabet, and you just have every single possible concatenation of every single possible set of letters in that alphabet. So you have every possible book, and people say, well, if it's finite, the books have to be of some length, like 500 pages long or whatever. So you don't actually have every book, because some books are 600 pages long. The answer to that is that's just a two volume set [laughter] If you have a 600 page book that can be found by including the right first volume and the right second volume.

0:42:15.3 SC: So all the books really are there. Likewise, the same thing goes true for these questions. Even if the questions are a finite length written in a finite alphabet, there are not a finite number of them because there's an infinite number of ways you can string together these strings of finite questions. So anyway, that's just a little footnote there. There's a very good question that is being asked here, if you imagine the space, well, maybe it's a good question or not. The literal question that Keith asks is what additional constraints really narrows it down to the subspace of what would be considered questions? So what is the space that you're narrowing down, is it just the space of all sequences of letters or sequences of words or sequences of words with a question mark after it? I think that matters a lot.

0:43:06.9 SC: So let's imagine that what we're considering are grammatically correct sentences, then you know that some grammatically correct sentences take the form of a question and some don't. This is a purely, syntactic issue syntax, as opposed to semantics. It's not about what the words mean, it's about their grammatical relationship with each other. So the rules of grammar tell us that certain question, certain strings of words take the form of a question certain ones don't. That's little ambiguous there we would like the rules of grammar to be a little bit more cut and dry than they really are. Sometimes you can just add a question mark to the end and get a different sentence, right? If you say, this is red, that sounds like a declarative sentence. If you say, this is red? Then you're asking a question, meaning really, is this red just asked in a slightly poetic way?

0:44:03.5 SC: But I think that what you're getting at is the issue of what does it mean to be a question aside from issues of syntax and grammar and things like that. And then, yeah unless I'm missing something deep, I think that question is something that in principle sets up a possible space of allowed responses. Like when you say, what color is the book cover? So you know that the possible responses are colors. There's hopefully a finite number of colors that you could refer to when you gave your answer. And then it is setting up that space of possible responses, and there is a right response. Or at least if the question is well posed and about something true in the world, then there could be a right response to it. So questions are about setting up some uncertainty about what a response is in a class of conceivable responses.

0:45:02.4 SC: And then looking for the right one, I don't know, I just made that up. So maybe someone out there in Logic Land has a better answer than I do. John Shawn Ning says, I've seen other sources claim that the magnetic field is really just the derived effect from viewing the electric field from a different relativistic frame. If this is true, can it really be said that there is a magnetic field fundamentally, or are there just electric fields plus relativity? In part, this is one of those, it depends on how you wanna define your words questions, right? Let me tell you how I would put it. There is one thing, which is the electromagnetic field. One of the things that happened when we invented relativity is that we unified different conceptions with each other, right? Famously space and time were separate before relativity came along.

0:45:53.9 SC: After relativity comes along they're both part of one underlying unified space time. Less well known is the fact that the same exact thing happens to the electric and magnetic fields. It's not that the electric field is the real thing any more than space is the real thing. You haven't reduced the number, you haven't reduced one thing to a pre-existing thing. You have unified two things together. So it is true that if you have an electric field and zero magnetic field as seen by one observer, then as seen by another observer, you will have a different electric field and some non-zero magnetic field. But it's also true that if you have a magnetic field and no electric field as seen by some observer, then to a different observer, they will see both a non-zero electric field and a magnetic field. And the underlying thing is a tensor field.

0:46:49.5 SC: As readers of The Biggest Ideas volume one are well aware, there is an electromagnetic field strength tensor that, includes both the electric field and the magnetic field. And that is the thing that exists in some sense. And how you divide it up into electric and magnetic fields will be observer dependent in exactly the same way that how you divide space time up into space and time will also be observer dependent. Rue Phillips says, can you talk about the future of many worlds? What would have to be true, or what milestones do you think we'll need to achieve for the majority of the physics community to adopt many worlds as a proper model of foundational physics? I think there's two things going on. One is already happening, which is that you just need to appreciate the need for a proper model of foundational physics, proper model of quantum physics.

0:47:40.8 SC: In particular, we've been muddling along with the Copenhagen interpretation, which is hilariously ill-defined for a very long time now. But as technology improves and as physicists are paying more attention to truly quantum phenomena, especially entanglement and measurement and so forth, they are becoming easier to convince that we need to get quantum mechanics, right? So whether it's many worlds or something else, first we need to convince physicists that it's worth spending the effort to think carefully about the foundations of quantum mechanics. The other is why many worlds versus anything else? And I do think that physicists are relentlessly practical. At the end of the day, the thing that will make many worlds very popular is to convince people to demonstrate that it helps, that thinking about quantum physics in this language actually helps you solve other problems. I think that's true. And so I think that that will become more and more clear.

0:48:40.4 SC: It's not quick, it's not like next year it's gonna happen or anything like that. But I think that even though many worlds does help you solve problems, there's no killer app as of yet. There's no case you can point to which you can say like, look how enormously clearer this is in many worlds than in other areas. There are some pretty, pretty clear examples to me, like the delayed choice quantum eraser that I once wrote about on my blog, how enormously easier it is to understand in many worlds than in conventional Copenhagen kind of language, what I'm really hoping for is understanding something in quantum field theory or quantum gravity, that just makes so much more sense. If many worlds is the case. Now, I don't know what that thing is, so maybe it won't happen. But if that does happen, then the rest of the community I think will eventually come along.

0:49:34.6 SC: Nick C says, when one thinks back in time toward the big bang, there's probably a lot of agreement back until at least Electroweak Unification, but as you go further back, there's presumably a lot less agreement and experimental observational proof and things get more speculative. How do you think one should address this when communicating with the public who don't necessarily know what's speculative and what's not? It seems like sometimes scientists speaking to the public will talk about their own highly speculative idea in the same manner as ideas that are much more established. Yeah, that's true. And I guess, I think scientists should be honest, [laughter] I think that it's, I get why they're not, and they're not dishonest in the sense that they're saying something they know to be wrong. It is absolutely true that scientists are a little vague sometimes about ideas that are clearly on the right track versus ideas that are just very, very promising versus ideas that many people don't agree with, but they think are right.

0:50:34.9 SC: I think that, you have to be clear about those different possibilities. It's a service to the people you're talking to, but it also slows you down, and it can be confusing, right? Like you'll hear me over and over again. I've already done it several times in this AMA, when people ask questions about quantum mechanics, I have to stop and say, well, it depends on your view of the fundamental foundations of quantum mechanics, right? And that's enormously, I don't wanna say irritating, maybe a little bit irritating, but it certainly slows you down, when I wrote volume two of The Biggest Ideas in the Universe, the whole philosophy of the biggest ideas books is I'm going to teach you ideas that are more or less accepted and will continue to be accepted for the next 500 years. Okay? In the same way that Newtonian mechanics is still accepted today, not that it's fundamental, not that it's perfect, we know that there are cases where it doesn't work, but there's also cases where it still does work. You can get to the moon perfectly well with Newtonian mechanics, and that will always be the case. There will always be the case that Newtonian Mechanics works in its domain of applicability. So for the biggest ideas, those are the ideas I wanted to focus on, and that's why I did not want to spend a lot of time talking about different foundations of quantum mechanics theories. But what that meant was I had to talk about things.

0:52:01.0 SC: I was threatening to get bogged down when I'm talking about quantum fields or wave functions or virtual particles or whatever. And every single time I'd have to like add a paragraph saying, well, you could say this, or you could say that, or you could say something else. So ultimately I just for the most part, chose a perspective where I said, look, here is my way of talking about quantum mechanics. I will let you know right here and now that other people disagree, and here's the ways in which they disagree, but I'm gonna assume it for the rest of the book. Okay? But at least I was that obvious, that honest. Whereas I do agree when people, when it comes to the early universe, we have a pretty good theory, but parts of it are speculative inflation being the most obvious idea of something that is still speculative.

0:52:48.9 SC: We don't have cut and dried evidence that it's on the right track. It's not at that level of Newtonian mechanics. We don't know whether the universe inflated at the end of the day. And I do think that as a shortcut, some people talk as if we do. So I don't think I do, I try not to anyway. I try to be honest about what we know and what we don't. And by the way, this isn't just scientists talking to the popular press either or popular audiences. They talk that way to each other, right? They say like, well, during inflation this must have happened. But the difference is that they all know what level of uncertainty they should put on it. And maybe someone thinks that inflation is 90% likely to happen and someone else thinks it's 40%, but they can talk to each other and they can say like... They can translate into, oh, if inflation happened, then this must have happened.

0:53:42.8 SC: Whereas to a popular audience, they might not be able to do that translation 'cause like you are implying they don't know which parts are perfectly well established and which parts are not. Even when you go back to the time of electroweak unification, that is a much earlier time and a much higher temperature than we actually have data about in the early universe. Okay? So there's the reason why people act for some justification, pretty confident about what happened at those early times is because we have particle physics experiments. We do have some data about what the laws of physics were like back then, okay, that comes from the Large Hadron Collider and other particle physics experiments, etcetera. But we don't have direct cosmological data about that. So it's absolutely possible that the actual time of the electroweak phase transition was one where the cosmological situation was pretty different than we think it probably was.

0:54:50.0 SC: The actual time when we have the earliest empirical evidence in the universe is Big Bang Nucleosynthesis. That's between a second and a few minutes after the Big Bang. I wrote a paper once with Manoj Kaplinghat where we said, what if it were different? Are we really sure? Like a lot of people take the standard model of not just of particle physics, but also cosmology, etcetera. And they say, well, if you had more neutrinos or more dark matter, or whatever, how constrained is that by Nucleosynthesis? And what Manoj and I did is just to say, let's say we don't take the standard model, let's just say we have an expanding universe. We take the standard model of particle physics, but not cosmology, so not general relativity etcetera. What do we know about how the universe was expanding at that time? So not just the rate of expansion, but the rate of change of expansion.

0:55:38.5 SC: And actually the answers were pretty constrained. At that time when you were making, heavy nuclei, sorry, light nuclei like helium, lithium, deuterium, there's enough experimental data that you can't do anything dramatically different from the standard picture in terms of both the expansion rate and how it is evolving over time. But before that, that's a temperature of what MEV millions of electron volts, the electroweak transition is hundreds of billions of electron volts. So that's a lot higher temperature. We don't know what the universe was doing back then, but we can talk about it with some confidence because we have particle physics data. But to be honest, you don't really know for sure. Brendan says, I realize you have a few trade books in a textbook in the works. Have you ever considered writing science books for children?

0:56:33.5 SC: I think you have an amazing ability to convey complex concepts in a way that is easy to understand and that seems like it would help to make really good science children books. I've not really taken that idea seriously. No. I mean, I take lots of ideas casually non-seriously. The number of books I've contemplated writing is truly enormous in the size. I'm not at all sure that I would be good at it. At least I'm not sure that I would be good enough at it to be better than anyone else, right? I think that my value added is much more on the other side. Like I think that I can actually, even in my popular books within the span of popular books, my popular books are more towards the serious side, right? A little bit more detail, a little bit more.

0:57:22.8 SC: Even my book on the Higgs Boson, which is probably my most popular book there, once you get two thirds of the way through, there's gauge in variance and symmetry breaking and things like that. And I've had more than one person say like, I really like the book up until then and then it became hard. And maybe that's just me. Maybe if I'd just forced myself to do better at it, I could. But I'd like that. I'd like working at that edge where I'm gonna push you a little bit, I'm gonna assume that you're pretty smart, you're motivated, and I want you to stretch your brain a little bit bit. That's my favorite place to be. So that's usually where I am. But who knows? I've changed my mind before.

0:58:04.0 SC: Jeffrey Siegel says, in the past I've read speculations about manipulating black holes by making them charged and then moving them using electric fields. However, I gather that electric interactions can be expressed in terms of exchanges of photons. I was curious then how to imagine the photon exchanges with black holes. If nothing ever leaves the black hole except for hawking radiation. Do all the interactions of a black hole with an electric field involve photons getting trapped by the black hole independent of whether the black hole is being repelled or attracted by an external electric field? I think there's various ways to answer this question, but the short answer is do not think of electric interactions as being expressed in terms of exchanges of photons. There are certain circumstances under which that's something you can do. Like when an electron and a positron scatter off of each other in a scattering experiment, then you have Feynman Diagrams and you calculate with exchanges of photons and it all makes perfect sense.

0:59:02.4 SC: But when you have a single particle that is just sitting there with a static field around it, a so-called Coulomb field because it's Coulomb's law that says the electrical field force goes as one over the inverse distance squared, then a static field surrounding a single particle is not easily described, not naturally described in terms of photons. You absolutely can do it, no problem doing it if you just wanna run through the math. It doesn't give you the direct intuition that you get just by thinking of it in terms of a classical field sitting around the particle. And the same thing is exactly true for black holes, right? I mean, black holes have a gravitational field. And guess what? You can think of that gravitational field as arising from the exchange of gravitons. It's not the natural way to think about it in that context.

0:59:54.1 SC: It's not easy. It doesn't help you understand anything. But if you really were dedicated to doing that, you could likewise with the electric field around a black hole, it's just much easier to think of the electric field around a black hole. Don't torture yourself by turning it into photons. Black holes can have electric fields. You have electric lines of force. They enter the black hole. There you go. The black hole has an electric field. In fact, think of it this way. There is Gauss's law which says that if I have some region of space and I take the electric field that goes through the boundary of the region, so I have some three-dimensional region of space, some boundary of that region, which is kind of like spherical in topology, but it might be lumpy, who caress it doesn't matter. I have a boundary that is a two-dimensional boundary.

1:00:44.1 SC: And there's an electric field. There may or may not be an electric field, but there could be an electric field sort of going through the boundary at different points. And Gauss's law says the total electric field escaping through that boundary is proportional to the amount of electric charge inside, right? So this is most obviously interesting and important. If you just have a single particle, you can integrate the electric field and a little sphere that is nearby the particle and you're getting an answer. How much electric field is coming out through the sphere, very close to the particle, then you consider a different sphere far away from the particle, but still no other particles have entered or left. So you still only have that one particle and you can integrate the electric field around that bigger sphere. At any one point on the bigger sphere, there's much less electric field 'cause you're further away from the particle, but there's a lot more area to that sphere.

1:01:40.6 SC: It's bigger and those two effects exactly cancel each other out. That's what Gauss's law tells you. So just by counting the electric field in a region, you know how much charge is inside. Now imagine that I have a bunch of electric charge inside and it collapses to a black hole. Gauss's law doesn't care that it's a black hole. It says far away. You can count up all the electric field add it up and you find out how much charge is inside and that you get a very similar issue in gravity. You get exactly the same kind of thing. You can figure out how much mass is inside the region. So these long range forces of gravity and electromagnetism exist outside of, whatever particle or object you're talking about. And that's just as true for black holes as for anything else. Therefore, yes, you could absolutely imagine charging up a black hole and using that to move it around.

1:02:38.0 SC: In fact, you don't even need to charge it up. You could move it around just using gravity if you were really good at it, but electromagnetism is much easier to manipulate. So that might be a smart idea. I'm gonna group together these two questions. Dylan Hall says, is there any explanation at all for three generations of matter and specifically why it's only Fermions and not Bosons, even though some Bosons have mass? Is it inherent in the Pauli's exclusion principle somehow. It feels like there's an important connection that I'm missing. And Kevin O'Toole says, is there evidence against quarks heavier than top quarks or are there theoretical arguments against them? Naively, it seems more natural to have an infinite series of ever heavier particles than to have exactly three levels of quarks and leptons. So both questions are about more quarks and leptons beyond what we've actually already seen.

1:03:30.5 SC: So we don't know a once and for all fundamental reason why there are three generations of matter. So what's being referred to here is forgetting about the Bosons, forgetting about the quark, sorry, the gluons the photons, the W and Z, the graviton, the Higgs Boson, forgetting about those, those are all force carrying boson particles. The matter particles of the standard model of particle physics fall into this nice structure of generations. That is to say there is a quark with a charge plus two thirds the up quark. There's a charge... A quark with charge minus one third the down quark, there is a lepton that is to say, not a quark, a particle that does not feel the strong force. There's a lepton on with a charge minus one the electron, and there is a neutral lepton the electron neutrino. So those are four particles up quark, down quark, electron, neutrino.

1:04:29.3 SC: And that pattern of those four particles repeats two other times. There's a heavier generation with charm and strange quark muon and muon neutrino and a yet heavier generation with bottom and top quark, tau lepton and, tau neutrino. And then that's it. There's no even heavier ones, okay? That we know about. I think that there's not absolutely a 100% evidence that there isn't a heavier generation. People have certainly looked. Look, it's... Let's just put it this way. Whenever you ask a question like this, it's good to ask these questions. By all means, you should wonder. But probably particle physicists have thought of this already. So probably people have thought about it and there is a reason why either we haven't seen it or we're not trying very hard. So people have certainly thought about heavier generations of Fermions. They tend to be complete generations, and there are reasons for that they get a little subtle.

1:05:30.8 SC: The most obvious one is something called anomaly cancellation. And I'm not gonna get the details right on this for you. But just so you know that there's something out there, you know that all these particles, all these Fermions interact through gauge forces, right? Through forces carried by photons and gluons, and W and Z Bosons, and all these are related to symmetries. That's why they are gauge forces. I feel I can use these big words because I know that in May you're gonna be buying Quanta and Fields biggest idea is volume two. And you will learn about what all these words mean, like gauge theories and things like that. So it turns out that, as I've said many times before, the way that we generally go about constructing quantum mechanical theories is to start with a classical theory and quantize it.

1:06:22.0 SC: You don't have to do that. But that's typically what physicists do. So it also turns out that when you start with a gauge theory, a symmetry group that gives rise to some Bosons, there's a danger that when you quantize it, the symmetry will be broken. Symmetries can be broken in the real world, and it's okay, but only under certain carefully controlled circumstances. And again I won't go into what all the details are, the kinds of breaking that is purely quantum mechanical. There's a kind of symmetry breaking that purely comes from the process of quantizing, a classical symmetry that is called an anomaly, a quantum anomaly in the classical symmetry. And if it's a gauge, symmetry anomalies are bad. You don't want to break truly, fundamentally, really your gauge symmetry when you go from a classical symmetry to a quantum symmetry. And what that means is there's only certain combinations of particles that are allowed because you can calculate what the anomaly will be.

1:07:26.1 SC: And if the anomaly is not zero, then you're in trouble. A theory like the standard model of particle physics in each generation, the anomaly is exactly canceled. So there's like a number you calculate and it gets a contribution from the electron and the neutrino and the Up quark and the down quark. And then that number, you want to be zero. And it is, good thing to calculate as a little homework problem if you're a graduate student studying quantum field theory. And because it cancels for each generation, it will also cancel for three generations, right? It doesn't matter, you just... Three times zero is still zero. But if you try to add some subset of particles, like I'm gonna add a new positively charged quark, you're running in big danger of creating an anomaly where there wasn't any and therefore ruining your gauge symmetry. So there might be other clever ways to cancel it.

1:08:17.8 SC: It's not an argument that you have to work generation by generation, but it's certainly the safest way to work. And if that's true, then if you have a heavier quark, you will also have leptons that go along with it as part of the generation, including neutrinos. And those would probably be light, right? And we haven't seen those yet. So that's a good reason to think that there just aren't any heavier particles. Heavier fermions than the ones that we know about. It has nothing to do with the Pauli's Exclusion Principle, just so you're there. The other thing is, is it natural to have an infinite series of ever heavier particles? It's actually not that natural. And the reason why is because the word heavier is doing something here. The word heavier means that you're talking about particles that have mass and all of these particles that we're talking about, the top quark, etcetera, these all get mass from the Higgs mechanism, from the fact that the Higgs boson has a non-zero expectation value in empty space.

1:09:21.8 SC: And so all of these particle masses are given by some dimensionalist number, some coupling constant called the Yukawa coupling times the expectation value of the Higgs boson and the heaviest of them, the top quark is pretty close. It's like one times the expectation value of the Higgs boson, that's not exactly true, but it's an order one, the electron is much lighter, etcetera. You don't expect particles that get their mass from the Higgs boson to have masses much higher than the Higgs expectation value. And therefore it makes sense that the top quark is where it should be. Everything else is a little bit lighter, but there's no obvious reason why you would want to go even heavier than that. You could, you could imagine doing it, but it wouldn't be the most natural thing in the world. Okay, AJ says, I think you said something to the effect that free will is real, but not fundamental.

1:10:18.7 SC: That makes sense to me right now. But assuming we are at some future point able to explain the macro phenomenon of free will from whatever constitutes it, would it still be right to call it free will? That is, doesn't the Compatibilist position break down at some point? Sure, absolutely no worries about that. But guess what? I don't think it's gonna happen, philosophers, physicists, neuroscientists, they sometimes like to grant themselves thought experiment omniscience, right? Like, what if we were laplace's demon? What if we knew everything that was going on in the brain, etcetera. What if from some amount of data that we could store on a hard drive, we could predict everything a human being could do? Okay, you're not, it's not gonna happen. The human brain has 85 or 86 I forget the number billion neurons in it, and they are connected to each other in many, many, many, many connections.

1:11:18.2 SC: So the actual number of connections in the brain, the connectome as it is called, is way bigger than 85 billion. Okay? It's... I don't know what the number is. Trillions and trillions and trillions. So the idea that somehow you would have enough information to predict what that brain is gonna do next, it's not a very good idea. It's not very promising, I would say I could be wrong about that. There's no law of physics that says it couldn't happen. It's pretty close because it's not just the state of the brain, but the brain is not a closed system, right? The brain is interacting with the body, the body's interacting with the world. And so you need to be constantly updating your 85 billion neurons and all their connections to really know what the brain is doing. But yeah, in principle, you could imagine knowing enough to be able to predict exactly what someone's gonna do without the need for talking about things like free will and choices, etcetera, in exactly the same way that you could predict where the billiard balls are gonna go on a billiard table by knowing exactly what every molecule of the felt and the billiard balls and everything were doing.

1:12:29.8 SC: It's not practical, it's not gonna happen. I'm not worried about it, like in the list of things to worry about, that's just not on my list. But yes, it could happen in principle even then, even if that did happen, I think it's still extremely possible that it wouldn't be worth it, right? That it's still true, that we can talk about human beings as agents making choices in an enormously more efficient way. And so maybe there's some special conditions under which you really wanna read out the state of every neuron and every connection between neurons in a human brain, but it just isn't worth the effort usually, right? And in that case, once again, you would talk about people having free will. Spencer Hargis says, if a photon of light hits a telescope's CCD, what was the shape of its wave function right before that? Is it the surface of a sphere or hemisphere, the radius of the distance to the star? Well, it depends. You gotta be... You gotta think very carefully about this. It depends on whether or not you wanna treat this in a spherical cow kind of way, in an otherwise empty universe. If you have a star and nothing in between the star and us here on earth, then yes, absolutely.

1:13:40.2 SC: The wave function of the photon was in a kind of a spherical shell. It could be distorted a little bit depending on exactly the process that made it, but usually roughly spherical as it spreads across the universe. Now, what would prevent that from happening? What would, distort that is, whether it was in some sense measured along the way. By which we mean interfered or interacted with the environment enough to become decoherent. That's actually pretty easy to do. Photons interact fairly readily with charged particles, and there's a lot of charged particles out there in the universe. The problem is that they're... These charged particles out there in the universe are fairly thinly spread out. Here's the thing, when we're in ordinary quantum mechanics here on earth in a laboratory, the reason why the Copenhagen interpretation kind of sloppiness works so well is because there's a pretty cut and dried distinction between the microscopic realm where there's just a small number of particles and the particles rarely interact with each other.

1:14:47.3 SC: You can keep one particle from getting decohered if you want to, versus the macroscopic world where there's Avogadro's number of particles, some huge number of particles in a typical macroscopic system. And therefore it's pretty clear when a tiny system becomes entangled with its environment, the environment is so big and either it is entangled or it isn't roughly speaking. So it's pretty cut and dried when a quantum superposition has been measured out there in space, that's no longer true because in principle the... There's not a sharp distinction between having been measured or having decohered and not, it's a gradual thing in principle, if you only had a few particles in your system, then you could be a little bit decohered in some sense, and that's kind of the situation you're in out there in space. I don't know the numbers, so I don't know actually how many interactions actually happen with that photon?

1:15:44.0 SC: I bet it is a lot actually. So I bet it is effectively decohered long before it gets to us, in which case the wave function would not really be a sphere. It would already sort of be focused in the direction which we are seeing it. But honestly, that's a quantitative question and I don't really quite know the quantities. Sorry about that. Some dude says, are you a gamer of any sort, video games, tabletop sports, whatever. I was an old school gamer in the sense that I grew up begging my mom for quarters, so we could go to Spaceport and play asteroids and centipede and space invaders, and I got the original Atari when it came out and we would play adventure and other things, the various tank games that they had. But one changes over the course of one's life.

1:16:34.9 SC: And I'm not doing that much gaming anymore. I do play poker, although I don't... I haven't played poker live in a casino probably since the pandemic. Both because at first there wasn't any. And then I got busy moving across the country. So not as many casinos around here in Baltimore as there were in Los Angeles, but, and then I also just play cheap little games on my iPhone or iPad to while away the time just to take a break from doing science or thinking too hard. But nothing serious. Nothing that a real gamer would recognize as gaming. Ahmad Shaker says, is there a deeper physical reason why there are no massless Fermions? I guess that depends what you mean by a deep physical reason, masslessness having a mass equals zero is not the generic situation, right?

1:17:24.4 SC: There's only one value of mass, which is zero, but there's an infinite number of values of mass that are not zero. I think what you would expect in quantum field theory is that most particles have a mass unless there's some really good reason for them not to. Typically some symmetry that is preventing a mass from existing. So for things like gravitons and photons and gluons, there are symmetries that prevent those particles from having mass. That's why they're massless for Fermions genetic... The fermions that we have in the standard model are a special case, actually. They're pretty light compared to the Planck scale or whatever. So there's, even though they're not zero mass, there's still a question to be asked as to why they're so light in the first place. Why isn't their mass much larger than it could be? And the answer there is that there is a symmetry that in principle could prevent them from getting a mass the chiral symmetry.

1:18:16.0 SC: The difference between, well, a symmetry that acts differently between the left-handed part of the particle and the right-handed part, the part that is spinning clockwise with respect to its direction of motion versus counterclockwise with respect to its direction of motion. So if you treat those two parts separately, as is done in the standard model, the SU [2] part of SU [3] cross, SU [2] cross U [1] acts on left-handed particles not on right-handed particles, then you can prevent particles from getting a mass. That's why you need the Higgs mechanism. The Higgs mechanism breaks the symmetry and allows you to re-give the particles a mass that they would've generally had, if you hadn't had that symmetry in the first place. So as I think I said in the last AMA or somewhere, I remember saying it's not true that the Higgs boson or the Higgs mechanism is in general necessary to explain the existence of something called mass.

1:19:10.2 SC: Okay? They're not related. You can have mass without the Higgs mechanism. You can not have mass, even with the Higgs mechanism, like the photon still has mass. It didn't lose that masslessness just when the Higgs boson got its expectation value. It's a specific feature of the specific theory, the standard model of particle physics, where you need the Higgs boson to give particles masses. So there's no connection to the Higgs and gravity or inertia or anything like that. It's a completely different set of ideas. Go Messant says, under multiverse theory are all possible truths real? Short answer is no. But longer answer is it depends. Guess what on what you mean by multiverse theory? There's different multiverse theories. There is the cosmological multiverse where you just have regions far away, where conditions look very, very different. Clearly there not all possible truths are real. I mean, it depends if you mean by possible truth also, but there's no reason why in a cosmological multiverse you need to have, for example, regions where space is six dimensional.

1:20:18.4 SC: Maybe you do, that's absolutely possible, but it's not necessary. And we honestly don't know. It's not something that you have to have be true. Then there is the many worlds of quantum mechanics. And again, you don't get all possible truths. You get what is predicted by the Schrödinger equation. Of course, you can always go backwards after the fact and define the word possible as what the Schrödinger equation gives you. And in that case you get it. But there are plenty of configurations of stuff that just the Schrödinger equation doesn't lead to, so you don't get it. But there's also the possible, the multiverse theory called modal realism, where you think that all possible worlds exist. This is a favorite of philosophers like David Lewis and Max Tegmark has a version of it. So there, by definition, yes, all possible truths are real. There is, in my mind an enormous difference between the sort of possible world's ontology versus the cosmological or quantum multiverses.

1:21:20.3 SC: Namely that in the physics cases, in the cosmology or quantum cases, there is a mechanism that generates the multiverse. There are equations that tell you, you start from this position and you make a multiverse. Whereas in the possible worlds ontology, you're just positing that all possible worlds exist, because I don't know, it makes you feel better for some reason. So I put a lot less credence on that version myself. But if it were true, then all possible truths would be real. Blake Brazier says, do you collect or have ever bought original artwork such as paintings or sculptures? If so, how do you go about finding the artwork you'd like to acquire, and where do you keep it? I don't, it depends on, again. I hate to be all persnickety here all the time, but it depends what You mean by collect? I own a couple of pieces of actual paintings that a human being painted onto a canvas with a paintbrush.

1:22:17.2 SC: I'm trying to think how many, not too many. Jennifer, years ago was very good friends with an artist, Adam Sitonovich, I'm gonna not pronounce his name correctly. I never met him, but, he was a pretty successful artist and she helped him with various things and got paid in paintings. So she has a couple of paintings of his. And we purchased a couple of paintings along the way as we got older, one by Nash Hayyan, who did a whole series on the periodic table. So different, paintings devoted to different... Or devoted to is too strong, inspired by different elements of the periodic table. So we own Gadolinium in our living room. And one I picked up in Paris for a couple of hundred bucks, that just looks pretty. I don't know who did it, etcetera.

1:23:07.9 SC: So I wouldn't call us collectors in any serious way, we don't have an agenda. We don't go out looking for specific things and trying to build up some comprehensive set of things in any genre or artist or anything like that. We just have walls in our house that we like to put paintings on sometimes they're original paintings and sometimes not, depending on what we have to fill the wall at that time. So, as to where we now are in a bigger house 'cause we moved to Baltimore, and so we have more wall space. So in principle, we can put more paintings up there. We haven't really gotten around to that, it's more like the money that we have is being poured into keeping the house from falling down.

1:23:49.4 SC: That's an exaggeration. We're renovating the house, it's a 100 year old house. It needs some tender love and care, and it's almost there we're very, very close to having it, be in very good shape. The roof is gonna need to be done at some point. So hanging paintings on the walls has been less of a high priority there. I can mention the two ways I do like to look at paintings and contemplate buying them. There's some excellent places online like saachi.com. And there's another one, I'm gonna forget the name, but it's a wonderful resource you can put in here is how big I want a painting to be. Here's how much I'm willing to pay. Here's the primary color that I like, here is the subject of the painting, right?

1:24:36.3 SC: And it'll pop up with original artworks by people around the world that fit these criteria. Maybe you wanna spend a couple of 100 bucks, maybe you wanna spend a couple of hundred thousand bucks. That depends on you. But, and you can do your best, like they have zoomed in pictures. You can see what it looks like up close, and then they put it in context and everything. So I think that there's plenty of room for many more people to... Once you're past the struggling graduate student stage of your life, if you can afford a little bit, you absolutely can own some nice paintings for your walls. And the other place, of course, is in Santa Fe. When I go to visit Santa Fe. That's one of the world's centers for art galleries and things like that. And those will be a little bit pricier in general, but it's nice to look around.

1:25:19.7 SC: There's a place called Canyon Road in Santa Fe, which basically both sides of the road for, I don't know, a mile or two, are nothing but art galleries, which is wonderful with original art of all different kinds. I encourage anyone who visits Santa Fe to check that out. Alex Miller says are there any works of literature which you have found genuinely productive in thinking about the nature of fundamental reality? The Borges story, Tlön Uqbar Orbis Tertius, which you discussed in your most recent episode, was one such story for me as well as one I came to after Leonard Susskind praised it at the outside of a lecture... At the outset of a lecture, Kafka's investigations of a dog. I don't think so, at least I don't have any off the top of my head. This is a situation where it's very, very possible that I read a story years ago and it did actually affect my view of the fundamental nature of reality.

1:26:10.7 SC: And I've forgotten, I have to confess, I've confessed this before. I'm very, very bad at remembering stories that I read, even movies that I've watched, TV shows, whatever, like the plots and things like that of stories I love reading and I love watching, and I don't remember, I don't know why it is so very, very possible. I will, nevertheless, I'll give two examples, neither one of which relates to the fundamental nature of reality, but they're vaguely, within the realm of stories that I do remember, and I bring up when certain topics come together. One is in Julian Barnes' novel History of the Universe. In 10½ chapters, there's a wonderful chapter about Heaven, which I'm sure I've talked about before on the podcast, and spoiler alert, but I'll tell you what it... What basically the upshot of it is, which is that, you won't actually enjoy being immortal forever.

[laughter]

1:27:07.4 SC: That's the fundamental aspect of reality sort of point that he's trying to make. So you might think it'd be great to be in heaven and be alive forever and enjoy yourself, but it's much harder than you think. And of course, that theme was rehearsed once again more recently in the TV show. The Good Place, if you watch The Good Place, you'll get a similar vibe from that. And then the other story that is relevant to my thinking about the world is Isaac Asimov's Foundation trilogy, which is not for the reasons you might think, we had David Goyer on the podcast. He's the one who, is turning foundation into a TV series. There's no way to turn the foundation into a TV series. So he is very loosely adapting it, and that is getting people mad if they want an exact adaptation, but an exact adaptation would be horrible in my view.

1:27:57.3 SC: So he is wisely changing the story in certain important ways. But the story involves the idea of psychohistory. The idea that you can kind of imagine an analogy between thermodynamics and sociology, or psychology or political science or history, or whatever you wanna call it. The analogy being that in thermodynamics you can get rules governing the collective behavior of billions of particles, even though you don't know the specific details of every individual particle. Whereas Asimov wants to say the same thing could happen with human beings. You don't know the individual behavior of every individual human being, but nevertheless, you might be able to come up with psychohistory with a set of quantitative rules governing the evolution of society because there's so many people and their little idiosyncrasies average out. The reason why this is important to me is because it's wrong, because I get it.

1:28:51.1 SC: And when I read the story, when I was much, much younger, I thought that was a brilliant idea. I loved it. Now I know better. That's not a good analogy between Human beings and atoms, because human beings are individually complex, whereas atoms are not. So there's a new possibility that opens up in the collective behavior of human beings, which is non-linearities. Small, tiny changes can actually amplify and grow to enormous changes on the large scale in a way that just can't happen for atoms and thermodynamics and things like that. So it's an important cautionary tale. And that's just as important as an important insight. Mike Meyer says, what is the least interesting research project you have worked on? Or are there any publications of yours that you're not very fond of anymore? I guess I'm still fond of everything. I put some effort into every research project that I've worked on.

1:29:49.3 SC: Some have there's different ways in which a research project can be a success, right? One of my absolute favorite research projects to work on was on closed timelike curves in general relativity. I wrote a couple of papers with Eddie Farhi and Alan Guth and Ken Olum. And especially we wrote a short paper at the beginning to give a basic answer saying why in two plus one dimensional gravity. So gravity in flatland, basically, which is a simplification of three plus one dimensional gravity with infinitely long parallel cosmic strings. There was a paper by Richard Gott, an astrophysicist who pointed out that you could get closed timelike curves if the cosmic strings were moving fast enough. And we proved that you could only do that if they were sort of cooked into the universe from the start. Basically, if there was enough angular moment in the original configuration of cosmic strings, then you would get closed timelike curves, but they would've been there all along.

1:30:54.5 SC: You weren't making them out of nothing. And in fact, we showed that you couldn't make them out of nothing. And this was, I had a great time working on this paper. There was some... I contributed some like very nice insights, etcetera, and it just didn't make... Like no one caress really. Like no one was really moved to imagine that they were worried that you were gonna make closed timelike curves from infinitely long parallel cosmic strings. So no one paid a lot of attention to it. But I've had other papers where I put a lot of work in no one cared. And in retrospect, I was like, eh, should I really put that much work into it again? I liked it. I liked doing it in the moment, and I thought that the work that we did was good. For example, a series of papers I wrote with Wadi Taylor and Miguel Ortiz on two dimensional quantum gravity using dynamical triangulations.

1:31:43.6 SC: We, especially Wadi, who's really good at this, Wadi is the nickname for Washington Taylor, who's a professor at MIT now, and, came up with some very clever techniques for doing these matrix model versions of Euclidean quantum gravity in two dimensions, putting matter on them so they could couple in different ways, showing there were phase transitions and things like that. And, yeah, nobody cared. We did so much work on that and nobody cared. And I cared in the sense that I liked doing it. And I thought it was fun while we were doing it, but it wasn't part of my bigger research agenda, right? It didn't lead to anything for me personally. And I think that it was actually a valuable lesson. I say this all the time to my own students. I probably said it here on the podcast, but at some point in your training as a scientist, you go from not being able to write papers to being able to write papers.

1:32:40.5 SC: And that's a hugely important transition, and you can be very happy and proud of it. But there's another transition to go to, which is to write the papers that you should write, the papers that matter, the papers that have an impact and make a difference, and are building up to something more important. And rather than just writing papers, because you can write them and it'll be kind of cool, right? Because despite what you might think when you're 25 years old, you don't have infinity years ahead of you. You have to pick and choose what are the papers that you have to put your effort into. What are the papers that are worthwhile to do that? A more recent example from me would be, it's not very recent, but I wrote a paper with Mark Trodden and Kim Boddy that we were very proud of on strongly interacting dark matter.

1:33:26.7 SC: We had this idea when dark matter is strongly interacting, it's astrophysically interesting because maybe it changes details about galaxy structure or large scale structure, or clusters of galaxies or something like that. So people have been playing around with this idea for a long time. But if the dark matter is strongly interacting, then you naively do not predict the right relic abundance, right? The whole idea of weakly interacting massive particles is that the weak interactions give you the right strength of interactions to get the right amount of dark matter at the end of the day. So a lot of these scenarios just played fast and loose with that fact. They just posited the right amount of dark matter and let it be strongly interacting later. And we were, I think, a little bit more careful about that, and we really thought about it, how could you get the right relic abundance at the... At late times, even though the dark matter is strongly interacting?

1:34:24.5 SC: And so we had a mechanism where it was weak interacting at early times and strongly interacting at later times. And this was a huge amount of work to make sure that we calculated everything right and avoided all of the constraints from experiments and things like that. And again, Kim, who was the grad student at the time, she's now a professor at University of Texas, but she did a lot of the work here. And for nothing, like nobody cared about this paper. I'm a little shocked that nobody cared about it. I thought that was like very much in the mainstream, a thing that people cared about. But that, even though, and I don't mean anything against Mark and Kim who are two of my favorite people in the world, but that paper like ended my career as a particle cosmologist, I'm sorry.

1:35:07.5 SC: It just made me realize like, okay, if I only do have a finite number of hours left to do research in my life, I'm gonna do it on things that I care about more deeply than this. So it's not that there are papers that you're not very fond of, but it's the papers you learn from in the sense of, I could better spend my time doing something else. Let me put it that way. Nalita S says, do you see a future where the basics of quantum mechanics would be incorporated in schools curriculum? By which I mean maybe... Maybe you mean high school or I don't know, college for non-physics majors? I don't have strong feelings either way, honestly. I don't think you can do it right. I don't think you can do quantum mechanics quantitatively at the high school level.

1:35:57.8 SC: I mean you could, if you dropped everything, right? I mean, you could use my upcoming book, Quanta and Fields: The Biggest Ideas in the Universe or something like that, and you could say, okay, we're gonna give... Make you more or less comfortable with the basics of calculus and complex numbers and things like that and teach you enough quantum mechanics to solve the Schrödinger equation. I think you could do that for high school students if you did nothing else, if you taught them no other math and physics and just devoted yourself to doing that. But you don't wanna do that. I don't think you... There's other things to learn in high school. You have to pick and choose. So if you were gonna do quantum mechanics at all, it would be more hand wavy and more like giving people an impression of some of the ideas of modern physics.

1:36:42.1 SC: Now, even though I just said that in a slightly dismissive way, I actually think that's a perfectly good idea. I do think that forgetting about the specifics of quantum mechanics, I think that when you teach high school students physics, it need not be just a super watered down version of physics 101 right? Where you teach pendulums going back and forth and incline planes and all that stuff. There's no reason why you can't teach high school students or pre-high school students for that matter, things about relativity and quantum mechanics and black holes and the big bang. Or for that matter superconductivity or the structure of atoms or whatever is your favorite topic in physics, these people are mostly not going to grow up to be physicists. There's no reason why their physics education should be just the watered down version of Newtonian mechanics. But on the third hand, I do not know a lot about the high school curriculum, and I shouldn't really say what should be in it because there's always unintended consequences, right?

1:37:43.2 SC: There's always, things that are a certain way for certain reasons that maybe we've forgotten. So people who are much more expert than I should say, whether or not that would actually work to get people excited about physics at a young age. Elif and Lucas Wagner say, this is about the last section of something deeply hidden. You mentioned that time and space might be emergent from something else. Do you have any thoughts or ideas on how one might represent, as opposed to... I'm not quite sure I'm reading this question correctly, how one might represent a wave function from which space and time is emergent? Well, I think that's the interesting thing. You shouldn't is the short answer. In other words, when we start with wave functions, this is why I stumbled reading the question, because there's a parenthetical that says, as opposed to position momentum, etcetera.

1:38:35.2 SC: What this means is when we start with wave functions, we think about the wave function of a particle as a function of some observable quantity or a set of quantities, okay? So we think of a wave function as a function of position, or a function of momentum or a function of spin. Various things you could then observe about it. And then the value of the wave function for each observable quantity squared, gives you the probability. So I think the question is how would that work for emergent space time? But I think that's not the right question to ask because the point is, you're not starting with a wave function that already is a function of some concrete observable quantity. You're starting with a completely abstract wave function. What I mean by that is think about the wave function as a function of position, okay, Si of X, you know that you could write that as a function of moment instead.

1:39:31.7 SC: If you know a little bit about quantum mechanics, that's a true fact. Again, read my book, we'll talk all about it. So Si of X or Si of P, these are two equivalent, but different looking ways to characterize the same information. The quantum state of the system, they're not two separate wave functions, they're the same wave function written in different ways. They're the same underlying quantum state. So what matters is that underlying quantum state, not how you've represented it, representing things in different ways is just the choice of basis, right? It's the choice of coordinates. It's not something that is physically relevant. So mathematically, that wave function, that quantum state is a vector in Hilbert space. And then you can sort of, after the fact find out that it is convenient to represent that vector as a function of position or something like that. In the picture that we have of emergent space-time, the vector is not a function of anything.

1:40:31.0 SC: It's not even easily represented as a function of anything. It's just a vector. It's just a quantum state. It's an element of Hilbert space. And then there is a set of things you gotta do. There is homework for you as the working physicist to say, okay, how can we represent this as something that looks like something we would recognize as space? And the answer is you have to sort of factorized it. You have to divide it up into subsystems. And the subsystems all look like the set of quantum degrees of freedom associated with a point in space-time. Okay? Or a point in space, I guess. It then evolves over time. So the cheap and easy answer is, it's kind of like a lattice and it's kind of like a set of points in space, and you have degrees of freedom at every point, and that's what the wave function looks like.

1:41:19.2 SC: But that's not exactly true. It's not a... For one, a pre-given lattice, you have to sort of figure out what kind of lattice you have. And number two, it's not even really a lattice. The lattice doesn't even persist when gravity becomes important, right? When gravity becomes strong, like in a black hole or AdS/CFT, something like that, when holography kicks in, then it doesn't look like a lattice at all. So you just have to trust the quantum state. You just have to accept that that's what it is. You have to give up on your training, which is to wrestle that quantum state into some form that depends on something else, because that thing it depends on position, momentum, etcetera. Those are relics of your classical intuition. Those are not things that are really fundamental in this picture. Edward Seconjer says to prove Noether's theorem, which relates symmetry to conserved quantities, we assume that the equations of motion can be derived from the principle of least action.

1:42:13.6 SC: This is great for classical mechanics, but in quantum mechanics, the Feynman path integral also includes contributions from paths other than the one with least action. Could you please comment on the applicability of Noether's theorem to quantum physics? Sure. This is actually a good issue and it's one that in the book that I keep talking about, I kind of danced around because even though I mentioned Noether's theorem in volume one, I never proved it. I had a proof that I did in the videos, it just seemed to me to be a little bit unenlightening. It's like... It's more a demonstration of the author's cleverness than it is a useful pedagogical exercise. So I didn't, not my cleverness, whoever first authored these proofs of Noether's theorem. So I left it out of the book because I don't think you learn much from it other than yes, Noether's theorem is true.

1:43:06.5 SC: And then in the quantum version, you just have to prove it again, it's a different theorem, right? Sometimes depending on how careful your quantum field theory professor is, if you take quantum field theory, or for that matter, quantum mechanics, you will be carefully told. Well, sometimes you will not be told that the classical version of Noether's theorem doesn't suffice as you say in the question, in quantum mechanics, there are paths that are not the minimum value of the action that still contribute to the final answer. So why should we expect something like Noether's theorem some connection between symmetries in conserved quantities to still be true? The answer is you gotta prove it again, you can prove once again that there's an operator that gives you a value for these conserved quantities, and that operator is conserved over time as long as you are just, evolving according to the Schrödinger equation.

1:44:06.1 SC: But it's a proof that it sort of needs to be done over again. Once you stop doing classical physics and start doing quantum mechanics, Mark says, do you care how and for how long you'll be remembered? And if so or not Why? No, I do not care at all. And I don't mind if people do care. Like I do believe the following thing, that as we exist here and now, part of what makes us happy or sad or gives life value is our image of the future. Even though... Even if we insist that we will all someday be dead and there's no afterlife, etcetera, it's still okay to care about the future. In particular, I do care about the future long after I'm dead. In terms of things like the state of the planet and the state of the human race, those things I care about a lot. I don't particularly care about how I will be remembered.

1:45:09.7 SC: I hope that the people who knew me personally remember me fondly, that would be nice. But those people are gonna die too, right? So before too long, there'll be no one left to remember me. I don't care about even right now, like being famous. I wish I were less famous than I am, to be honest. The only reason why it's good for some small number of people to know who I am and recognize me is two reasons. Number one, it helps me sell books and podcasts, which brings me money and I like money. And then number two, more importantly, I get a feeling that hopefully I am improving these people's lives by teaching them science or telling them about cool things about the nature of reality, etcetera. I think that's a worthwhile project to do. And if I do think it's a worthwhile project to do, it's better to do it to more people than to less people, right?

1:46:00.5 SC: If I think it's useful to teach people science, then it's even more useful to teach more people science. But I'd be very happy if I could do that anonymously. [laughter] If I could get enough money to fix my roof and also reach a lot of people but no one knew who my name was, I'd be perfectly happy with that. Like in science, in academia, you have to make an impact if you want to get a job, right? If you want to be employed, you have to write papers that people recognize and go, oh yes, that person's papers are very good, but now I have a job, so I'm good with that. Right? So there's various instrumental reasons why it's important that people know your name depending on what your goals are. But to me, those reasons are not intrinsic to having people know my name.

1:46:45.1 SC: I get no special happiness out of people, more than five people knowing my name. That's not something. And when I'm gone, certainly not. Oh my goodness, no, I have no interest whatsoever personally. Travis Martin says, what is science? That's a good question. What is science? I think you have to be cautious when you ask a question like this because you're not gonna get a once and for all final answer. This is a whole research program. People devote their lives into figuring out what is science. There's famously the demarcation problem between science and non-science. Falsifiability put forward by Karl Popper purports to be an answer to the demarcation problem. It's not taken seriously by most working philosophers, but it, that's what it's aiming at doing. Separating science from non-science. When people try to argue against whatever intelligent design or crackpot theories or global warming denial, they will often try to argue against those bad ideas by saying, well, they're not really science.

1:47:48.6 SC: And they'll then sum some definition of science which tries to exclude them. I think this is kind of mixing, trying to understand the world with trying to make the world a better place, right? There is such a thing as science that is bad. [chuckle] So I have no trouble calling intelligent design science. It's just bad science. It's hilariously wrong science that doesn't there are ways to argue against it without saying that it's not science. Okay. With all that as is preliminary and therefore a promise that we're not gonna get the once and for all crisp, clean answer here. I think science is an attempt to understand how the world works in the very, very broad sense, based on a back and forth between hypotheses and data, between ideas about how the world works and input from experiment and observation on how it actually does work.

1:48:42.3 SC: That's a very subtle kind of thing. So I don't include math in science because math, you don't need any data to do math, right? It's related, math is very closely related to science. It's intrinsically important to doing science, but it is not itself science. You have to be careful about what you mean by getting the data 'cause people will say, well, wait a minute. You believe in the many world's interpretation of quantum mechanics. Yes, I believe in the many world's interpretation of quantum mechanics, because that is a prediction of the Schrödinger equation, which was derived based on trying to fit the data. So you have to understand the give and take aspects here. It's not just a one way street. You get data, you invent the simplest theory that explains it and you're done. It's a constant back and forth between improving your ideas, improving your sensory inputs, and trying to make things better.

1:49:36.6 SC: And furthermore, there are questions we have scientifically, to which the data don't tell us the answer. If you believe that there is dark matter, you don't know whether the dark matter is weakly interacting massive particles or axions or something else. And nevertheless, you have opinions about what is more likely, and you should have opinions 'cause you need to spend money and you need to spend your research time, as we were just talking about, you need to decide what it's worth working on. Right? So you need to have opinions about that. Where do those opinions come from? Where does that credence come from on axion versus weakly interacting massive particles. This is part of science, absolutely, but it's not very algorithmic, it's not very clear. It's not as quite as crisp and systematizable as people might want it to be.

1:50:21.0 SC: So science can be a little bit messy sometimes. Craig Stevens says, scientists at Cornell University recently announced the highest resolution photo of an atom ever taken. Given that atoms are not really particles, but instead probability distributions of waves, what exactly are they taking a photo of and what do they hope to see? So I'm not gonna answer this explicitly 'cause I have no idea what they did. I do not know what this photo is, but I will point out that people sometimes when they claim to be taking a photo of a wave function, in some sense, they cheat because you can't take photos of wave function, not in the sense that you can take a photo of a human being. What you could do, and again, I have no idea with whether this is what they did or not. What you could do is you could sample a certain kind of wave function over and over again to build up statistically a picture of what it looks like, right?

1:51:15.5 SC: You know that 'cause it's quantum mechanics. If you take just an ordinary image of where the electron is in an atom, you don't see its weight function. You see it at a point you could try to be more clever and have your observable not exactly be the position of the electron, etcetera. But one way or the other, when you make an observation in quantum mechanics, you do disturb the system. If you're doing something that is just an atom, which is very reproducible, right? You put the electrons in their lowest energy states and they're gonna be in exactly the same state in every version of that atom, then you can just do it again and again and build up what the wave function must have been to give you that observational probability distribution. Maybe they did something like that. I just don't know.

1:52:00.7 SC: Michael Monhab says, does the collapse of the wave function affect the gravitational field of the particle that is all of a sudden in a particular location? Yeah, yeah, it's supposed to. That's right. Although I have to hesitate a little bit because even people who believe in certain versions of the foundations of quantum mechanics, whether it's many worlds or anything else, don't necessarily agree on every detail. So in particular, in many worlds, people don't necessarily agree on should you imagine that the world branches simultaneously all at once in some reference frame versus branching inside of a light cone or something like that. But having put that caveat aside, basically yes, the wave function collapse affects the gravitational field everywhere. Note that this is not a big deal because you don't get a big gravitational field unless you have a lot of mass, right? Like a single electron's gravitational field is more or less completely ignorable. And once you have a lot of mass, you have a macroscopic system that is basically being monitored by its environment all the time. So in practice, in the real world, you don't get gravitational fields suddenly shifting all throughout the universe because of quantum measurement and wave function collapse. They've already been localized and they stay localized for realistic situations.

1:53:23.2 SC: Johann Falk says I have for some time been trying to understand why AI experts make radically different assessments concerning AI risks. I get that people who don't know a lot about AI have different conclusions, but I thought experts would agree more or at least not disagree wildly. I have two main hypotheses and would love to hear your thoughts. One, being an expert in AI technology does not automatically make you an expert in AI risks. And two, people have different predispositions when it comes to perceiving and reacting to low probability risks. I think both of these are true. I mean, especially number one, I think it's almost perfectly obvious that being an expert in AI technology does not automatically make you an expert in AI risks. Nothing to do with AI, it's just a feature of technology. The kind of skillset that helps you develop a technology is just not the kind of skillset that automatically makes you an expert in the risks of that technology.

1:54:15.2 SC: All you have to do is look at history and see all sorts of unintended consequences coming from new technologies. Maybe nobody is an expert in these risks, but certainly the fact that you've built something doesn't mean you know what it will do once you've built it. That's just a feature of these things. And I think that there's more to it than that also. I mean, there's a certain kind of person who goes into AI for certain predisposition reasons, and that might not line up well with the best judgment when it comes to what the risks are. Not that I have a very good idea of what the risks are but I do agree.

1:54:54.0 SC: Well, what you said is that people who are experts in AI reach different conclusions. That's certainly true. People who are experts in AI absolutely do not have a consensus as to what the risks are, how big the risks are, how to deal with the risks, anything like that. But I think in addition to that, I'm kind of disappointed in the level of discourse. I think that the people who work on AI seem no less than anyone else subject to over anthropomorphizing the AIs. I mean you heard about the guy at Google who got fired because he was claiming that a chatbot was sentient, right? There are philosophical issues here in what's going on in AI and how to interpret the fact that they seem so lifelike in some ways, and you can make pretty easy mistakes. And I don't think that the skillset that is required to sit down and code and write a large language model and or deep learning network or anything like that are the same as the skills that lead you to really think carefully about what the philosophical implications are.

1:56:07.1 SC: Physicists were not great at coming up with the implications of the atomic bomb, even though they helped build it, right? Why would they be? That's just not what they're good at. I don't know. I'm halfway contemplating doing a solo podcast on AI and what I think about it, from a semi-educated perspective, not a, I don't know how to code and write up a deep learning network, but I've read a lot and thought about a lot about AI and the philosophical implications of it. So if that's worthwhile, maybe that's something I will do. Aaron Berger says, I've heard that a scalar field is an object with some value at every point in space. I've also heard that according to some inflationary theories, we could live in a part of a cosmological multiverse in which there are numerous other causally separate universes.

1:56:51.0 SC: If these two propositions are true, doesn't that imply that every universe in the cosmological multiverse would have the same set of scalar fields? Is another yes and no question because if, well, for one thing, it depends on a certain view of the laws of physics, a certain kind of semi classical quantum field theory point of view, even in the presence of quantum gravity. So, yeah, but let's work with that. That's what most people do who are talking about this, that there are things called scalar fields that we all agree on what they are, the number of them is the same, etcetera. So the thing is that in ordinary field theory, there is a way of counting the number of degrees of freedom, basically a number, a way of counting the number of different directions in which fields as a whole can vibrate.

1:57:41.5 SC: Okay. But how you classify those fields as scalar or vector or whatever, is not in variant under different other external circumstances. So there are two important things that change how you might go about counting the number of scalar fields. One is that a certain degree of freedom can show up under some circumstances as a scalar, some circumstances as a vector or something very different. Classic example is in the good old standard model of particle physics. The original Higgs field has four degrees of freedom. It is what we call a doublet. There are two parts to the field, but it's a complex doublet. So each part is a complex number two real numbers. So four real numbers overall. And yet when you look at the Large Hadron Collider, you detect a single real scalar field that is the Higgs boson. What happened to the other three?

1:58:38.6 SC: The answer is that they were eaten by the W and Z bosons. A massless gauge boson only has two degrees of freedom. A massive gauge boson has three. And so what happens when the Higgs boson gets its expectation value, three of the degrees of freedom that were part of the Higgs field now become part of the W and Z bosons. So now they're not scalar fields anymore. Now they're part of the vector fields. So you can't invariably count the number of scalar fields. The other thing is the, there's a sort of question of how visible the fields are. If the fields are low mass, then they might typically be visible. But if they're very, very high mass, you just can't make them. They're kind of not relevant to you, even though they exist in principle, in the fundamental laws of physics. So at any energy scale where you have access to fields at that energy scale or below, but not above, you might be missing some of the scalar fields out there.

1:59:38.3 SC: So in practice, it is not true that there's the same number of scalor fields everywhere, even principle there could be deep down, but that doesn't matter to a single kind of observer who has access to certain things. Sean B says, sorry if this is a naive question, but I've been pondering a relationship between the many world's interpretation of quantum mechanics in the nature of dark matter, given that dark matter particles are massive, yet weakly interacting, could they be related to particles from separate branches of the wave function in many worlds, short answer is no, they cannot be. And the reason is because when you branch, when you deco here and split off different parts of the universe, the gravitational field goes along with you. So there is a gravitational field that is responding to the matter particles on your branch. And on a different branch, there is a different gravitational field responding to a different set of particles, the ones on that branch. So there's no interaction even through gravity, between particles on one branch and particles on another branch.

2:00:43.2 SC: LE McAndrew says, is it just a philosophical question or a reality as things line up within the judicial legislative, and maybe soon the executive branches of our democracy, sorry, of our government to destroy democracy, there appears to be such a headwind against the rule of law and facts that it is hard to see a clear way ahead. Well, I don't know if there's a headwind or not. This is a question of course about here in the United States where there are increasingly people, not only people on the street, but people in positions of power whether or not it's the executive or the governors or the local school boards or the judges or whatever, who are more interested in maintaining their power than in being loyal to the principles of democracy.

2:01:33.1 SC: The thing about democracy is it requires that the people in the democracy, at least a majority of them, be willing to lose elections. You need to be willing to fight for your beliefs and put out the best case that you have and put up good candidates. But then if you lose, you concede that you have lost. That is the thing that is increasingly disappearing. And I don't think it's equally spread between the various parties. I think it's pretty clear that one party has this particular perspective, and it is fed, I think mostly from the outside, right? I think that the people who are in positions of power, if the outside pressures had been different, would be perfectly willing to go along with the institutional norms, etcetera. But they're not especially courageous people. And when the external factors, whether it is social media or Fox News or conspiracy theorists with YouTube channels or podcasts, they buckled.

[laughter]

2:02:34.8 SC: They folded like a cheap suit, and now they're much more willing to just throw away the principles of democracy, such as sometimes you can lose elections. They're much more willing to do everything they can in their power to rig the rules of the actual voting so that it's impossible for their party to lose, even if they're in the minority. And I think that's a problem. I don't know how far it will go. I don't know if there's headwind there. It's certainly a recent change, but then there's a whole nother party that doesn't agree with that. And there's also still people within that party that don't want to go along. The obvious problem is when you have a somewhat powerful group of people who don't believe that they should ever be allowed to lose elections, once they actually get into power, they can rig things so that they never do lose elections. And that is something we absolutely have to guard against. I don't know what to say other than it is clear that most Americans are not motivated by a love of democracy.

2:03:37.1 SC: They might be motivated by love of their side on the democracy, but then and this is something that I think is true on both sides. So in terms of the political parties and the establishment, there's a huge imbalance in terms of which party is going to defend democracy and which one is not. I think that the voters on either side very often would be happy to get rid of democracy if they think that they could install their side as permanent victors. And that's too bad. I mean, that's really a shame. That's something that I don't like. I don't know how to change it. I mean, clearly we have not done a good job in this country in inculcating a sense of civic responsibility and an appreciation for the benefits of democracy. The thing about democracy is even if you do lose the system is going to be better for you in the long term for most people. So there's a short-term gain, long-term benefit kind of thing.

[chuckle]

2:04:33.9 SC: So short-term sacrifice, long-term benefit kind of thing, people don't wanna think that way anymore. And that is a shame, but I have no constructive suggestions here. I'm not an expert on how to defend democracy. I'm in favor of doing it, but I'm also a empirical data-driven kind of guy. And I don't know what the research says about how, what is the best way, or even if there is any research, the best way to get people to really buy in to this kind of system of government. It's not easy to do. And we kind of got lucky for a couple hundred years in having it work as well as it did. And now we are running up against some obstacles. So I'm worried and I don't have any constructive suggestions. I'm gonna group some questions together here.

2:05:15.4 SC: I anticipated these questions a little bit. One is Benjamin Barbrel says, one thing I enjoy a lot about your podcast is how your teaching skills help me understand the guest's arguments, even when the subject is quite subtle. I must admit that you somewhat failed me in the recent David Deutsch episode. Several parts remain quite obscure to me, which is frustrating because the topics felt very interesting. Do you think you could reformulate his point about the nontransitivity of support that feminist banker argument he used stumped me despite careful re-listening, and John Hague says, knock, knock, who is there? Linda. Linda who? Would you lend us a hand by clarifying the Linda example used by David Deutsch, please? So I'm gonna let you down...

[chuckle]

2:06:00.4 SC: On this question for two reasons. One is I don't understand the Linda argument, I don't understand. I tried, I read the paper by David and his collaborator against Bayesianism. I am personally pro-Bayesian. So I was resistant to whatever I was trying to say. I tried my best to open my mind and figure out what was going on. You can prove theorems. There's a theorem proven by Popper and Milder, but then there's sort of, okay, what are we supposed to get from that theorem? And I understand the theorem, but I just don't understand the leap to denying Bayesianism as a good way of doing epistemology. I think, I actually think there are better arguments against Bayesianism, even though I'm in favor of bayesianism as David correctly distinguished. No one is arguing with Bayes' theorem. That's a theorem, okay? But the question is, is the way to think about how we gain knowledge, sort of one that is a natural fit to Bayes' theorem in the sense that you say, what we do is we have propositions and we attach credences to all the propositions, and we update the credences as new data comes in.

2:07:13.1 SC: If your objection to that was something like nobody actually lists all the propositions, or nobody really has credences for all the propositions or people have credences, but they're not all internally consistent. They don't all add up to one for exclusive propositions and things like that. [chuckle] Or when people get new data, they don't actually update their credences using Bayes' theorem. All of those I think are perfectly valid objections to thinking of Bayesian epistemology as the way things work. My rough response to them is that I think it's the way things should work, not the way things actually do work, but I take the force of those objections. That's not David's objection. He wants to say that this idea that we get new evidence and just update our credences doesn't always correctly lead us to what he would call the best explanatory theory.

2:08:11.0 SC: And somehow he wants to connect it to this Popper-Miller theorem by the point of that theorem in his mind is to say that if you think about new evidence that comes in and you wanna say, okay, my new evidence supports a certain theory, right? Supports a certain view of the world, a certain proposition, what he wants to say is you can divide the theory for which you are looking for support into the parts that are directly supported by the evidence and the parts that are not directly supported by the evidence. And then he wants to claim that the new evidence, according to Bayesian logic, increases the support for the part of the theory that directly supports the evidence, but decreases your credence for the part of the theory that is not directly supported by the evidence. That's what he wants to say.

2:09:03.0 SC: And I don't think that it came clear, it came through clearly in the Linda example. So one problem is that I don't understand what the argument is, but the other problem is he didn't say it very well. I know that in the podcast he admitted that he misspoke at one point and he tried to correct it. I don't think the correction was entirely comprehensive. I think that example just kind of got a little garbled. So I would recommend if you're really interested in this stuff, reading David's papers or his book The Beginning of Infinity for example, and just trying to get into his mindset. It's a very minority point of view, by the way. Most not, I don't know most people, I don't know how you count, I don't know what the measure is, but some version of Bayesianism seems to me to be like the default epistemology among all sorts of different people.

2:09:54.0 SC: So that would be my point of view. Orin Harris asks a priority question. Remember, have we not gotten a priority question yet this time? Priority questions are once in your lifetime you get to ask a question and I will do my best to answer it. I don't get a chance to answer all the questions in all the AMAs. So Orin asks Bohr like complementarity in quantum mechanics and the relativism of special and general relativity are similar in their lack of realism about observer states, e.g., Wigner's friend versus relativity of simultaneity or black hole complementarity for in-falling versus external observers, etcetera. And for us realists Everett provides a realist account of quantum mechanics. While I'm aware of no analogous realist account of special or general relativity complementarity. So if you are unhappy with anti-realist accounts of quantum mechanics, why are you not similarly unhappy with the current metaphysical account of special relativity in general relativity?

2:10:49.6 SC: Well, I think that it's not so simple as to say a lack of realism about observer states. In both cases in general, relativity, in relativity, whether it's special or general and in Everettian quantum mechanics, they're realist about certain things, not realist about others. So in relativity, you're realist about spacetime. You're not realist about space or time individually. Those are relative to observers. Likewise, in Everettian quantum mechanics, you're realist about the wave function, the quantum state of the universe. You're not really realist at a fundamental level about branches of the wave function. Those are observed by different observers, right? That's why different Everettians will have different schemes for how the universe branches and so forth. The branching is a emergent higher level convenience to human beings. So I think in absolutely both cases, it's the same story that you have to be careful about what it is you are realist.

2:11:48.3 SC: You can't just say, oh, I'm a realist without saying a realist about what? And in both cases, I think it's clear what you should be realist about. Ari Modi says, my elderly mom, 89 years old, is a fundamentalist Christian and convinced we are in the end times. I am an atheist who rejects all of this. She's at the phase of her life where she's devastated that I'll be in hell for eternity and tells me this all the time. I'm struggling how to deal with this heavyweight on her heart. Any advice? Every time we speak, she begs me to renounce my atheism. And this is a horrible way to live for both of us. I think that this question, I mean, it's a tough question. I feel bad for you being in this position. I don't think it has a lot to do specifically with atheism or religion or anything else.

2:12:33.0 SC: I think it has to do with how you deal with people who react emotionally to your cherished beliefs, right? Which, what gives your care and your love for these other people or your loyalty to your cherished beliefs? And I don't think there's a hard and fast rule. I think that it's gonna depend on the circumstances. So one way of saying it is, I don't think that there's any moral obligation to be honest...

[chuckle]

2:13:05.5 SC: With your mom or with anyone else about what your beliefs are if you think it would make the world a better place. So, and this is a case, in other words, where I am a consequentialist, not a day ontologist. So if you think it would make your, the world a better place to just tell your mom that you're wavering in your atheism or you're open to religion or whatever, then that's fine if you wanna do that. But I also think there are two people to consider here. There's your mom and there is you, right? You both have some standing here. And if on the other hand you feel that not being honest about your beliefs would be a self betrayal of some kind, then I don't think that you have any obligation to tell your mom that you have renounced your beliefs. That if she is not going to be reasonable about accepting your beliefs, then someone's gonna be unhappy. And that's gonna be the way it is. So therefore, I don't think that there's any from on high hard and fast rule about how to deal with a situation like this. You have to try to balance, you have to try to weigh your feelings against her feelings. Would she even believe you if you said that you were renouncing your atheism would it make you miserable for the rest of your life if you were to do that?

2:14:22.8 SC: You're in a bad position and I do feel sorry for you about that. She's the one who's being unreasonable, no question about that. But she's 89 years old and she's elderly and you might wanna try to be considerate to her wishes. When people are reaching at age of 89, they're gonna be a little unreasonable. I'm sure I'm gonna be totally unreasonable when I'm 89 and I hope people learn to put up with me a little bit. Your hard decision is how much exactly you want to put up with her. I can't help you with that, but I do wish you luck. Ken Schneider says, what is the significance of the fact that the standard model can't be quantized? I do not know what you're talking about, Ken. The standard model is the quintessential quantum mechanical theory.

2:15:07.8 SC: Gravity can't be quantized all the way gravity can be quantized fine if it's just weak field gravity. That's part of the core theory. There's a perfectly good quantum theory of gravity that works here in the solar system as an effective field theory. But that's not part of the standard model. Standard model is just everything else. And it is quantized. There's one other loophole that is a little bit subtle. The standard model has in it, of course a U1 gauge theory which descends after symmetry breaking to electromagnetism. And that is not perfectly 100% well-defined. There's something called the Landau pole, which is a singularity that happens way, way above the Planck scale where you can't extend the rules of quantum electrodynamics to infinitely high energies. Quantum chromodynamics you can, it works perfectly well, non abelian gauge theories with asymptotic freedom you can quantize but not quantum electrodynamics all the way, but you can still quantize it as an effective field theory below some cutoff perfectly well.

2:16:08.7 SC: So I wouldn't worry about this. Standard model is super duper quantized. Rob Gabler says, could there be a consistent theory of gravity also allowing for anti-gravity, I.e., the force being repulsive instead of attractive for some particles, even it doesn't apply to our real world? And could that be compatible with both relativity and quantum mechanics, at least in low energy EFT regime? Well you can always write down a theory, which is exactly the theory that we have a general relativity plus various matter fields and just have as a coupling constant. You have Newton's constant of gravity, right? So you could just make that negative. You could just say, I'm gonna look at a world where G Big G Newton's constant is negative.

2:16:54.3 SC: And then when you plug into the formula for Newtonian gravity, you would get a repulsive force. But there's a problem with that, which is that that same coupling constant G also in the action in the action being the thing that we use to define the entire theory also tells you the kinetic energy of gravitons. So if we're thinking of specifically basically general relativity, we're not inventing a whole new theory of gravity, we're thinking just general relativity with a negative Newton's constant. Then not only do you make gravity repulsive by changing the sign of Newton's constant, but you make gravitons have negative energy. That's just bad. That's just not allowed because a negative energy particle means that empty space is unstable. This is a point that I made in my paper with Mark Trodden and Mark Hoffman years ago about phantom energy. You can't just make the kinetic energy of particles negative, it leads to wild instabilities of empty space.

2:17:55.5 SC: So that would be bad. Is there something more clever you can do. That I don't know, actually, I thought about this a little bit. But I don't know. I've tried. You would like to have something spatially dependent or something like that so that gravity can be repulsive in certain cases or it's hard though. And the basic reason is hard, it always comes down to the same basic reason that it's hard to make gravity repulsive while keeping the masses and energies of regular particles all positive. I don't know what to do about that. Ken Wolf has said, I've heard the mechanism of Hawking radiation described as a particle pair arising at the event horizon, where a particle of negative mass falls into the black hole and a particle of positive mass escapes.

2:18:41.6 SC: Is this a valid way of looking at it? And if not, why? Well, it's kind of a valid way of looking at it. I mean, it's fine. It's not the right correct final answer. The right correct final answer involves solving for the vacuum state of a quantum field theory in a black hole spacetime background, right? And ultimately that's the right answer. Do the equations okay. Then you wanna attach words to those equations to describe what's going on. The closest words you can come up with will have things to do about quantum fields and vacuum modes and boundary conditions and things like that. So that's not very intuitive. You can use all those words and you can get the right answer and it would be correct, but not a lot of us come across those things in our everyday lives. So what you notice in general relativity, just classical general relativity, is that you can define the energy of a particle as observed from infinity and you can attach a number to that.

2:19:44.1 SC: And if a particle is inside the black holes of event horizon, that number, the energy as observed from infinity can be negative, okay? This is a classical general relativity statement. It is just correct if you define exactly what you mean by energy. The energy of a particle inside the event horizon as observed from infinity can be a negative number. And when you look at the quantum field theory calculation that Hawking does, there's a flux of radiation out and a flux of radiation in and the radiation in has a negative mass as observed from infinity. That's why the black hole shrinks. So it is a little poetic and a little bit of an exaggeration, but nevertheless does track more or less what happens to say that a particle pair at the event horizon in splits apart and a negative mass particle goes in and a positive mass particle goes out.

2:20:35.5 SC: As long as you appreciate that, this is a somewhat poetic and colorful translation of an underlying careful calculation, you should be fine. Bob Zanelli says, axions are a leading cold dark matter candidate. What process can create such low mass particles as cold dark matter? One would think they'd be highly relativistic like neutrinos. Yeah, this is a great question. This really confused me when I first learned about axions. So for those of you who don't know, when we talk about weekly interacting massive particles, as I said before with my paper with Kim Body Mark Trodden about strongly interacting dark matter, there is a conventional story that we tell that tells us the relic abundance of such particles. And that conventional story is called thermal freeze out. At very high temperatures in the very, very early universe, all the particles were bumping into each other and they had some thermal distribution and they would both annihilate when they hit another antiparticle, but also be created when a particle decayed into them or two particles scattered and created them.

2:21:40.3 SC: So they are in some sort of equilibrium state. But then the temperature is of course changing 'cause the universe is expanding and cooling off. And at some point, which you can calculate, you reach a point where the particles just don't collide anymore. They just don't annihilate anymore because the universe is, the too low and the density is too low. So the probability of any one particle bumping into an antiparticle and annihilating just is effectively zero. And that is called freeze out. It's very sensitive to things like the mass and the cross-section for annihilation of the particle, but you can do it, you can calculate the relic abundance of these particles and what you want to get, what you get for a good dark matter candidate is a relatively heavy particle with a weak interaction cross-section that gives you the right relic abundance.

2:22:30.4 SC: Now axions are super light particles, the mass of an axion in the most popular models, we've not detected the axion yet. We don't even know if it exists. So I shouldn't act like it exists. But in the most popular models for axion dark matter, their masses are very low. In fact, the mass of the axion is comparable to maybe a little bit less than the energy of a typical photon in the cosmic microwave background. What that means is in the early universe, if the axions were in thermal equilibrium, then they'd be moving at the speed of light. Their energies would be much, much higher than their mass which means they're moving very close to the speed of light. And even today they'd be close to the speed of light of order the speed of light, they wouldn't have slowed down very much.

2:23:17.9 SC: Therefore, if their particles moving close to the speed of light, they are not cold, dark matter, they are super hot, dark matter. So what's going on? How can axion be a leading cold dark matter candidate? The answer is they were never in thermal equilibrium, even though the universe was a hot, dense state. In order to be in thermal equilibrium particles have to be able to interact with the surrounding plasma well enough to equilibrate. If you have two objects that are different temperatures and you put them into contact with each other, they equilibrate and they come to the same temperature. That's true. Even if your "Objects" are two different species of gas in the same container. But if you don't put them together, if they don't interact with each other, then the two things are not gonna come to the same temperature.

2:24:04.6 SC: So axions interact very, very weakly. So they do not equilibrate with the rest of the universe. And the way that they're produced is just entirely different. It is not freeze out from an initial high temperature state. There's a whole story, I'm not gonna tell it here, but there's a whole story about how axions arise from spontaneous symmetry breaking. And then there is a moment, typically the QCD phase transition when the axion starts oscillating back and forth as a very low mass zero temperature Bose-Einstein condensate of particles. But you can still do the same kind of calculation. It's not, sorry, not the same kind of calculation. You can still do a calculation that tells you what the relic abundance is. And that's why we have opinions about things like what the mass of the axion should be, because we know that the relic abundance at the end of the day depends on the mass of the axion, and we know what we need the relic abundance of dark matter to be all of this still speculative of course, but it does hang together very nicely.

2:25:07.6 SC: So the short answer is the axions were never in thermal equilibrium. They were created in a completely different mechanism. Adam Rotmill says, if the theory of eternalism holds up where moments in the past, present and future are all equally real, then what happens to our sense of identity? Like, are those parts of ourselves still conscious in their local time coordinates? I suppose Hume called this the notion of resemblance that we continue to resemble ourselves even though we keep changing. But you've said in the past that different branching copies of ourselves are different people. So where should we draw the line on identity and why? Well I think that the whole idea of identity, in my view of the world, is a higher level emergent phenomenon.

2:25:45.8 SC: So it's going to be imprecise, right? Just like tables and chairs are imprecise. At what point do you stop calling a table, a table if you remove it, atom from atom, atom from atom, from the table. Likewise in a world where people are changing what exists, what really exists? Forget about quantum mechanics in many worlds and stuff like that. You have atoms and molecules with world lines persisting over time. But you know that a human being is not the same collection of atoms over time. So it's not that this particular collection of atoms is your personal identity. It's some kind of pattern that is formed by the atoms that gives you memory and personality and physical continuity through time, etcetera. So I think that's the right way to think about it. And that's going to be, I'm not quite sure what eternalism has to do with it, honestly.

2:26:44.1 SC: Whether you're a presentist and think that there's some continuity between your present self and your past self, but only the present self really exists, or you are an eternalist and you think that they both have an existence, the story is the same as far as I can tell. I would just warn you against saying things like those parts of ourselves are still conscious because I know you put, so you don't, you folks listening to this don't know, but Adam put the word still in quotes, don't even put them in quotes.

[laughter]

2:27:12.5 SC: In the moment when some previous version of you exists, that version of you exists, right? And that moment of existence has just as much reality as the current moment of existence, but it doesn't exist at the same time or simultaneously or still. They both just are real any more than this location in space exists at the same location as some other location in space, even though they both exist and are real. Just think of it that way. Chris Rogers says, beyond the event horizon, does a black hole have a surface? If not, why not? 'Cause I don't understand. Well, I think it'd be a little bit more specific about what it is you don't understand, Chris. A black hole is a region of spacetime, a region where anything that goes in can't come out without moving faster than the speed of light. That's the definition of a black hole. It's not an object, it's a region. In many ways, it acts like an object. So we know why you might think that it is object-like, it has a mass, you could move it around, it radiates and things like that. But what it really is, is a region of spacetime. So no, there is no surface beneath the event horizon. There is a singularity, but as I've often remarked, the singularity is in the future, not in some location in space.

2:28:32.1 SC: So the singularity is something to which you will evolve once you're inside the black hole. Not a surface you could imagine standing on. Valanorian asks a priority question. What do you think of my new past eternal cosmological model? And there's a link to a Wikipedia page. In it, all ancient koshy surfaces of the default kind are as nice as the bounce surface in your Carroll-Chen Model. And there is no reversal of the arrow of time. I don't have any thoughts about it. 'cause I don't know.

[laughter]

2:29:03.9 SC: I don't know about it. Sorry, I don't know anything about it. So I can't really say anything. I think, by the way, I will say parenthetically that the reversal of the arrow of time is not only allowed, but actually good. I think that the idea that the arrow of time points in different directions in the far past, in the far future is actually very nice. It adds a symmetry, restores a symmetry to the overall cosmological history that we believe the underlying laws of physics have. So I think that's a good thing. So getting rid of that, I don't think to see as any particular kind of virtue. Adam Small says I was having a conversation here with the community about emergence and it's appears from a whole bunch of basic units of, let me see, sorry. And that it appears from a whole bunch of basic units from whatever macroscopic thing you're talking about. So then I said, do a thought experiment and remove one of the particles or units at a time until you don't get emergent behavior. One of the smart members said that's called the sorites paradox. Just wondering how you think about that and what it means. It seems to really mess up the concept of emergence since by definition it can't depend on a single particle or unit of the whole. I don't think this is a paradox.

[chuckle]

2:30:13.7 SC: The sorites paradox is a well known thing. It's just a reflection of the fact that, as we just said, higher level immersion phenomena are approximations that are going to be valid in certain circumstances and not. And the thing about approximations is they don't stop instantly being valid at any one point. They just become worse and worse. It's like saying, how tall does a person have to be to be tall rather than short? Different people are gonna disagree on the exact number, the exact number of centimeters or inches in and feet that you have to be tall. That doesn't mean that the concept of being tall is useless, right? It conveys information. It's not precise, it's approximate information. It might be slightly different information for different people, and yet it is still useful. So ideas like tables and chairs are much more well-defined than that, but they share this property that they're not precisely defined.

2:31:09.1 SC: It's not a boundary. For those of you who don't know the sorites paradox is anyway my way of thinking of it, you have something like a table, you agree it's a table. Now you remove an atom, is it still a table? Yes. Remove another atom, still a table, yes. But if you keep removing atoms, eventually you'll get down to just one atom when is clearly not a table. So at what point do you cross the threshold from being a table to not being a table? That's a silly question. It just becomes less and less table-like. There's no threshold for you to cross. Elliot Spect says, I enjoyed your podcast with Mari Ruti in which you discussed Lacan's version of psychoanalysis. The analysis is based on universal origins for human angst posited by Lacan. You asked Ruti whether this theory was scientific, I.e., Verifiable, and she conceded that it was not, she described it as mythology.

2:31:58.7 SC: This answers just half the question. There are certainly useful ways of thinking that are not scientific. My question, should critics such as you demand that scientific techniques be used to determine whether Lacan's analysis really is useful. Anecdotal accounts seem like a weak basis for choosing a medical treatment? Well, there's a lot going on here. I actually have a quite expansive definition of what is scientific. As I said, I have no trouble thinking of Lacanian psychoanalysis as scientific. I think of it as very similar to traditional Chinese medicine, for example. I can absolutely imagine that there are cures or not diagnoses, but prescriptions from ancient traditional medicines, whether Chinese or anywhere else that help you, that help you get better. And I could even imagine doing a test, doing a double blind experiment that shows they help you get better.

2:32:52.9 SC: And I can imagine that all this happens without us understanding why it helps you get better, right? I don't know. I actually have no strong opinions about whether or not acupuncture actually helps. My understanding is that no one understands why acupuncture should help. To me, that's no barrier to it helping. I think there's a separate question. If it does help, then that should be testable, right? That the fact that it helps should be verifiable. But again, the verification is kind of not on or off. It's not yes or no. You build your way up to it. If there is any psychological theory, psychological theory, not just psychoanalytic, it might be very difficult to test it. But whether or not it helps people at the level of therapy, that's an empirical question that you can just go out and test. So whether or not the you understand why it helps is kind of irrelevant.

2:33:51.4 SC: So I may or may not be convinced by the specificities of one or someone else's psychoanalytic or psychological theory, but I'm pretty loosey-goosey when it comes to asking that it be verified. I don't think you need to collect rigorous quantitative data or have a mechanistic understanding of what lies beneath. I just want it to work. And work means something that is actually verifiable or demonstrable using empirical testing. Tim Converse says, great to hear about your collaboration with Scott Aaronson. I understand from Aaronson's popularization that there are hard physics-based limits on how much computation you can do in a region of space. My question is, if some civilization were trying to max out their computational resources and driving right up to physical limits, might there be observational signatures that would indicate that was happening? Something analogous to observing Dyson spheres, but for computation rather than for energy?

2:34:50.3 SC: I haven't thought about this very carefully, but my short guess is no. And I'll tell you why. Because if you really think about maxing out your computational resources, there are ways to do computations that are what we call reversible. Okay? They do not actually generate entropy. But most computations and in fact the interesting ones, the ones where we involve not just calculating something but then reading out the outcome of the calculation putting into memory, things like that, those do require entropy increase and therefore heat and therefore radiation. And I'm betting without having thought about it very carefully, that the most efficient way to do this would be to just have the radiation you're giving out, be black-body radiation at a certain temperature. So at the end of the day, even though you're doing something very advanced and fancy and technologically sophisticated, what you're emitting out to the universe is black-body radiation.

2:35:50.5 SC: And that comes from a lot of places, Dyson spheres, but maybe other things as well. So I don't think it would be an especially unique or noticeable signature out there in space. Again, I haven't thought about this in any detail, so I could be wrong about that. Casey Mahomes says, if I'm doing the math correctly, you met your wife relatively late in your life. Did you ever feel hopeless about finding love? I don't know if you're familiar with the current dating climate at all, but the world of dating apps seems like a complete waste of time and it feels challenging to meet other people once you are out of school. Any words of wisdom? Yeah, no I did not ever feel hopeless about finding love, but I felt perfectly content with the idea of remaining single for my whole life.

2:36:32.9 SC: I thought I was having a good time. I always thought that it would be better to be in love with somebody and have a partner and things like that, but I wasn't despondent, nor did I rely on that for my own personal happiness. So I was doing fine I think. One thing I was always devoted to was not rushing into getting married or a long-term committed relationship just for the sake of having a marriage or a committed relationship. I wanted it only to happen if I was with a person who I knew that it's the person I wanted to be with, not the institution of marriage or commitment, etcetera. So I was comfortable being relatively old and still being single. I am not that familiar with the current dating climate, and I absolutely acknowledge that the dating climate is something that changes over time. So what it is like now might be radically different than what it was 10 or 20 years ago, much less when I was in high school or college or something like that.

2:37:33.4 SC: So I feel for you. I feel that it is difficult, especially as you say, once you're out of school, college, and even graduate school is an easy place to meet people. You're still pretty young when you're in college, so it might not be the best place to meet people who are exactly right for you. The one thing I will say I'm all in favor in principle of dating apps and things like that. That is a way to meet people. There's obviously one huge advantage of them, and one huge flaw, and you should be aware of both. The huge advantage is by construction, The two people who meet already agree that they're looking to be in a relationship or at least looking to date, right? They're not. It's not like you're gonna go on a dating app, meet somebody, set up an arrangement, and then come to meet them, and they say, oh no I'm already in a committed relationship with somebody else.

2:38:21.8 SC: At least not unless they're just yanking your chain or something like that. So that's a huge head up. Like, if you meet someone at work or in a social situation, you might not know whether they are looking or committed or whatever. So, huge benefit to the dating apps that way. There's also a huge disadvantage which I think is not given a lot of attention, which is that we human beings are not that good at knowing what it is we want, especially when it comes to looking for possible romantic partners. We tend to think in terms of a list, right? There's a checklist. I want someone with this and this and this, and that's never right, the things that make two people compatible with each other are very rarely reducible to items in a list. Or if they are, you only discover them after you meet the person and go, oh, that's what I needed all along, right?

2:39:15.5 SC: You need some kind of personal compatibility with a person. I've noticed when we, for example, accept new graduate students into a physics department, I've often noticed that we put a lot of work into accepting the right students, right? And we look at their file and their scores and their grades and their letters of recommendation and their personal statement and whatever, and then I meet them and in both directions, both like, oh my goodness, we've let this person in and we shouldn't. Or, oh my goodness, we didn't let this person in and we should, 10 minutes of talking to a person about physics is more informative than their entire file, no matter how carefully you look at it. And I think something very similar happens with dating people. There is such a thing as a personal connection. There are things you have to be open to the fact that you, or the possibility anyway, that you cannot articulate what it is you're looking for.

2:40:08.1 SC: That's not bad. That's not a terrible flaw. It would be nice maybe if you could articulate it, but you have to be open to the fact that you don't know. You have to be open to the fact that you're meeting people, getting to know them, and changing your mind about what it is you were looking for. And I think that dating apps really sort of pretend that this list-based way of finding human beings is the right way to do it and the lists might be, it doesn't need to be like, oh, you have certain height, a certain income, or anything like that. Even just a list of your interests. I'm looking for people with similar interests. Well, are you really [laughter] Is that really the most important thing? Maybe we'll find somebody who has very different interests, but nevertheless, you're compatible being with them and you can begin to share their interests.

2:40:57.5 SC: So I think that dating apps have not at the current state of technology really given us a window into what it is that people actually require in terms of making a connection and moving forward. So the only thing to do about that is keep trying, the nice thing, like I said about dating apps is at least you know everyone on them is trying to date. So go through them very quickly once you're older, by the way, everyone realized every, most people recognize this phenomenon when you were young and you started to read a book, you felt you had to finish it. You started it later on in life, you're like, nah, this book [laughter] isn't working out for me. After two chapters, I'm putting it down. Think likewise about people. Save yourself some time if you know someone is not working out, go to onto somebody else, right?

2:41:45.4 SC: Like, if you're already in love and invested and whatever, then try to work it out. But early on, I think it's okay to move quickly. Dan Berliner says, you mentioned in an earlier AMA that one of the ways gravity is special is that it interacts with everything. Does this mean Gravitons would interact with themselves, and if so, would they not experience confinement in the same way as Gluons? So yes, gravitons do interact with themselves. And in some sense, again, these are stories that we tell. This is not the math underlying the stories, but in some sense, it's that interaction of gravity with itself that makes general relativity a little bit different than Newtonian gravity that gives rise to the difference in the procession of the orbit of mercury in Einstein versus Newton. But that doesn't mean that they have to be confined.

2:42:33.3 SC: That depends not on whether there are interactions, but whether the interactions are strong or weak. And really, when you get into it, it depends on how the strength of the interactions changes with energy. The thing about the strong nuclear force is not only that its interactions are strong, but that they get stronger as the energy gets lower. Okay? As you go to lower and lower energies, which is longer and longer distances in quantum field theory or particle physics, the strength of the interaction goes up. So when you try to pull two quarks together, the force pulling them toward each other the energy in that force grows enough that eventually you just make a quark anti quark pair and you've not actually separated the quarks. Just like when you pull apart the ends of a piece of string, if it breaks in the middle, you don't have two unaccompanied ends.

2:43:23.4 SC: You just have two pieces of string with two ends each. So that's a feature that you need to get confinement. And that feature feature does not exist in gravity. Gravitons interact with each other, but only very weekly and un noticeably when the energy becomes very low. In fact, guess what? Photons do the same thing. Photons interact with each other. They don't interact with each other directly. But there are such a thing as virtual particles. And two photons can interact by exchanging virtual electrons and protons and positrons, sorry. So, or for that matter, by exchanging virtual protons and antiprotons. But that's a much tinier effect at low energies below the mass of the electron where you don't see real electrons. It is as if photons are just interacting with each other, but again, their interactions are so weak you would never notice.

2:44:15.7 SC: It's a theoretical calculation you can do. It's not something that is very relevant to the actual behavioral electromagnetic fields in the macroscopic world. Mike McManus says, is supersymmetry dead? Nope. It is not dead [laughter], its credence has gone down since the days of even just 20 years ago. 20 years ago, there was still a lot of room both for discovering new super symmetric particles at the large head run collider and discovering particles in dark matter detectors that would be eventually explicable in terms of supersymmetry. Neither one of those things has happened. So by any rational accounting, whatever credence you used to put on supersymmetry, you should put less now because you could have discovered it there and you didn't. But it's certainly not dead. It's absolutely possible that supersymmetry is a real part of the world. It just is broken at very, very high energies and has nothing to do with our low energy world to many working particle physicists.

2:45:16.7 SC: That's a shame if that turns out to be true, because low energy supersymmetry was really useful. It helps explain the hierarchy problem, which is the difference in mass between the Higgs Boson and the plank scale. It maybe helps explain what the dark matter is. It was just good for all sorts of reasons. Plus it gave full employment to particle physicists building accelerators. But the idea of supersymmetry is much more broad and robust than just this specific sub idea of low energy supersymmetry, even low energy supersymmetry is not dead, but it's on life support. I would put it that way. We could have found it very, very easily, and we haven't. So you have to stretch your imagination a little bit to imagine that it's there. But high energy supersymmetry is still 100% alive. Jeff b says, I'm trying to wrap my head around emergent Space-Time.

2:46:05.2 SC: I think I understand that you are defining the wave function to exist in a more abstract space of degrees of freedom. And these degrees of freedom happen to be connected in such a way that they produce the appearance of Space-Time dimensions. My question is, if the degrees of freedom are so perfectly coordinated that they mimic Space-Time geometry, wouldn't it be more logical to simply assert that there is something called Space-Time, which ties these degrees of freedom together? I maybe could have grouped this question with the previous one about immersion Space-Time, but the answer is no, [laughter], it is not more logical to simply assert. There's something called Space-Time that ties these degrees of freedom together. Because the thing about emergence, this seems to be a theme of today's AMA, it's in a, it's a higher level approximation. It is not the fundamental thing.

2:46:48.5 SC: So if Space-Time emerges from some more fundamental sets of degrees of freedom, it will very likely plausibly do so in some regimes, but not in others. So it's crucially important that you have some degrees of freedom, and they are only emerging into Space-Time, rather than in some sense being Space-Time from the start because it gives you more flexibility. And the evidence is that that flexibility is going to be useful because our notion of Space-Time seems to be different when it comes to situations where holography is important, like in a black hole or in a cosmology, or probably at the Big Bang in places like that. So it really does matter, which is the more fundamental thing, which is the emergent thing. Valor up says, if space flight in the near future becomes commercially feasible, would you participate? Would you visit a space hotel or a hotel on the moon?

2:47:38.7 SC: What sort of fun and interesting attractions could you see being presented? In principle I would, but there's a race against time here, and I'm losing [laughter] this race against time. I don't think that feasible space hotels or hotels on the moon are coming very, very soon. I don't, by very, very soon, I mean two decades from now, right? So I'm gonna be in my 70s two decades from now. And I don't think that flying in a spaceship to the moon and hanging out there will be the best for my long-term life expectancy. I think it's okay. It's fine. Again, It was never, my motivation since I was a kid was never like to go to space. I like reading stories about it. I like reading about astrophysics and cosmology, but my interest is in how the world works, not in visiting outer space.

2:48:33.2 SC: So if there's a finite number of slots on the spacecraft that would go to the moon or whatever, there are people out there who would get more out of it than I would, and I'll be a little bit over the hill by the time that time happens. So in different circumstances where I was super healthy and I was absolutely convinced that the travel was completely safe, then I would be 100% in favor of it. I would love to do it. It sounds like fun, sounds like a little adventure, right? I like going adventures like going to different countries here on earth. So of course I would like an adventure that would go into space, but again, I'm not really convinced that it will be safe. I think that it will happen before it becomes safe, right? It's easy to imagine especially if private companies can do it and make money out of it.

2:49:17.5 SC: We know from how private companies work that they're more interested in making money than making things completely safe. And it might be 90% safe, but 10% not safe is a huge amount of not safe. Space travel is dangerous. It just is. And maybe someday it won't be. But our current version very much is. So I am happy to enjoy my life here on earth. Too many good things I gotta do to risk it all for one little trip to the moon. Hail Zeus says, I really enjoyed something deeply hidden, and I have a question that I don't think you explicitly addressed in your book. When solving for the time of flight of a thrown ball, we get one positive and one negative answer, and we discard the negative answer as non-physical, which no one loses sleepover. There are numerous other examples in physics of suddenly throwing out wrong answers by analogy.

2:50:06.1 SC: Then what is the issue with throwing out some of the answers to the Schrödinger equation? I think I understand the logic of if the Schrödinger equation is perfect in all encompassing then many worlds, but the if statement here seems to be carrying an awful lot of water, yeah, this is a very insightful question, I think, but the situations are not as analogous as you are making them out to be when it comes to forget about Space-Time and the time of travel. Think about pythagoras' theorem, right? The hypotonus C of a right angle of right triangle compared to its shorter sides. A and B satisfies A squared plus B squared equals C squared. So straightforwardly looking at that equation, you say, well, A and B could be positive, but C could be negative. And I could solve that equation with a negative distance between the two points.

2:50:54.9 SC: So what, like, [laughter], you're missing the point. The distance is just something that is defined to be positive. It happens to obey that equation, but it's positive, even though, we know it's positive, even though there is a negative solution to the equation that's a little bit different than a situation like the Dirac equation where there are different solutions which we now know represent positrons and as well as electrons, right? That was an extra solution, which you, it would've been a mistake to throw away. People thought about throwing it away 'cause they didn't know the positrons existed, but that would've been a mistake. The Schrödinger equation is actually a much simpler example because there aren't extra solutions. The Schrödinger equation is linear, it is not Si squared equals something. So you can find Si or minus Si, it's just s Si equals something. You can figure out the solution and then you gotta accept the solution or not.

2:51:48.6 SC: So you can change the Schrödinger equation. People try to do that. Those are the objective collapse models. But if you don't change the Schrödinger equation, you get the multiple worlds. That's all there is to it. It's not a matter of getting extra solutions that you're throwing out. The solution includes multiple worlds. That's a different situation. Only normal person says. Could you elaborate on how moral constructivism gets through the art from his problem? It seems like an accurate way to describe constructivism would be, it is true that almost all humans have moral inclinations, even if specific intuitions are not universal. So we ought to work to make our inclinations as coherent and consistent as possible. That certainly works for me intuitively, but I feel like I must be missing a piece of formal logic based on your other statements. Yeah, I would argue that yes, you are missing a piece of formal logic here.

2:52:38.9 SC: I never say we ought to make our inclinations as coherent, as consistent as possible. I have goals and desires. So to me, it's not that ought doesn't exist, ought can exist. It can't be derived from is if I say a certain thing ought to be the case or ought to happen, I cannot do an experiment to show that I'm wrong as I would be able to do if I were making a scientific statement about the behavior of the world. That doesn't mean that I can't talk the language of oughts I can say, you ought to do this, but what it means is something different. What it means is within this framework that I personally, subjectively, individually have come to accept based on my human intuitions and inclinations and whatever the history of myself as an individual and descendant of a species, etcetera, I've come up with these ideas about what ought to be the case and what not ought to be the case.

2:53:37.1 SC: And I can tell you what they are. What that doesn't mean is that I've derived what ought to be true from what is true in the world. I'm just expressing how I feel. And I don't think that, I think that the problem in deriving ought from is, so let's put it this way, to me the closest that you can come to arguing that you should be able to derive ought from is, is to say, look, is is all there is [laughter] by being a naturalist, by being a physicalist. You're saying that all that exists is the physical world, right? So everything that is true should follow from what is true in the world, including the ought statements. That sounds almost convincing there. And the mistaken it is to think that ought statements are true things that are derivable, that's the problem.

2:54:30.4 SC: So they're not, they are different kind of thing. They're judgments. Judgments are not true false statements that are derivable in the same way that empirical statements or mathematical statements or whatever are. So, I think it all fits together. Just admitting that you cannot derive ought from is. Laurent deLamare says, I enjoyed watching the YouTube replay of your debate on is consciousness fundamental with Philip Goff on September 8th. Your humor makes dry subjects so much easier to understand. I may be biased, but I feel like Philip's way to convince people that he's right is by being super passionate at exposing strange arguments. My question to you is, why do you think he never answers you? When you ask whether he wants to change The Core theory or not change it, it's a simple enough question. Did he answer that elsewhere? No, he doesn't answer that. I think he's actually pretty explicit that he doesn't answer that, and that's fine.

2:55:22.1 SC: He's honest that he doesn't answer it. He says that maybe The Core theory will have to be changed. Maybe it won't. His attitude is that consciousness comes first, that The Core theory may or may not have to adapt to it, he doesn't know, but what he knows is how consciousness works. I just don't think this is a sensible attitude for a panpsychism to have, because I think that we understand, well, it's not just I think, it's obviously true that we understand The Core theory enormously better, and we have enormously more empirical support for it than any statement we can make about consciousness. Other than that it exists any specific statement about what consciousness is, how it works, anything like that. These are much, much harder claims to make than anything about The Core theory. So I think that it just makes sense intellectually that if you have something that is extremely well established and something else that is not really well understood, then you'd better be clear about what you're doing with the well-established piece before you draw conclusions based on your intuitions or thought experiments about the not very well understood piece.

2:56:33.8 SC: So, I disagree with Philip's stance, but at least he's clear on what his stance is. Go Messent says there's a set of philosophers/scientists out there who believe that free will is an illusion. Sapolsky comes to mind as the champion of this view. What I've never heard them explain is the Hitler problem, as I call it, relative to the non-existence of free will, that is, doesn't the lack of free will give Hitler a pass. So I get the argument, I think what you're getting at is the, is the following idea that when we talk about morality or we talk about ethics or we talk about judgment more generally, right? This is good, this is bad, this is right, this is wrong. It does seem to rest on the possibility that things could have been different, right?

2:57:23.3 SC: If you say, well, you did the wrong thing, and I say, well, I didn't have any choice but to do it. Like, if someone throws me out the window and I land on another person and injure their back and in general, injuring someone else's back is the wrong thing to do. But here, I had no choice. I was thrown out the window, the law of gravity took over. I'm kind of blameless for that, right? And if you take a very strong opinion about free will that it just doesn't exist. You never have any choices. There are no choices, then there absolutely is an argument that no one is to be blamed for anything right? Now I think that some anti-free will people kind of just avoid this, right? Kind of don't face up to this issue. And you can give them a hard time for that.

2:58:09.6 SC: I do think that others take it seriously and they do have a response. And again, my understanding might be imperfect here, but the idea is that they're basically consequentialist. They're saying yes, we are not attributing praise or blame to individual people. What we are instead doing is saying, what can we do to make the world a better place? Like what is the right thing to have happen to improve the conditions of the world, to make sure generally positive outcomes happen. So you don't need to blame Hitler, but you can still stop them. You can still stop Hitler from doing what he did maybe even killing Hitler, maybe even throwing him in jail or whatever. But certainly the fight against Hitler can be completely justified, even if you don't believe in free will. Even that little bit of slightly better response has the problem that you're trying to persuade people, [laughter] of this point of view, but you've admit that people don't make decisions, including yourself.

2:59:09.2 SC: You haven't really made the decision to try to persuade people. I'm not sure if there is any coherent landing spot for people who don't believe in free will and who still wanna get through the day making choices of their own, trying to persuade other people to make choices, etcetera. But that's not my job to make that coherent. That's their job. Daniel Bagley says, I'm a layperson reading Hawkings a brief history of time in the chapter on the origin of fate of the universe. When he talks about the inflationary period after the Big Bang, he says, the different regions should not have been able to achieve the uniformity we see now because the information would not have been able to travel faster than the speed of light. But couldn't entanglement have played a role such that particles weren't exchanging information but were simultaneously affecting each other?

2:59:52.4 SC: Or does this represent a fundamental misunderstanding of entanglement? Sorry to say, but yes, this represents a fundamental misunderstanding of entanglement. Entanglement does not allow you to push around one particle based on affecting the other particle. In particular, if I have two particles that are entangled, so I know that let's say that there's the spin of two particles is entangled, okay? But one particle has a location far away from the other. I can take the particle that is far away, one of the particles, so their spins are entangled. I can move it, I can manipulate it, I can pick it up. I can put it in a basket, I can put it electric field. I can do all sorts of things to it. As long as I don't measure its spin. Literally nothing happens to the other particle. There's no force, no influence, no communication between them.

3:00:42.5 SC: The only thing that would eventually be an effect is if I measure the spin of that one particle, and then I know what the spin of the other particle is. But guess what, that doesn't make them more smooth or homogeneous. It just brings to life one of the options that were already there in the wave function of the combined two particle system. So there's no way that you can actually use entanglement to smooth things out if they fundamentally weren't, or to communicate from one part of the universe to the other, what their local condition should be at some certain moment in time. So it doesn't help in particular.

3:01:18.2 SC: In smoothing the universe out at early times in the way that inflation does by good old-fashioned local physics. Matt Smith says, could you please help me understand why the thermodynamic arrow of time is necessary to explain why we perceive there to be a past and future. We are part of a universe and the act of us experiencing something and it being committed to our memory is itself a physical process, a physical process that will like all physical processes be reversible. I appreciate that the thermodynamic era of time explains why it is overwhelmingly unlikely that a physical process is as complicated as seeing an egg breaking and creating a memory of that could happen in reverse. But it seems to me that a more fundamental point is that even if the physical process were to happen in reverse, then we wouldn't have the memory.

3:02:05.0 SC: Yeah, I think that this whole story about the arrow of time and memories and things like that really doesn't make any sense unless you accept a distinction between the microscopic level and the macroscopic level. At the microscopic level, All of these processes are reversible, like you say, but at the macroscopic level, they are not, they're not reversible in a thermodynamic sense, in a probabilistic sense, but still for all intents and purposes, they are irreversible. And that has crucial consequences. But the argument is subtle to be honest. You can say, as I've said, I wrote a whole book about it from eternity to here that you're welcome to read where I go into these in great detail if you care about them. But the point is that in you, you are saying on the one hand that entropy is increasing and that gives rise to an arrow of time.

3:03:00.8 SC: But on the other hand, the arrow of time has a lot of manifestations like causality, aging, memory, etcetera. It is still work to show why the fact that entropy is increasing is the reason why you have memories of the past and not the future. But you can do it. So for example, think of a simple example. Think of a footprint in the sand, right? A footprint in the sand is in some sense a memory. You see it and you go, aha, I bet there was a foot there at some point. In other words, the way it works is there's some feature of the current macroscopic world, the footprint from which you can deduce something about the past, the existence of a foot that stepped there. You can't deduce everything. You don't know when the foot went there, but you can say something's about it.

3:03:45.3 SC: It wasn't five years ago or it would've washed away by now, if it's a fossil, that's a different question. And you don't have that same access to the future. You can't say from the existence of the footprint now what it will be like a day from now. Maybe it will wash away. Maybe an asteroid will hit the earth and destroy everything. You just don't know. Maybe another foot will step there and put another footprint there. So where does that asymmetry come from? And the answer is from entropy increasing. Because when you take the footprint in the current moment of the universe and ask what you can infer from that about both the past and the future, when you're inferring about the past, you have an extra piece of information, an extra handle that tells you something. And implicitly or explicitly that extra handle is the fact that the early universe had low entropy from that fact, from the combination of number one, the early universe had low entropy.

3:04:44.7 SC: And number two, there is a footprint here in the sand today. You can infer number three, that there was a foot that put the footprint there. And the way that you do it, and again, no one does this explicitly, but this is the underlying idea, is you consider all of the different ways in which the fundamental laws of physics could have ended up with a footprint there. And you ask what is most probable given the boundary conditions of the low entry past and the footprint? And it turns out that with those boundary conditions, the most probable intermediate trajectories involve feet stepping on the sand. You can't do the same thing with the future because there is no future boundary condition of low entropy. The universe, as far as we know, just wanders off into some state in the far future, and that gives you much less of an epistemological handle on what can happen in the future.

3:05:35.5 SC: So it is ultimately all because entropy is increasing. If you were Laplace's demon and you knew what every little micro atom in the universe was doing exactly, then you would have perfect knowledge of both the past and the future. And you wouldn't talk about memory or causality or entropy or any of those things, but you are not Laplace's demon. As I often remind people, Philip Malinowski says, in conversation with David Deutsch, he was talking about an ever ready interpretation of quantum mechanics in the Heisenberg picture, when observables are evolving instead of the wave function at the level of basic quantum mechanics, the Schrödinger and Heisenberg pictures are equivalent, but they seem to suggest different interpretations of quantum mechanics. In your favorite case of many world's interpretation, is there a difference between Schrödinger Everett and Heisenberg Everett? It feels, for example, the Bowie interpretation would be hard to reconcile with the Heisenberg picture.

3:06:25.9 SC: I haven't thought about this very deeply, but my belief is that it is not just at a level of basic QM that Schrödinger and Heisenberg pictures are equivalent. I think they are exactly equivalent. I think that there's no sense in which it, the theory works in one picture and doesn't work in the other. It is very frequently and commonly the case in theoretical physics that you can have two equivalent pictures and one is more helpful than the other. And the word helpful can be interpreted in various ways. It can be helpful in the sense of solving the equations, or it can be helpful in the sense of suggesting ways to expand the equations and go beyond the picture you currently have. So there are often very good reasons to work in one picture rather than the other. But at a philosophical Level.

3:07:14.8 SC: At a deep ontological level where you're asking about what the world is, I truly think there is no difference. Jason Ricciardi says, I liked your X post formerly Twitter that states responses to this have been kind of depressing. I have my own degree of skepticism, but there's been way too much superficial dismissal. Ambitious new theories will often seem weird. Knee-jerk reactions make it harder to investigate truly novel ideas. This is Sean talking now, something that I put in response to the assembly theory paper that appeared in nature. This is a paper by a collaboration of people, Lee Cronin and Sarah Walker. Sarah Walker is a former Mindscape guest. Were the most were the people most closely associated with this assembly theory idea, but also Christopher Kemps and Michael Lockman are two other friends of mine from the Santa Fe Institute who worked on it, and also Abhishek Sharma and Danielle Zagel worked on it.

3:08:14.1 SC: So they all have some involvement here. Don't, it's always a mistake to sort of collapse a paper to it's most famous authors, by the way. Anyway let me finish Jason's question. Can you explain how Dr. Cronin's assembly theory, see, I don't like that, it's a shared assembly theory and their tech stack at Chem Phi, which I have no knowledge of challenges, the status quo. What are the novel ideas versus already established ideas, and why has it caused friction in the community? So for those of you who don't know, assembly theory is, a set of ideas about how complex structures are assembled, thus the title in the real world. And I have, like I say in the tweet, I have my own degree of skepticism, but I also think that there's a lot to it, and I don't understand it very deeply.

3:09:03.5 SC: I've not written any papers about it or anything like that. But what they're pointing out is that the space of all possible complicated things you can make is very, very big. Okay? And this is something, I mean, it's a trivial statement, but it's also a very, very important statement, and one that I myself have been thinking about quite a lot. The space of things you can make is very big, and therefore, it's important to consider the ways in which things get made, because we don't make, by we, I mean, the universe doesn't make every complicated thing and try it out, okay? There is a history that helps explain why certain things are made and certain things are not. So, assembly theory tries to take advantage of this fact by pointing out that if you see certain kinds of configurations of complicated things on some other planet, then the most likely way to make those complicated things under certain conditions, which they try to explain, is because there is life, there is some element of design, even if it's just the illusion of design that you often get from natural selection.

3:10:07.9 SC: It's not just random selections of different chemicals coming together. There's some process that is making them, and that could be very helpful when you're looking for life on other planets. It can also just be helpful trying to explain the evolution, I guess, of life here on Earth. What I don't like is some advocates of assembly theory and here Sarah Walker and Lee Cronin, I think are guilty, are way more willing than I would ever be to say that somehow this view suggests that we should modify the underlying laws of physics. No longtime listener of Mindscape will be surprised to hear that I am very skeptical about the idea we should go from something complicated like the origin and evolution of life to something basic like, let's change the laws of physics. I think that's utterly wrong. There's no justification for that. It's kind of silly.

3:11:03.6 SC: And the paper that appeared in nature a little while ago, kind of doesn't push that aspect of the theory very strongly, but does allude to it, and it has some provocative sentences in there. So what I was disappointed about that I indicated in the tweet is, sure, you can be skeptical of some of the more outrageous aspects of this structure. You can be absolutely critical of some of the ways the theory was presented in this paper and how it was written, but it, there's still good ideas in the theory. And rather than talking about personalities or writing styles or whatever, talk about the ideas. I don't care whether it's challenging the status quo, and whether it's young upstarts versus established ideas, who cares? Who cares about any of that? I care about whether it's right [laughter] And most of the time when you have a new idea, it might be at best, partly right and partly wrong.

3:12:04.5 SC: So, okay, so you do the work. You say like, what aspects of this are useful and which are not. The thing that I didn't like about the knee jerk reaction on Twitter and elsewhere was an idea. Basically, there is a feature of interesting new ideas that they are too easy to dismiss. And look, I know as well as anyone that this kind of statement is one that crackpots love to make, because crackpots will always say, you're just dismissing my ideas without paying close attention to them, right? So there's a judgment call that comes in. I am totally in favor of dismissing crackpot ideas. I'm totally in, also in favor of taking seriously ideas that are not crackpot, but are a little bit out of the mainstream. And there's a judgment call that comes in, in distinguishing between those two cases.

3:12:57.6 SC: To me, assembly theory is clearly on the side of not completely crackpot. There's parts of it that I disagree with, but there's some interesting stuff there. So if you disagree with it, that's fine. People who I know and admire on Twitter including some former Mindscape guests who I won't name, appeared on both sides of this argument. Some were saying like, ah, this is just nonsense. Others were saying, no, actually, here's why. Here's the kernel of an interesting idea there. That's fine. I mean, the disagreement is fine. It might all turn out to be useless and wrong for all I know, but there was just too much dismissal on the basis of style and not enough dismissal on the basis of, or not enough engagement on the basis of the substance. And I think that's too easy to do with new interesting ideas to engage with their style rather than with their substance.

3:13:52.4 SC: And by doing that, we make it hard to propose new ideas. Let's put it this way. Something that anyone who has ever tried to write an interesting science paper knows, is that it is way easier to get an uninteresting science paper past the referees than it is an interesting science paper because interesting and uninteresting are orthogonal to correct and incorrect [laughter], right? You can be interesting but incorrect. Interesting, incorrect uninteresting. Also, likewise, if you are uninteresting and correct, that is the easiest way to get your paper published, because some journal is absolutely gonna publish your paper if it's uninteresting and correct, if it's interesting and possibly correct, but not, obviously it is much harder to get your paper published because there could be some referee that just looks at it and goes, eh, this is not the way I usually think about these things. Reject.

3:14:47.5 SC: Clearly. I mean, obviously I can give you many examples in my own career where the boring papers I've written sail through the refereeing process, and the interesting ones sometimes have trouble, sometimes they're accepted 'cause people get it, but sometimes they run into trouble because people just don't want to engage with something that is new and different. And I think that's what that's part of what the assembly theory paper is running up against. People just find it too easy to sneer and move on with their lives rather than think hard about a new idea. Castor Pollocks said, is there a way to revert a black hole to normal matter like a neutron star or just a regular star? Or once you go black, you never go back. I'm talking about the theoretical possibility, not a practical solution. It depends on how theoretical you want to be.

3:15:36.8 SC: A black hole is the highest entropy configuration that you can have within a certain sized region of Space-Time. So if you want to limit yourself to physical processes that are compatible with the second law of thermodynamics, you have to make higher entropy configuration than the black hole. And you can't make a higher entropy configuration in a limited region of space. You can, if you give yourself more space. So the reason why black holes can evaporate is because they're evaporation products, the photons, etcetera, that come out, spread themselves over a much larger region of space and have a higher entropy than the original black hole did. If you wanna take that black hole without spreading it out all over space and turning it into a neutron star or a regular star or whatever, you will be decreasing its entropy. Therefore, it's not gonna happen in the real world.

3:16:34.7 SC: But of course, as Boltzmann taught us, it could happen. It's possible. It's just super duper duper unlikely. So don't actually try to do it. It's, when I say it's hard to impress upon people how super duper unlikely it is, it's not anything that you should even waste a moment of your time thinking about. Let's put it that way. Giorgio says, I was wondering how you, on a personal level deal with the discrepancy of personal wellbeing and the suffering of so many people around the world. How can I ever be happy knowing there are humans around the world who never in their lives have felt secure or protected and live in constant fear of pain or worse, especially, of course, in regards to recent events around the world. I often find myself incapable of enjoying my own life that is so very different. Do you struggle with similar feelings?

3:17:20.9 SC: And if so, how do you deal with that? I think it's very valid to struggle with these feelings. But I think I'm pretty consistent here. I am going to advocate for a diverse ecosystem. [laughter] that diverse ecosystem applies to your personal feelings and activities as well as it does to other aspects of life. So I think it's important to care about the suffering of others elsewhere in the world. I also think it's important not to be overwhelmed by the suffering of others elsewhere in the world. You're not doing them any good by just being overwhelmed by their suffering. If you want to quit your job and move around the world and try to help other people that's great. That's a thing to do. I don't think there's any moral obligation to do that. I do think that you should try to make the world a little bit better, but I don't think that the way to act in life is to look around the world, decide what is the worst thing going on, and devote yourself to fixing that thing that is absolutely an okay thing to do.

3:18:25.6 SC: In fact, it's a very admirable thing to do. Don't get me wrong. I just don't think there's any obligation to do it. I think that different people will have different roles to play in the world, and there are different puzzles to be solved, different problems to be solved. If I didn't think that, I certainly wouldn't be a theoretical physicist or philosopher, right? I think the problems that I devote myself to are important, but they're probably not the most pressing problems in the world right now. They are some intersection of important problems, problems that I am personally able to make some progress on, and problems that I enjoy trying to make progress on. So there's some combination of that. I also allocate some fraction of my time and attention and monetary donations to other problems that I think are worth pressing on.

3:19:19.2 SC: So I will talk about other things. This is, as you might have noticed, this podcast is not purely about physics and/or philosophy. It's about lots of different things because I think many things are important at once. So I think that it's perfectly okay to care about these things, to feel bad that the whole world is not happy. That's something that is absolutely valid to care about, but the solution is not to let that paralyze you. And the solution is not to feel guilt about it. The solution is to sort of live a life that makes you happy while also working to make other people happy in whatever little way that you can. Let's put it this way, if everyone did that, the world would be in enormously better place. Kevin Contreras says, I have a philosophical question involving maths, thinking about how mathematical models can make sense internally, but don't correspond with our experience of the world, or just can't be proven.

3:20:12.9 SC: It makes me wonder why does this happen? It's maybe like math is like any other language in the sense that it can't really grasp concepts outside of our experience, like they're just an extension of our own capacity and by default prone to our own errors. Or are they really, or do they really, or some there's a misprint in [laughter] in this sentence. Sorry. Do they really, are they really something separate that we are just not interpreting correctly? I do struggle with this idea of the objectivity of math without getting into questions of the realism of math, right? I think that this is an important distinction I learned from Justin Clark Don, you can have, in principle, you could imagine that math is objective without being a mathematical realist, okay? I do think that math is objective. I do think that if you have a set of axioms and you prove theorems from it, that is an objective fact in the sense that every single person who understands those axioms and the rules of math would prove the same theorems, or would at least agree that you have proven those theorems.

3:21:15.4 SC: And I don't think that those theorems necessarily need to have any connection with the real world, the world might be finite, but mathematically we can still talk about infinity. Let's give that as a particular example, but there might be many other examples. I do think, it's at least possible to me, I would like to suggest the possibility, even though I can't advocate it fully, because I'm not an expert here, that there are various puzzles and paradoxes in mathematics and mathematical logic that arise from the fact that precisely the fact that we take math and push it beyond what happens in the physical world. And maybe we shouldn't be bothered by these puzzles and paradoxes if they only arise in the parts of math that don't correspond to anything in the physical world. I don't know know if that's a valid perspective or not, but I would at least like to pull it up the flag pole, see if anyone salutes, Chris V says, it can be argued that it is morally possible to become a billionaire.

3:22:16.9 SC: However, do you think it is morally defensible to remain one over time, a billion being simply an arbitrary threshold? Sure. Yeah, I have no problem with that. In fact let me extend, let me use your question, Chris to rant about this a little bit. I think that in the modern world, there is too much inequality. I think there are too many super rich people and far too many poor people. I think that's true, but I also think that people who object to this state of affairs are far too prone to making it somehow about individual virtue. I think that's just a huge mistake. I think that putting the onus on wealthy people to feel guilty and give away all of their money is in some sense, in some very real sense, making the problem worse, or at least providing cover for the problem to continue to exist.

3:23:11.4 SC: This is a societal level problem. This is not an individual level problem. If a certain individual spends their time complaining about billionaires being too rich, etcetera, maybe that makes them feel better and makes them morally superior to think that they would give away all their money that they don't actually have. And they feel good because they're giving away other people's money and their imaginary thought experiments. It's not making the world a better place. The way to make the world a better place is to change the system. Sorry, that's a little bit less romantic, a little bit harder to do. People try to do it all the time. It rarely works. I get it. It's still the right way to do it. And by changing the system, I don't mean revolution and overthrowing everything. I just mean having taxes that people pay.

[laughter]

3:24:00.7 SC: I said this before and I still think it's true. Billionaires are not the problem. The problem is taxes should be higher on billionaires and they should pay them, and then we can use that money to do good things in the world. This is a resolutely unromantic point of view. I get it. And furthermore, I absolutely feel the objection that people try this and it doesn't work. Okay. But guess what? People also try guilting billionaires into giving away their money, and that doesn't work either. So I don't think that's a great argument. I think that the best possible hope for more equitable society is to work at the level of changing society, not working at the level of making individuals feel bad. Walter Miller says, is there a magnetic field that permeates the entire universe? And if so, what generates that field? Well, I wouldn't say there is a magnetic field that permeates the universe.

3:24:49.3 SC: There are, there is at every point a value of the magnetic field. There's one magnetic field that takes on different values in different places. Some places it could be zero, some places it might not be. I think the scientific version of this question is, are there magnetic fields, let's say in between the galaxies, okay, in galaxies, there are processes like supernova and star formation and just good old stellar processes that generate some magnetic fields. And there are dynamos and batteries that people in astrophysics talk about and study in great detail. Neutron stars, for example, have extremely strong magnetic fields. What we wanna know is, are there primordial magnetic fields? Were they there in the early universe? And do they, are there still remnants of them in between the galaxies where there are no stars to generate them, etcetera? We don't know. There's some evidence, there's some tentative astrophysical evidence that this is true, that there are some magnetic fields out there.

3:25:46.4 SC: They're not too big, so they're not like crazily inexplicable or anything. But there's some motivation for thinking about how to generate primordial magnetic fields that could help us explain the existence of magnetic fields in between galaxies. I know about this, 'cause I just wrote a paper about it, this is generally not what I do these days, but my advisor from grad school, George Field, and I just did collaborate on a way to possibly make magnetic fields, by the way, using axions, using the same axions that could be the dark matter. There's, this is an old idea that if you have axion like fields, you can generate magnetic fields through the evolution of the axion. It is generally thought that that would have to happen in the super early universe before inflation or something like that, or during inflation, because otherwise you have a charged plasma.

3:26:38.3 SC: It's very highly electrically conducting, and it's hard to make fields when you have a in the middle of a conductor, okay? They're just shorted out by the motions of the particles. What we point out is that in the presence of this axion field, there's kind of a new degree of freedom. There's a mode of the magnetic field or the electric field, really, that is shorted out by the motions of charge particles. But there's also one that isn't. There's a new degree of freedom because of the axion. So you can have some fields that are not shorted out and they can actually grow. And we talk about how much they could grow by, I don't know if that's the right theory, but if we find that axions are the dark matter, I think it's be very interesting to check whether or not they're also responsible for intergalactic magnetic fields.

3:27:27.1 SC: Joe Ader, or Ader, A-D-E-R says, do you believe the lone genius idea drives scientific progress more than collaborative science? Do you think this has shifted over time? Do you think awards like the Nobel Prize are helping or hurting with respect to this? I do believe there are lone geniuses. I don't think that they play a huge role in the progress of science. Here is one little piece of evidence for that comes from Jeffrey West, our former Mindscape guest and former president of the Santa Fe Institute. If you look at the number of good ideas that are generated by people you can figure out how you're gonna measure that by patents or Nobel Prizes or whatever. Guess what, in cities or in dense concentrations of people, not only are there more patents in Nobel prizes because there are more people, there are more patents in Nobel prizes per person that come out of highly collected, highly concentrated sets of people than you would expect than there are in dilute sets of people.

3:28:30.5 SC: And in retrospect, this makes no surprise at all because people talk to each other. Even the geniuses, even the Einsteins, the Newtons, whatever, they absolutely build on the ideas of other people, there were a lot of people of Einstein or Isaac Newton level IQs back there who lived 10,000 years ago. None of them invented Newtonian mechanics. Why [laughter] Well, because it takes a lot of buildup. It might be imperceptible, it might be cumulative in very, very background kind of ways. But Newton knew about Kepler's laws. He knew about Galileo's experiments. That stuff matters. And so, even if it's not immediately visible, I think that science advances through collaboration and communication. As far as the Nobel Prize, I don't know, I like the Nobel Prizes 'cause they bring attention and publicity to good science that might not otherwise get it. They certainly also distort science in various ways. I have no way of judging whether the positive aspects are bigger or smaller than the negative ones.

3:29:37.0 SC: Adam small says If a bubble chamber was placed between the slits in the screen and double slit experiment in the same particles that produce tracks in a bubble chamber or pass through the slits, would you see interference or would you see the tracks as they emerged from the slits? If I understand your set-up, you would just see tracks going from the slits, because a bubble chamber that can detect particles is certainly measuring the particles. Once the particles are being measured in between the slits and the detection screen on the other side, you have measured them they're now particle like they have tracks, they're not gonna interfere with each other, there are no longer waves. Yeses asks a priority question, can the CMB that run the energy from the early universe be the reason for dark matter, and if not, why?

3:30:23.5 SC: I don't know what you mean by the reason for dark matter, but putting that confusion aside, the answer is no, it cannot be. For the reason that the CMB is not matter, it is radiation. When astronomers or cosmologists use the word matter, they don't just mean stuff for very particular reasons, mean stuff that is moving slowly compared to the speed of light, and cosmology speak, matter is particles that are moving slowly compared to the speed of light, radiation is particles that are moving at or near the speed of light, so very, very low mass neutrinos can still be radiation, even though they're particles and photons are just not matter because they're always moving at the speed of light.

3:31:09.7 SC: The reason for that distinction is the amount of energy density in matter versus radiation, thus construed changes in a different way as the universe expands. If you're moving slowly compared to the speed of light, then your energy comes mostly from E=MC squared from the mass of you as a particle, and if you have a bunch of particles who each get their energy from their masses, then the energy density just goes as the number density of particles, which goes down as the volume of the universe goes up, whereas if you are moving at or close to the speed of light, your energy does not come from E=MC squared, your energy comes from your kinetic energy, or if you're moving at the speed of light, just from your frequency as a wave, and therefore you Redshift, you lose energy. As the universe expands, and as a result, the total energy density goes down both because the number density particles goes down as the university expands and because every particle is losing energy, so do a cosmologists. Matter is particles that don't individually lose energy as the universe expands, radiation is particles that do move, they do lose energy as the universe expands.

3:32:28.6 SC: One thing we know about dark matter is that it's matter. In fact, not only is it slowly moving compared to the speed of light now, it was slowly moving compared to the speed of light in the early universe, if it hadn't been, it would not have settled into galaxies and clusters and made the large-scale structure we observe today, so the micro-background radiation cannot be the dark matter. Ryan Santos says, as an educator, do you find that variety in course work and school activities improve student performance, if so, do you have any theories as to why this is? I don't have a lot of data about this, and I am a believer in data, so you should not really believe my anecdotal impressions compared to someone who's actually studied it, but my anecdotal impression is that probably, yes.

3:33:17.4 SC: Most learning happens in the minds of the students, not in the words of the professors or the teachers, or the reading or whatever. If the student is not going to engage with the material. Then they're not gonna learn as much. And I think the variety is important for two reasons, Number one, different students become engaged for different reasons, so you never know what is gonna engage a particular student, and number two, even one particular student might get bored of the same kind of thing over and over again, right? Whether it's a lecture every day or problem sets every day or whatever. Variety is the spice of life. So it's a variety because you want to keep the students engaged, that I think sounds useful to me, but I don't have a lot of data one way or the other. Owe, O-W-E says, I'm trying to spin up on post modern philosophy, and I was reading through the Wikipedia page as a starter, it states, "many post-modernists appear to deny that an objective reality exists and appeared to deny that there are objective moral values. The latter seems reasonable to me, but the latter seems."

3:34:25.6 SC: I think maybe there's a mistake here, one of them, it says the latter in both cases, but I think that probably what Owe was trying to say is the latter seems reasonable, but the former seems pretty hard to take at face value, what is the basis for rejecting an objective reality, is it anything other than a reductive [3:34:40.6] ____ to take that the existence of reality can truly be proven, how is this useful in practice? So there's a lot to say here.

3:34:48.7 SC: Number one, Wikipedia, as much as I love it, which is very much probably isn't the best place to get an education on post-modern philosophy, especially a sentence that starts something like many post-modernists appear to deny something, right? So it's just full of weasel words. What do you mean? Many post-modern is how many? Which ones? And they appear to deny, so you're saying they didn't actually deny?

3:35:15.7 SC: I think they are the reason why such a sentence like that exists in the Wikipedia article is, probably someone had a belief that postmodernists do deny the existence objective reality, but they couldn't find a quote where any actual postmodernist was doing that. So they kind of made up a sentence that said they appeared to deny it. I think you have not... It's not impossible to find such quotes, but they're not as common as you might think, because most postmodernists, the good ones, are a little bit more subtle than that. The other thing is that most postmodernists are not talking about science, they're not talking about physics or chemistry or biology.

3:35:55.0 SC: Some of them are. There's a whole science studies subfield that takes a post-modern tack on what science does, but when people are willing to say the reality around you is socially constructed, they usually are thinking about in the back of their minds, social reality, social categories, social norms, things like that. And they're the case that it's socially constructed is much easier to make, even when it does come to physics or other scientific things at a strict level, of course, things are socially constructed. The idea of a clark is something that someone invented at some point, someone literally constructed that idea right? Now, it might map on to something objectively real about the world, but the idea of it is socially constructive, so I think that you can listen the closest.

3:36:50.1 SC: I think we came here was Sally Haslanger was on the podcast and talked precisely about the social construction of reality, so you should revisit that podcast. I think you can absolutely believe that the categories that we use to organize reality or socially constructed while the reality itself is real is objectively real. So that's what I would say, and I think I would bet that a large number of post-modernists would actually go along with that, I don't know, I don't have a poll of postmodernist, but I think that to a science oriented person, they seem to be slippery in some of their statements, and a lot of that slipperiness can be accounted for when you keep in mind that they're not trying to talk about or to science, they're trying to talk about the social world.

3:37:38.9 SC: Peter Gale says, Please explain how inflationary theory implies other universes. If indeed it does. Well, it depends on your definition of universe, so cosmologists have adopted a definition of the word universe that they use in the context of the cosmological multiverse, and you might agree or disagree with the definition. I don't really care whether you agree or disagree. I'm just trying to explain what it means. Okay, what it means is, a large region of space where the local laws of physics, so the masses of the particles, how they interact with each other and so forth, are all more or less uniform through the region, okay? So our observable universe is one such universe, you look out at the distant galaxies and the Milky Way background, they clearly seem to be compatible with the same laws of physics that we have here, but in the cosmological multiverse idea, there could be regions very, very far away, that are themselves very large, and the local laws of physics look different. So we call those different universes, then if you accept that definition, Inflation...

3:38:49.1 SC: Often... I don't wanna say always, so I don't think that's true, and I don't even know how often it is, but let's just say often, models of inflation lead to this kind of cosmological multiverse or at least, let's be even more careful than that, open the possibility of this kind of cosmological multiverse, different models of inflation allow. So let's back up, one thing that inflation is supposed to do is to explain or to provide an account of the fluctuations in temperature and density that we see in our universe, including the cosmic Milky Way background. And those fluctuations are small one part in 10 to the five, but it is easy, in fact, it naturally pops out that if inflation happened, there is some regime in which when inflation was happening, the fluctuations were large. Much bigger than ten to the minus five. So large that even though the inflaton field that is causing inflation and is gradually diminishing in energy sometimes fluctuates upward in energy and makes a new universe, keeps going. And this is what we call eternal inflation, because somewhere in space, inflation is gonna keep going, and that's gonna make more and more universes. And that is half the battle.

3:40:01.4 SC: If you want to make a cosmological multiverse, the other half of the battle is, Can you in different post inflationary regions have different local laws of physics, there are many versions of fundamental physics in which you can... String theory is one such version, there's other versions also, but we don't know if any of those theories of fundamental physics is right, so inflation can bring to life different local laws of physics if the fundamental laws of physics allow for such a thing, but we don't know if it happens, so inflation can imply other universes, but it doesn't necessarily have to.

3:40:40.1 SC: Cubit says, Well, I was trying to understand the concept of effective field theories. I wondered about the following point, you have mentioned that the Infinities of quantum field theoretic calculations disappear when formulated on a discrete lattice, having some experience in numerical simulations. That doesn't seem to make any sense to me. Is it really the case that I'm not allowed to choose an arbitrarily fine grid within my simulation because otherwise my results become worse at some unknown point? Well, I think this is... I don't know what kind of numerical simulations you have experience with.

3:41:11.5 SC: There is a difference between classical field theory or fluid mechanics, etcetera, and quantum field theory in this case, and one of the differences is this, in a classical field theory, you expect that the real fields you're looking at, become smoother and smoother as you look at them in smaller and smaller length scales. There is usually some typical scale of oscillation, or maybe there is some smallest scale at which oscillations are really important because very, very small scale oscillations can cost a lot of energy. So they're just not there. In a classical field, you would expect if you zoomed in on a region of space and looked at smaller and smaller regions, you would eventually start seeing less and less motion, less and less divergence from place to place. In quantum field theory it's not like that, because in quantum field theory, you have virtual particles as well as real ones, and for real particles, sure, they would all have some wavelength that's longer than a certain amount, etcetera. But the whole point of quantum field theory is that there are contributions to scattering experiments, etcetera, from Wave modes with shorter and shorter and shorter wavelengths, every wave length keeps contributing.

3:42:28.8 SC: You keep getting new contributions as you go to smaller and smaller wavelengths. Indeed, that is where the infinities come from. So you can very easily imagine a situation where the classical numerical simulation has no infinities in it, you just need to pick a lattice spacing that is small enough that things react relatively smoothly, whereas a quantum version of that, things become infinite. When you get smaller and smaller grid sizes, and what Wilson realized is that in some real sense, those Infinities that only begin to show up as you look at smaller and smaller grid sizes are not really physical. They're not really there. We know that the final answer is a finite number. So the Infinities that showed up in your calculation are artifacts of your calculation, they're not real, and so he invented effective field theory to show you how to get finite answers without ever having infinities at some intermediate point along the way.

3:43:27.3 SC: Colleen Edwards says, with psychedelic drugs like MDMA, mescaline, etcetera, being studied more as potential therapies for treating post traumatic stress disorder, depression and anxiety disorders, etcetera. I'm curious what your take is on this, if you're for it or against it, and why have you ever experimented and if so, were you lucky enough to have had a profound experience on your trip? So. Yeah. I'm for it. I mean, I kind of want to skirt around the question because why am I supposed to know anything, or why are you or why is anyone supposed to know anything about the relative merits of different possible therapies for these kinds of disorders? There's probably lots of different things that may or may not be useful for these kinds of disorders, I think that individuals feel that they have some insight into psychedelic one way or the other psychedelics, some people have very positive feelings about them, some people have very negative feelings about them in ways that they might not have very strong feelings about non-psychedelic chemicals and drugs and things like that. So I feel about them the same way I feel about any other possible therapy or possible treatment for these disorders, try 'em out, see if they work I have no strong emotional attachment one way or the other.

3:44:49.8 SC: I had if you're interested, Robin Carhart-Harris on the podcast where we talked about exactly this, he is someone who does research into psychedelics, and I think that from talking to him and from talking to other people, etcetera, it seems that these psychedelic therapies are potentially extremely useful. There's even kind of a cheap and easy hand way, the explanation for why they are useful, namely that the brain is a complicated thing, it works in a very complicated way, but one thing is that it tries different things, it tries not to fall into a rut, and all of these examples of disorders, depression, anxiety disorders, PTSD, in some way, in different ways, all of these are examples of falling into ruts.

3:45:39.4 SC: And thinking the same thing over and over again, and not being able to escape, and again, some very handwavy way, a common thread of different psychedelics is that they help the brain and escape from its ruts. They let the brain explore a little bit more, and that's why it kind of seems psychedelic and a little hallucinatory, you're exploring things that you don't usually explore, actually those things again at a handwavy level are already there in your brain, and what the psychedelics are doing are lowering your inhibitions to noticing them, they're not creating them in the first place. So if that story holds together then I'm completely willing to think that these are gonna play a huge role, therapeutically, but one way or the other, I want people to do the tests and find out whether they actually work. As I have mentioned previously on the podcast and elsewhere, I did LSD once, that was my only psychedelic experience, and I would say it was fun, but it was not profound.

3:46:37.9 SC: I don't know what... I'm very skeptical of the profoundness of such experiences. I mean, we do know a little bit about what's going on when you do psychedelics. You're lowering the inhibitions that your brain usually throws up to listening into the noise that is going on in the background of your brain at all times. That can be fun. I don't see how it would be profound, I certainly don't see any way that it's giving you any special insight in the fundamental nature of reality or anything like that.

3:47:08.2 SC: Kyle, Steven says Isaac Asimov Foundation series describes the future where knowledge becomes fragmented and specialized to the point where scientists become increasingly focused on narrow topics and the broader understanding of science is lost. Do you think this fragmentation is happening today, is any of your motivation to be cross-disciplinary to keep this from happening? I don't think I can keep this from happening all by myself. I do think it's happening. I have mixed feelings about it. Look, I think you have to be honest about the fact that science is still progressing at an amazing rate, we're doing wonderful things in science, there are absolutely good reasons why scientists hyperspecialize.

3:47:47.9 SC: There's too much knowledge out there. You can't be an expert in everything. If you had to pick one single way to be a scientist and make every scientist be that way, the best way would be to hyperspecialize in different areas so that we could move forward in all these different areas. Now, we don't need to make a rule where every scientist has to be the same kind of scientist, we could imagine that there are some scientists whose job it is to hyperspecialize and push particular areas forward, and others whose job it is to be more generalists and understand multiple areas and try to draw connections between them. I would vastly prefer if academia were set up to encourage both modes of being a scientist, maybe most scientists or hyperspecialists, but there's some out there whose literal job it is to be interdisciplinary and to draw connections between them, that's not how academia is set up. I think that's a shame. I think one of the reasons why the Santa Fe Institute is so successful scientifically is that they actually do encourage this and they allow for it, it's one of the goals we have at Hopkins in setting up the natural philosophy forum. That we want to provide a place where scientists of different kinds and philosophers can get together and be cross disciplinary in an environment where that is encouraged, where that is actually the point rather than merely being tolerated.

3:49:17.4 SC: Robert Rux and rescue says, when you talk about Schrödinger's cat and you present the two scenarios where the cat is awake and standing and a sleep and sitting, and you mentioned the decoherence happens, the atoms and the cat are different in the two positions in the two scenarios. But what if the awake and the sleep cat are in exactly the same position, maybe both of the two versions are sitting still with their eyes closed, etcetera. The only difference is their conscious state, one is awake, the other is asleep, what then? Well, it's not a handwavy thing, okay, it's, do you become entangled with your environment? That is the question. The reason why it's nice to talk about the case where you're distinguishing between a cat that is awake and walking around, versus the cat that is asleep and lying on the ground, is because there it's obvious, that you do become entangled with the environment because photons hit you in different ways, when that's not so obvious, then you gotta do more work.

3:50:10.3 S2: You have to think about what is exactly the difference? I mean, clearly, there's some difference between the awake cat and the asleep cat, maybe their eyes are twitching at different rates. Certainly what's going on in their brains is happening at different rates, which might emit different kinds of infrared light or something like that, I don't know the details. That becomes a complicated biology question, but, I strongly suspect that they would decohere quite quickly, but you would have to actually do the calculation one way or the other, once one of them woke up, then they would be very, very different.

3:50:45.0 SC: Eric Fast says, I know that I should have a credence of one in 1024, that a fair coin will land heads 10 times in a row. I might have exactly the same credence that we will prove the Twin Prime Conjecture in the next century, but these intuitively feel like very different types of credences. In the second example, I'm uncertain whether I should have something closer to one in 10 or one in 10 million, is it enough to represent the credence as a number or should we use something like a set of probability distributions that capture our higher order uncertainties?

3:51:17.4 SC: I probably should have grouped this question, sorry to Eric, that's very down here late in the AMA, but I should have grouped this back up there talking about the very lower question and self locating uncertainty in many worlds. Here is my take on it, but I'll tell you that my take is not fully developed and maybe... There's two possibilities, one is that there is a take out there that is fully developed and I would sign on to. Or number two, I need to write a paper about this where I actually fully develop my take. But my take is that the kinds of Credences they are the same in both cases. One is a credence about the future of our universe, right? Where, and we don't know the future of our universe in terms of coins being flipped, maybe we live in a universe where you're gonna flip the coin and get the heads every time, but we don't know, okay? We nevertheless have a seemingly rational objective procedure for assigning that credence. So the differences between that and the Twin Prime Conjecture is not that is a different kind of credence, but that we have a different way of assigning it.

3:52:24.8 SC: You know what I mean? We have a handle out there objectively in the world that suggests to us that we should, if we have some objectives assigned credence in a certain way, whereas in the Twin Prime Conjecture case, we don't have those, we don't have anything to give us that objective way of developing the credence. So I would shift the difference, there is a difference in the two cases, but the difference is not in what kind of credence it is, but how we develop it, and I think that's okay. That would be the part that I have to develop more. I don't see any reason why that is a reason to feel uncomfortable about the assignments of these credence or treating them as the same kind of underlying uncertainty.

3:53:07.5 SC: Carlos Nunes says, Robert Sapolsky just published a book showing that we don't have free will, I don't think you did that, but he did publish a book, given your compatible stands, what evidence would persuade you to reject the notion of free will? Sure. I mean it's sort of as we discussed earlier, a better theory, a theory that fully accounted at least as well as ordinary psychology does for the behavior of human beings, that doesn't have the fact that human beings make choices as part of the theory. All you have to do is invent that theory, show that it matches the data I will sign on, I will give up on free will.

3:53:45.1 SC: Blake Sure says, is irreversibly just a function of our ignorance or fundamental? Well, we don't know the fundamental laws of physics, so that's always the caveat here. So if it's very possible that the truly fundamental laws of physics, Stephen Wolfram, for example, are just irreversible. I don't think that's the most likely thing. It's certainly not what we have right now. In many worlds, quantum mechanics, the fundamental laws are reversible, and then it just becomes a function of our ignorance. To me, like Sarah Walker and Lee Cronin in the Assembly theory thing, or like Philip Golf in the Panpsychism thing.

3:54:25.8 SC: Are leeping too quickly. From, there something about this microscopic phenomenon that's a little bit mysterious. To let's monkey with the fundamental laws of physics, you don't have to do that, there's enough room in our ignorance to say Some day we'll figure it out, I see no obstacle whatsoever in completely accounting for all of the irreversibility that we see here in the world in terms of the good old laws of physics, as we already know them. Boltzmann explained it to us, he kinda got it right. But of course, we'll have to see, maybe our notion of the fundamental laws of physics isn't in its final form yet.

3:55:01.0 SC: Zachary Danziger says, how useful do you think cellular automata are as a metaphor for physical processes or computation, E.G Conways Game of Life. Or is the actual substrate for theory physical law, e.g., Wolfram's rule had? Well, I think they're very useful as a metaphor. I think they're very useful as ways to learn things about certain, nice physical systems. I think the nice thing about cellular automata is they're easy to simulate, they're easy to put on a computer and see what happens, so you can study emergence in them. For example, one of the most famous philosophical papers on emergence is by Mark badal called weak emergence, where he really gets into the idea of weak emergence. And he uses Conway's game of life as an exemplar of exactly this, so it's very educational in that sense as an actual substrate for a theory of physical law, I think it's highly un-promising. I think that we have a situation right now where we... Well.

3:55:58.1 SC: The thing that bothers me the most, there's two things that bother me the most about cellular automata as an actual substrate for physical law. Number one, it takes locality too seriously. The whole idea, the hypergraph and Wolfram's recent physics stuff are not quite as wedded to the idea of locality as the original cellular automata theories were. But still it takes a little bit too literally, the idea of objects with locations in space, and I think that quantum gravity and Quantum mechanics more generally, warn us against that, and not everyone is listening to the warning. That's one thing. The other thing is, it doesn't take the idea of reversibility seriously enough. I think that the fundamental laws of physics since the time of Newton have been reversible. We understand why the universe. Looks irreversible to us on the basis of thermodynamics and emergence and the second law and the past hypothesis.

3:56:51.7 SC: The idea that you're gonna change the fundamental laws to go back to being irreversible like they were in Aristotle's time does not seem to me to be a step forward, I think you're cheating, not cheating, but I think that you're making your life easy at a calculational level while ignoring some of the realities of physics as we truly understand it. I could easily be wrong about that, but that's my feeling.

3:57:19.7 SC: Wayne J2B, that's quite a name. Asks, Nobel Prizes loom large in the public imagination. Is it true for physicists as well, especially when they start out as grad students or junior faculty, to what extent was your career affected by a desire to work on problems or to go places where you'd have a shot of doing ground breaking enough work to give you a chance at the Nobel? And Do you finally seek really hope that that you're working the foundations of physics will lead to a nobel? So let's be very clear, my work in the foundations of physics is not going to lead to the Nobel Prize.

3:57:49.4 SC: I know that for very, very high credence, it's just not the kind of work that even if it's really good and really successful leads to the Nobel Prize. The Nobel Prize is not the prize for the most important science that gets done. It's a very specific kind of prize, Ed Witten is never going to win the Nobel Prize. Stephen Hawking never did. They did very important work, but it was too far away from the experiments of their time to be able to win the Nobel Prize. And I think that most people who become scientists, not everyone for sure, but most figure this out very early. I remember very clearly as an undergraduate, I had it figured out that the Nobel Prize was not the thing to aim for, a lot of the Nobel Prize winners just get lucky. I mean Penzias and Wilson, we're not looking for the cosmic microwave background, but they found it they 100% deserve the Nobel prize 'cause they are the people who discovered it. But it wasn't because they were especially insightful, they were careful scientists, but they weren't even looking for what they found, how in the world do you arrange your career to maximize the chances of something like that happening?

3:58:57.9 SC: Even my friends, Saul Perlmutter, Adam Reiss, Brian Schmidt who won the Nobel Prize for the Accelerating Universe. They weren't looking for the accelerating universe, they were measuring the deceleration of the universe, they thought, they thought it was gonna be a really important result, but they didn't think it'd be the Nobel prize 'cause they thought there would be a decelerating universe. Right? So all power to the people who win the Nobel prize, but I understood very early on that that was not the way to organize your work. Which is not to say that I was ever especially good or clever at organizing my own research career, but I knew better than that.

3:59:35.4 SC: Nick gal says In your episode with Tyler Cowin... Cowen, I think you mean. You said we still need to understand what the laws of physics are that push the way function of the universe forward even once you've given the many worlds. Could you say more about what the laws of physics are needed to describe what drives the dynamics of the wave function? I.e., Provides the oomph. Yeah, so I'm not sure, Nick, exactly what your level here is at understanding, but the answer is the Schrödinger equation.

4:00:03.8 SC: Okay, so in the many worlds version of quantum mechanics, there's only one equation, it's the Schrödinger equation. The Schrödinger equation does exactly this, it tells you how the wave function of the universe evolves forward in time, and the way it does it is, it proposes a proportionality between the rate of change of the wave function, the time derivative of the wave function. And something called the Hamiltonian operator acting on the wave function, so you can think of the Hamiltonian operator as asking a question, how much energy is there in different parts of the wave function, and then it says, each one of these parts is going to evolve at a certain rate, depending on how much energy it has. The more energy, the faster it evolves.

4:00:45.0 SC: So that Hamiltonian in this picture, conveys... No. Contains all of the laws of physics, the Hamilton... The question that is, how fast is the wave function evolved, in what ways does it evolve is entirely answered by telling me what the Hamiltonian is. So the Hamiltonian is the laws of physics, and the Hamiltonian is a big matrix. Big means if you tell me the dimensionality of Hilbert space, call it capital N, then this Matrix is an N by N Matrix, we can always choose a basis for the Matrix where it's diagonal and it's just the N numbers on the diagonal of that N by N matrix, but it's a lot of numbers, if you have a big Hilbert space and those numbers tell you all you need to know, so from this perspective, the challenge, the interesting part is going from that matrix or this list of numbers on the diagonal to the specificity of the world to the fact that we have three dimensions of space, that we have certain gauge theories and stuff like that, that is the project that is currently being investigated.

4:01:49.8 SC: Red Antonov says, Do you have a favorite topic to teach? Well. No, I can say this. Probably the single most intellectually rewarding topic that I've ever taught is general relativity, it's not a coincidence that I wrote a textbook about it, I taught it before I wrote a textbook. So that's not because I wrote the textbook, but General Relativity as everyone will tell you, it's just beautiful and self-contained. Now, it does involve math, the challenge in teaching General Relativity is you spend a certain number of weeks early in the course just kind of doing math. I guess if you're super good at teaching, you could figure out how to scatter in physical applications along the way to keep people interested, but some of your students are gonna love the math, they're gonna think that that's great. Others are going to say, Why are we doing this? How do you connect this to the real world, and they're gonna get impatient during that part of the course, but by the time that part is over, and you give them Einstein's equation for general relativity and then you start putting it to work in cosmology and black holes and gravitational waves, it is both super rewarding because all of these topics are manifestly relevant to Modern Physics and to the world, but also super logical and self-contained, general relativity has almost no pre-requisites, you even teach Special relativity in the first week and that's fine.

4:03:14.6 SC: Special Relativity doesn't take a lot of time to teach. You need to know partial differential equations, it helps you if you know some Linear Algebra, but there's not a lot of that stuff, right? General activity is its own thing. Whereas if you teach cosmology that I've also taught every week, it's a different thing, this week you need thermal dynamics, this week you need Particle Physics this week you need statistics of large hail structure. And it all does add up, but hang together but it's not this cumulative sense of progress that you get when teaching general relativity. So if I had to pick one, it would be that. On the other hand, what I'm doing right now, like in a philosophy course, sitting around the table with 12 people really thinking hard and discussing ideas, that's also amazingly wonderful for separate reasons, so that's why it's hard to pick a single favorite topic.

4:04:06.9 SC: Chris Murray says in the road to reality, one objection Roger Penrose makes to many worlds is that an entangled state can be represented equally well as sums of many different pairs of states, and that there is no mathematical preference for which payer should count as the possible observations. What do you say to that? Even if correct, wouldn't this problem be shared by all interpretations? Maybe it'll be shared by all interpretations, maybe not. I can't speak for all interpretations at once, but this is absolutely not a problem for many worlds. As long as like me, you take the attitude that you don't have separate worlds until you have decoherence, and when you have decoherence, the question is answered.

4:04:45.1 SC: There is a... If you have a system and an environment, the whole thing, the whole shebang system plus environment is described by a quantum wave function, but the system by itself is described by a density matrix. You can sort of trace over, you can integrate out or ignore the environment and just talk about the system. And then there is a unique, very clear basis for that density matrix in which its diagonal, and that means that those are the worlds, so the question of What are the worlds? What basis should you represent them in? Is not ambiguous at all. You only get into trouble if you think that many worlds is saying that even before decoherence, there are many worlds, then you can't pick out what the worlds are and it would sound confusing, but that's not what I do. That's not what I think most people do when they talk about many worlds.

4:05:37.0 SC: Physics Kitten says, Can you help me understand why quantum computers are so much faster, at least for some applications, it can't be the amount of information being handled a bit has two possibilities 0 and 1, and a cubit has just one more possibility, spin up, spin down in a superposition of spin up and down. Well, I think there's two things going on here. As you said, I'm glad that you said, at least for some applications, quantum computers are not necessarily faster for every application, there are some applications for which they definitely are faster at least, if you give me the same number of qubits that the classical computer has in bits which is not the real world. The big quantum computers have dozens of qubits, the big classical computers have many, many more bits than that. So, and there's also problems, this is not answering your question, but there will be problems in scaling the quantum computers, it's hard to get many, many cubits entangled and working together without deco hearing, so that's a challenge for future quantum computing, but there are two ways in which quantum computers give you more oomph.

4:06:42.1 SC: Give you more handle on what can happen, One is that it's not true that a cubit has just one more possibility, if a bit has two possibilities zero or one, a cubit has an infinite number of possibilities. It is a superposition of spin up and spin down, but it's not necessarily an equal superposition, it is alpha times spin-up plus beta times spin down, where Alpha squared plus beta squared equals one.

4:07:09.6 SC: Or if you want, you could write it as alpha times spin up plus the square root of one minus alpha squared time spin down. But then Alpha can be any number between zero and one, in fact, that's up to a complex phase we can get rid of. So let's ignore that, but there are an infinite number of numbers between zero and one, you can have just a little bit of spin up a good amount, but not a lot, a lot. All of these are real possibilities that's enormously more flexible than a classical Computer, and then more importantly, but also true, the cubits can be entangled with each other, classical bits are not entangled, and you have a possibility in a quantum computer of having unentangled cubits and in fact, the quantum computer can do everything that a classical computer can do just by sticking to qubits that are either pure spin up or pure spin down and cubits that are unentangled, but the qubits can be in these arbitrary super positions.

4:08:06.5 SC: So that gives you more flexibility and they may or may not be entangled with each other, that gives you a whole another degree of flexibility. So in order to understand the precise way in which quantum computers actually are faster, it would help to look at a certain algorithm that is faster, and I encourage you to do that, but the fact that they can be faster should not be surprising. They're just much more flexible than classical computers are.

4:08:33.8 SC: Amy Ferguson says, when it comes to your thought process, do you feel as if your thoughts emerge out of nothingness, or is a sense that you actively construct them? Well, it's complicated. Can you talk about the brain. It's always complicated, right? They don't emerge out of nothingness. There's certainly something going on in my brain, This is one reason. Look, there's a very big reason why I do the podcast, Okay?

4:08:58.8 S2: The reason why it's nice doing the podcast, I like people listening to the podcast, that is true, I like having an audience, I like the feeling that I can talk to people and share some information with them, but deep down, I'm a selfish person and I just wanna learn things myself, and this is why... This is why I do not optimize the podcast for listenership, I could much, very easily, in various ways, increase the listenership of the podcast, one way would be just to make it purely physics. I'm not unaware of the fact that if you just focus and give value to your audience by having a strong brand identity, such as, this is the podcast to go to for the most recent word on modern physics, that would be a bigger sell to a lot of people, it would also be less useful to me. I get an enormous benefit from talking to all these different people about all these different kinds of ideas, and as we said before about the thought processes of individuals versus collaborations. You might not know what the benefit is.

4:10:07.2 SC: Very often, I will confess, I've done over 250 individual interviews with people on the podcast, I don't remember all of them. While I'm doing them, I don't even remember the day later what we talked about in great detail, because while I'm doing them, I'm thinking about the audio levels and the sound quality, and what question I'm gonna ask next and how much time is passed and all this stuff so I can often listen and be curious and engaged, but I'm not 100% paying attention to. As if I were listening to a podcast, right? But nevertheless, it makes an impact on my brain, I am thinking about what they say, even as I'm worried about audio levels, I'm absolutely listening to every word the guest says, I might not be consciously remembering it as well as I could, but unconsciously. I think it makes an impact. So anyway, podcast aside, I read books and I go to talks, and I read things on the internet and things like that, and it all feeds in, in ways that are impossible for me to reconstruct into the ultimate thoughts that I have. Now also, I play a role in actively constructing them to the extent you're getting into deep waters here about the self and I and free will and things like that, but there's a part of my brain that shapes the inputs that it's getting both from internal and external sources.

4:11:34.1 SC: So I think both happen, is the answer you can absolutely get insight by just going to sleep and waking up with better ideas, or by thinking about X and getting a better idea about Y. By distracting yourself, I think there's all sorts of techniques that you can use to give yourself a higher probability of getting a good idea about something, it's hard to predict exactly when it will happen.

4:12:00.4 SC: Brandon Lewis says, Suppose I have a collection of nuts and bolts placed randomly into little bins, if I sort the collection by size, thread, pitch head type, etcetera, has the mass of the collection increased by the corresponding decrease in entropy, or is this just a complete misunderstanding of the way entropy works, I think it's an intriguing suggestion, but at the end of the day is a misunderstanding of the way entropy works, there's no interaction, is the point between the different kinds of nuts and bolts you have in your little bins. If I have a fluid, I have oil and water in a glass, they interact with each other, the energy of the oil and water molecules depends on their configuration, and then there is a relationship between the energy or the mass of the fluid and its entropy. But for the nuts and bolts, if I take two nuts and a bolt and I put them in different arrangements, they're not bumping into each other. Yeah, I presume that you're not at very, very high temperatures where a literal macroscopic nut or bolt is bouncing around and bumping into each other, they're probably just sitting there on the table, right. In that case, there's essentially no interaction between them, the energy is completely independent of how you arrange them. So, no relationship there. Sorry.

4:13:18.9 SC: Okay, final question of this AMA comes from Sidd Huff. I don't know if we should give a prize for the final question. I do, I do, I'll confess right here as a reward, everyone who sat through this long thing, I do try to pick a good final question, I try to get a question that I think has something interesting for me to say, I don't wanna have the last question of the AMA, be like, Did you read this book? And you like it, and I to say No, I didn't read it. That would be a boring final question. So Sidd Huff says, Stephen Hawking once declared that philosophy is dead because it has not kept up with modern developments and science, particularly physics, for saying this, others accused hawking of Scientism IE putting an overly high value on science relative to other non-scientific disciplines, but is this really a bad thing?

4:14:04.6 SC: If say, physics is a better way to understand the world. Should it in fact be more highly valued than say philosophy and other such primarily non-scientific disciplines. Yes, it is a bad thing to be that kind of scientific-ness. I think that the word Scientism is thrown around and is more or less useless, you notice that I never use that particular word, it's just like an insult, it's not a substantive word that is telling me something useful about what is going on. But I do think that the quote from Hawking philosophy is dead because it's not kept up with modern developments of science, is utter horse shit. For one thing, philosophy has kept up with modern developments in science for another thing. Philosophy is not dead for a third thing, I know for a fact that hawking put that in his book. Just because people would talk about it and it would sell more books. Okay? He knows better than that himself, he doesn't know it now because he's no longer with us, but he did... He knew better than that. He was just trying to be provocative. And it worked. You fell for it.

4:15:04.2 SC: Other people fell for it. Also, there is an analogous or related or adjacent or nearby statement you could make that does matter, that does have some truth to it. I don't think that all philosophers pay as much attention to developments and science as they should. Famously, I'll give you an example of a truly great philosopher, David Lewis, we've already mentioned because of his work on moto realism in possible worlds, he famously has a quote that says, he does not want his metaphysics to be informed by developments in Physics, and he's right there saying, I don't want this to happen, I get the reason why he would say that, because metaphysics is supposed to be prior to physics is supposed to the logical structure of possibilities, not the actuality that physics cares about. But it is clearly ignoring the fact that when we human beings try to sit back and come up with metaphysics, try to come up with the logical structure of possibilities, we are terrible at it, we constantly make mistakes.

4:16:09.2 SC: If you are honest and humble, you will realize that paying attention to physics can help you correct those mistakes that you might have made. Maybe an ideal, perfect metaphysics made by beings that never make mistakes would not need input from science, but real world metaphysics does. So I think that's a valid criticism, but that's not what Hawking was getting at and what Hawkins getting at, is not true. To go to specifically Sid's question, I don't think... I mean I don't know what it means to say, physics is a better way to understand the world. Is physics a better way to understand biology, than biology is? I think that physics is a better way to understand physics. Is physics a better way to understand logic or ethics or aesthetics than philosophy is? No, I think that the only possible way that anyone could even utter such a thing is to confuse what philosophy tries to do with what Physics tries to do.

4:17:11.8 SC: Physics tries to do different things than philosophy tries to do. I mean, a classic mistake that scientists make when they complain about philosophy is they say, Well, I've never been affected by philosophy, the philosophy has never helped me in my science. It never seems to occur to them that that's not the job of philosophy to help them. Their job is to do science, and that's fine, they will always be doing philosophy, you can't do physics or other science without doing philosophy, you can only do it badly. The happy news is that for the vast, vast majority of ways of doing physics or questions you might wanna answer in physics or biology or anywhere else, you can do fine doing philosophy badly in precisely the same way that for the last 100 years, we've been able to do quantum mechanics fine with the Copenhagen interpretation, you can build a large Hadron Collider, you can make lasers, you can do a wonderful collection of things, even though you're doing Quantum Mechanics badly, because sometimes the question you're asks doesn't need that much precision, you don't need quantum mechanics or general relativity to plot a trajectory from the Earth to the Moon, and likewise, most physics questions don't need really subtle philosophical insight, however, that fact tends to blind physicists and other scientists to the fact that some of their questions kind of do require careful philosophical insight and they inevitably get it wrong.

4:18:48.0 SC: The foundations of quantum mechanics and the foundations of statistical mechanics are obvious examples, but other examples come about in the notions of probability and fine-tuning and naturalness, and the multiverse and testability and falsifiability, all of which physicists like to talk about in all of which they talk about very badly. Because they are not educated in philosophy, so I don't think there's a competition between Philosophy and Physics and other ways of understanding the world to see who is the best. I think that depending on what question you're asking, you should use whatever disciplines are appropriate to that question. And with that, I bid you a due for this month, this is the November AMA. We will have a December AMA at the beginning. Remember that we don't have a January AMA 'cause it's Christmas time, it's vacation time here at Mindscape World International headquarters. So come up with good questions for next month and I will talk to you then bye bye.

[music]

3 thoughts on “AMA | November 2023”

  1. If I don’t know anything about tensor and special relativity, how would you describe me the Einstein filed equation?

  2. There’s a difference between consciousness, sentience, and awareness, and everything going on with Ai will force us to a greater clarity / understanding of what those things actually mean. While it’s an amazing time for science and technology, I think it is also a bell weather moment for philosophy. Is consciousness an inevitability of ever evolving complex systems? IE, complex computer networks AI will ultimately develop consciousness ipso facto or is there something germane to biological systems that creates this “sense” of awareness.

Comments are closed.

Scroll to Top